Sie sind auf Seite 1von 76

CUPRINS

Cuvnt nainte din partea colectivului redacional


____________
Membrii Filialei Mehedini a S.S.M.R.
________________________
Per aspera ad astra - Eduard Bzvan
__________________
Andrei Ungureanu - la finalul carierei olimpice de elev n Romnia _

1
2
3
5

A doua ediie a Concursului interjudeean Petre Sergescu


Programul Simpozionului National ______________________________
Subiectele de concurs __________________________________________
Rezultatele concursului __________________________________________
O generalizare a problemei celor 12 bile
________________________
Elemente de probabiliti geometrice
________________________
Asupra unor identiti trigonometrice condiionate
____________
Generator de probleme __________________________________________
Primul concurs judetean de geometrie Gheorghe ieica
______

7
9
12
15
17
22
23
25

De la Severin la Rutgers

27

____________________________________

Teme pentru grupele de performan


Clasa a V-a
Metode de rezolvare a problemelor de matematica
____________
Clasa a VII-a
Formule de calcul prescurtat ____________________________________
Clasa a VIII-a
Probleme de algebr cu soluii geometrice ________________________
Clasa a IX-a
Algoritmul Plicului
__________________________________________
Clasa a IX-a
Teorema lui Casey
__________________________________________
Clasa a X-a
Comentarii metodica asupra unor probleme date la Olimpiadele SUA
Clasa a X-a
Tetraedrul i capcanele lui
____________________________________
Clasa a XI-a
Asupra teoremei lui Knaster
______________________________
Clasa a XII-a
Caracteristica unui inel
____________________________________
Clasa a XII-a
Teoremele lui Sylow
__________________________________________
Clasa a XII-a
Asupra polinomului caracteristic
______________________________
mATh Studio

29
32
33
35
38
40
42
44
45
49
51

________________________________________________

55

Probleme propuse
____________________________________
Premianii concursurilor din 2006 ______________________________
Rubrica rezolvitorilor
____________________________________

56
62
67

-0-

REDACTIA

SSM
H

-1-

EDITORIAL

SSM
H
Cuvnt nainte din partea colectivului
redacional

Aparem astazi in fata cititorilor nostri cu al saselea numar al revistei


RMM consecventi propunerilor initiale de a aduce in paginile revistei
evenimentele matematice din judetul nostru atat cat ele sunt cunoscute prin
intermediul colaboratorilor nostri membrii activi ai Filialei Mehedinti a
Societatii de Stiinte Matematice din Romania. Pentru a stimula si alti colegi
profesori de matematica si nu numai sa ni se alature in activitatea noastra
avand ca principal obiectiv asa cum reiese din statutul Societatii sa ridicam
nivelul cunostintelor de matematica ale tinerilor , publicam in acest numar
membrii Filialei la 31 decembrie 2005. Reamintim cititorilor ca prima
conditie de a deveni membru al Societatii noastre este de a achita la zi
cotizatia de 10 lei pe an. Consiliul Filialei va avea de grija ca acesti bani sa
fie folositi in conformitate cu statutul si hotararile luate in Adunarile
Generale ale Filialei Mehedinti a SSMR.
Anul 2006 a debutat din punctul nostru de vedere cu a doua editie a
Concursului Interjudetean Petre Sergescu, ecourile acestui eveniment le
veti gasi in paginile revistei noastre. De asemenea folosim paginile revistei
pentru a anunta ca pe 19 Ianuarie 2007 elevii claselor IV-XII impreuna cu
cadrele lor didactice sunt asteptate la cea de-a treia editie a concursului.
Persoane de contact: prof. Gheorghe Ciniceanu - gicadoc@yahoo.com , prof.
Prajea Manuela prajeamanuela@yahoo.com sau prof. Dan Nnui
dan_nanuti@yahoo.com . Pe 31 ianuarie s-a desfasurat pentru a cincea oara
in Colegiul National Traian American Mathematic Competition, rezultate si
comentarii veti gasi de asemenea aici. Concursuri interjudetene, Olimpiada
judeteana , Olimpiada nationala, Balcaniada, Olimpiada internationala,
toate sunt reprezentate in paginile revistei noastre prin prisma rezultatelor
obtinute de elevii nostrii.
Nu putem trece mai departe fara sa amintim aici ca, anul 2006, a
reprezentat prin medaliile de aur la OIM ale elevilor nostrii de exceptie
UNGUREANU ANDREI BOGDAN si BAZAVAN EDUARD, cel mai bun an din
istoria concursurilor de matematica pentru judetul nostru.
Ne bucuram ca in paginile Gazetei Matematice Metodice Nr.2/2006,
revista noastra este recenzata alaturi de alte publicatii prestigioase pe taram
matematic din tara. Inauguram in acest numar o rubrica noua, si anume
prezenterea unor scurte biografii a unor elevi de-ai judetului nostru care cu
ajutorul matematicii si-au croit un drum in viata in tara si pe alte
meleaguri. Fac apel la colegii nostrii ai caror elevi de exceptie au ales o
cariera legata de matematica sa ni se adreseze astfel ca publicand aceste
biografii sa stimulam pe mai tinerii matematicieni in activitatea lor de studiu
si cercetare.
In speranta ca si acest numar al revistei noastre dupa ce va fi ajuns
pe mesele de lucru ale matematicienilor mehedinteni si nu numai va
reprezenta un prilej de satisfactii intelectuale va adresam rugamintea sa va
alaturati noua in viitor atat prin materiale interesante cat si prin
popularizarea revistei in randurile elevilor si cadrelor didactice.
Presedintele Filialei Mehedinti a SSMR,
Profesor doctor Gh.Cainiceanu

-1-

EDITORIAL
Membrii Filialei Mehedini
a S.S.M.R.
- 31.XII.2005 -

1
2
3
4
5
6
7
8
9
10
11
12
13
14
15
16
17
18
19
20
21
22
23
24
25
26
27
28
29
30
31
32
33
34
35
36
37
38
39
40
41
42

-2-

Albu Elena
Antonie Rodica
Badaluta Anghel
Bala Dumitru
Baloi Valeria
Balu Nicoleta
Barbulescu Marin
Bejenaru Laviniu
Bizdoaca Claudia
Bobic Nicolae
Bogdan Dorel
Bondoc Gabriela
Bondoc Lucian
Bosneagu Dina
Budanescu Felicia
Buzatu Carmen
Cainiceanu Gheorghe
Calafeteanu Gheorghe
Carbunaru Dumitru
Chilea Ion
Ciuca Ionel
Coada Carmen
Constantin Magdalena
Cristel Elena
Croitoru Ion
Dan Daniel
Doman Elena
Draga Tatucu Porfirel
Dutu Constantin
Fallon Florica
Farago Alexandru
Florescu Violeta
Fritea Eugen
Gimoiu Iuliana
Giugiuc Constantin
Giugiuc Leonard
Gorun Sanda
Grecu Vasile
Hinoveanu Sorin
Ianasi Ion
Ionescu Adela
Lazar Alexandru

Sc.9
CNT
Sc.6
C.Univ.
Odobleja
Odobleja
Baia
CNT
C.E.
Lalescu
LM Orsova
L.Auto
Tribunal
Sc.6
Sc.9
Dl.Tudor.
CNT
Sc.9
Sc.4
Dl.Tudor.
Dl.Tudor.
Sc.2
Simian
Dl.Tudor.
C.E.
ieica
Baia
Simian
Sc.2
LM Orsova
Lalescu
Sc.4
Dl.Tudor.
CNT
Dl.Tudor
CNT
Lalescu
C.E.
Sc.6
Sc.4
C.Univ
Baia

43
44
45
46
47
48
49
50
51
52
53
54
55
56
57
58
59
60
61
62
63
64
65
66
67
68
69
70
71
72
73
74
75
76
77
78
79
80
81
82
83

Lugoj Tanta
Lupu Adrian
Mada Gheorghe
Malineanu Gabriela
Marin Felicia
Moclea Patrutescu Adr
Nanuti Dan
Nedeianu Dan
Oprita Manuela
Paponiu Dana
Pasov Nicoleta
Paulescu Petre
Petrache Elena
Pit Marica
Pit Vasile
Ploscaru Cornel
Popescu Eleodor
Popescu Marcel
Popescu Rodica
Prajea Manuela
Presneanu Doru
Pupaza Ecaterina
Raducan Emilia
Ramniceanu Elena
Sitaru Daniel
Stoican Victor
Stretcu Daniel
Stuparu Dragos
Tatucu Mariana
Ticusi Ovidiu
Tomita Vasile
Tudorescu Loredana
Tudose Elena
Ungureanu Octavian
Untaru Ilie
Vaduva Ion
Varzaru Mariana
Vasilcan Florentina
Vasilcanu Octavian
Vasiluta Ion
Vuc Ionela

Decebal
Decebal
Decebal
Sc.4
Sc.9
Sc.11
CNT
Dl.Tudor.
Decebal
CNT
L.Auto
Baia
LM Orsova
Sc.5
CNT
Baia
CNT
Odobleja
ieica
CNT
Odobleja.
Decebal
Decebal
Sc.11
C.E.
Baia
ieica
C.Univ.
ieica
Odobleja
C.Univ.
Baia
LM Orsova
ieica
Odobleja
Odobleja
L.Auto
Dl.Tudor.
Sc.6
Dl.Tudor.
Lalescu

EDITORIAL
Per aspera ad astra

SSM
H

Eduard Bzvan

elev Nol Negrea,


Liceul Teoretic ,,Gh. ieica

M ntreb cte epitete am putea aduga, chiar i n joac, numelui Eduard Bzvan. Nu sunt
sigur. tiu doar c, orict m-a strdui, a lsa pe dinafar prea multe. E lucrul care m determin s
nu pornesc n ,,temerara odisee sortit ab initio eecului. Chiar dac, printr-o minune, a reui s le
gsesc pe toate, rezultatul ar fi acelai: detaarea discret cu care destinatarul i-ar declina tcut
meritele, ca pe lucrul cel mai natural de pe lume...
Am avut privilegiul nemsurat de a m gsi o vreme e drept, nu att de mult pe ct mi-a fi
dorit! - n preajma unui om care a neles ca acceptarea adulaiei celorlali l-ar supune la un efort
extenuant i nociv pentru drumul pe care i l-a croit prin truda minii ctre o lume n care nu i-a gsit
locul dect percepia obiectiv. i asta pentru c, de ani buni, Eduard nvase s nu admit nruirea
carierei - pe care i-o cldea cu minuie prin alterarea atitudinilor propriei mini.
Ce pre va fi trebuind s plteasc pentru rbdarea i efortul lui ingrediente indispensabile
reuitei fr cusur - nu vom putea ti poate niciodat... In schimb, tim cu certitudine altceva:
Fr s se teama a fi, chiar i pentru o singura clip, excentric n opinii i fapte, Eduard nu a
fost niciodat, ceea ce un matematician prolific - dar i super excentric precum Pl Erds numea un
dispozitiv de turnat cafea n teoreme... Dac s-ar fi ntmplat asta, nu am fi aflat pn azi c activitatea
intelectual corect dirijat nu denota o deficiena vital sau un handicap, ci aa cum, cu discreie ne-a
fost revelat, un mod superior de a fi, un privilegiu i nicidecum o caren... Mai mult, a fost un fericit
prilej ca s tlmcim fr gre ceea ce Aristotel spunea n Metafizica: nainte de orice, tiinele
matematice exprim ordinea, simetria i limitarea, iar acestea sunt cele mai mree forme prin care se
manifest frumosul.
Dac ar fi s numim doar un singur merit pe care l are Eduard Bzvan, am putea afirma c i
datorm transformarea matematicii pure ntr-o miestrit poema de idei logice. E idealul pe care,
citndu-l pe William James, doar cei alei au reuit s l ating: fervoarea mpletit cu simul msurii
dublate de pasiunea pusa n slujba gsirii adevrului pe calea cea mai dreapt cu putin i care nu are
nevoie s se sprijine pe nici un subterfugiu ca s conduc invariabil spre rvnitul deziderat - excelena
suprem n dezarmanta ei simplitate. In numele ei st astzi mrturie o galerie impresionanta de
premii, medalii i diplome care se vor fi adunat laolalt de-a lungul anilor de coal. Pentru a cuantifica
totul, ai avea nevoie de timp ndelungat i rezultatul ar fi, fr ndoial, acelai. Ai ajunge s vezi pentru a cta oara? ct de just ar fi observaia pe care Eduard o face de fiecare dat cnd, ntr-un
fel sau altul, cineva ajunge s vorbeasc despre meritele lui incontestabile : Pentru mine, rsplata cea
mai mare e o ipostaz care m obliga la mai mult!
Modestia sa nnscut este dublat de floarea rar a recunotinei fa de oamenii care i-au
marcat destinul. Consider c profesorul de matematic din liceu, domnul Daniel Stretcu, este omul
providenial care l-a susinut, stimulat nencetat pentru a atinge perfeciunea pur a matematicii.
Dialogurile platoniciene au fost strmutate cu succes n solul tiinelor exacte.
Atunci cnd a decis s urmeze cursurile Politehnicii din Bucureti pentru a deveni inginer, a fost
ntrebat, n glum, dac este de acord cu teoria c singurul membru care nu se va atrofia complet la
om va fi degetul cu care apas butoanele mainriilor. A rspuns cu un citat din Bertold Brecht: Scopul
tiinei nu este acela de a deschide ua nelepciunii infinite, ci acela de a fixa o limit pentru eroarea
infinit. Cu un asemenea feedback, orice polemic cu el ar deveni deplasat. tim doar c e un adept
al ideii c prin adresarea necontenit de ntrebri impertinente se menine tot timpul pe un drum care
sper s-l conduc, ntr-o zi, la Rspunsul Pertinent pe care l caut cu nfrigurare sfidnd constant i
fr temeri dureroasa axioma: tiina este topografia ignoranei...
ncercnd s dibuim cheia n care s-a cldit Succesul sau am crezut c acesta st ascuns n
spatele unor abiliti cu totul ieite din comun i care sunt menite numai celor care au puterea s
foreze n fiecare clip graniele curiozitii. Nu tiu daca rezultatul cutrilor noastre a fost un succes

-3-

EDITORIAL
sau doar o mare dezamgire: ne-am plecat capetele dup ce am nvat de la Eduard c abilitile
speciale se pierd iremediabil n eter atunci cnd nu sunt dublate de atitudini inteligente. Ct despre
sofisticatele mecanisme pe care le exploreaz intens mintea lui nu am aflat dect ce reuise el nsui
s retin de la Alexander Chase: ,,Mintea este i acel lucru cu care omul a fost dotat ca s uite, nu
doar s construiasc lucruri minunate...
Dar, ca s nu uite i ca s poat crea toate lucrurile acelea minunate pe care le-a subsumat
unui palmares greu de atins dar i mai greu de depit n condiii normale Eduard a tiut s i
menin vie curiozitatea, s o lase s-l ,,macine, contient c dac plictiseala se vindec prin
curiozitate, pentru cea de pe urma nu s-a aflat nc vreun leac i deocamdat este bine c aceasta
descoperire nu a fost fcut pentru c, aa cum ne-a spus-o chiar el, dac nu ar mai exista curiozitate
fr vindec, oamenii mai precis, categoria din care el face parte nu ar mai avea prea multe anse
s devin experii care tiu tot mai mult despre tot mai puine lucruri
Cu astfel de gnduri a absolvit liceul ,,Gheorghe ieica prietenul nostru Edi Bzvan. Am
neles la desprire c lecia pe care am primit-o de la el este una fr seamn, nepreuit i pe care,
cu un aport din parte-ne, trebuie s o lsm ca pilda celor care vor pi pe coridoarele liceului nostru
atunci cnd vom pleca la rndul nostru, vom alege sa pim pe drumuri noi, n cutarea adevrurilor ce
nu se las uor desferecate. Vom merge i noi mai departe, atunci cnd va sosi clipa despririi de
lcaul de cultura care ne deschide nc i ne lrgete n fiecare zi drumul spre Adevr.
Ii mulumim, Eduard Bzvan, pentru c ne-ai ajutat s nelegem c noi - cei care azi suntem
nc elevi ai liceului n care ai adstat timp de patru ani trebuie s fim un ideal al vremurilor ce vin i
nu doar un modest rezultat al trecutului
,,Ce minune c eti, ce ntmplare c sunt! (Nichita Stnescu)
N.B. Pe parcursul celor patru ani pn n vara lui 2006 ct a fost elev al Liceului Gh.
ieica, Eduard Bzvan a obinut urmtoarele distincii :
Clasa a IX - a

Premiul II la Olimpiada de Matematic , faza Judeean


Meniune la Olimpiada de Matematic , faza Naional
Mentiune la Concursul Interjudeean de Matematic Ion Ciolac , Craiova
Mentiune la Concursul Interjudeean de Matematica Recreaii Matematice , Iai
Premiul I la Concursul revistei de Matematic Alpha
Premiul I la Concursul Interjudeean de Matematic Gheorghe ieica
Premiul III la concursul de matematic Cangurul, faza Naional
Premiul III la concursul de matematic MCM , Poiana Pinului
Premiul III la Concursul Interjudeean de Informatic Info-Oltenia

Clasa a X-a

Premiul II la Olimpiada de Matematic , faza Judeean


Medalie de Argint la Olimpiada de Matematic , faza Naional
Medalie de Bronz la Olimpiada Internaional Pluridisciplinar Tuymaada 2004
Premiul I la Olimpiada de Informatic , faza Judeean
Premiul III la Concursul Interjudeean de Matematic Gheorghe Dumitrescu
Premiu Special la Concursul Naional de Fizic-Matematic Vrnceanu Procopiu
Meniune la Concursul Interjudeean de Matematic Nicolae Pun
Premiul II la Concursul de Matematic Cangurul, faza Naional

Clasa a XI-a

Premiul II la Olimpiada de Matematic, faza Judeean


Medalie de Argint la Olimpiada de Matematic, faza Naional
Premiul III la Olimpiada de Informatic, faza Judeean
Premiul II la Concursul Interjudeean de Matematic Gheorghe Dumitrescu
Premiul II la Concursul Interjudeean de Matematic Petre Sergescu

Clasa a XII - a

Premiul I la Olimpiada de Matematic, faza Judeean


Medalie de Aur la Olimpiada de Matematic, faza Naional
Medalie de Aur la Olimpiada Balcanica de Matematic, Cipru 2006
Medalie de Aur la Olimpiada Internaional de Matematic, Slovenia 2006
Premiul I la Concursul de Matematic Cangurul, faza Naional
Premiul II la Concursul Interjudeean de Matematic Gheorghe Dumitrescu

-4-

EDITORIAL

Andrei Ungureanu

SSM
H

La finalul carierei olimpice de elev n Romnia


Ne-am despartit anul acesta, noi, Colegiul National Traian, de unul dintre cei mai
straluciti elevi ai scolii traianiste, de Andrei Bogdan Ungureanu, absolvent promotia 2006 si
actualmente student al Universitatii Princeton-Statele Unite ale Americii.
Medalia de Aur i Premiul I la Olimpiada Internaional de Matematic
desfurat n perioada 10-20 iulie 2005 -Marena - Mexic obinut de olimpicul internaional
Andrei Bogdan Ungureanu l situeaz pe tnrul matematician pe primele locuri pe
mapamond i confirm nc o dat valoarea incontestabil a acestuia pe plan internaional.
Performana unic n judeul Mehedini pn la ora actual (medaliat cu aur la olimpiada
internaional), rezultatul lui Andrei Bogdan Ungureanu se contureaz dup o carier
olimpic de succes nceput cu muli ani n urm.
Premiul I la toate olimpiadele judeene i naionale (ncepnd cu cls. a V-a i pn n
prezent actualmente este elev n clasa a XI-a) ar fi suficiente pentru a-l situa pe Andrei pe
treapta cea mai de sus a olimpiadelor naionale.
Dar rezultatele nu se opresc aici pentru c n spatele aparenelor avem un talent nativ,
o inteligen remarcabil susinut cu succes de un caracter de nvingtor, de o performan
uimitoare, de putere i druire. i pentru c gloria nu se ctig fr sacrificii, avem multe ore
de munc, mult ambiie i hotrre n dorina de birui.
Evoluia lui Andrei pe scara internaional a nceput gradat, treapt cu treapt, baraj
cu baraj (pentru selectarea lotului olimpic naional se susin n fiecare an alte 6 baraje),
medalie dup medalie la fiecare participare internaional.
n anul 2002, elev pe atunci n clasa a VIII-a, Andrei Bogdan Ungureanu obine
Medalia de Aur la Olimpiada Balcanic de Matematic pentru juniori, Trgu MureRomnia.
Urmeaz apoi trecerea la liceu, alte nceputuri, alte standarde, pe undeva o alt
matematic: arie extins ca volum i subtilitate, probleme adnci, acuitate maxim,
ingeniozitate n creaie i construcie.
n ciuda terenului nebttorit, reuim s doborm obstacolele, s ne ridicm deasupra
greutilor i Andrei se calific n lotul pentru seniori al Romniei (concureaz acum cu elevii
claselor IX-XII pe acelai tip de subiecte) obinnd Medalia de Bronz la Olimpiada de
Matematic Pluridisciplinar din Iacuia-Rusia.
Trecerea la seniori i selecia n lotul Romniei n fiecare an au fost probe foarte dure
att din punct de vedere tiinific ct i psihologic: rezisten la efort, emoii, presiuni,
concentrare.
n clasa a X-a, anul 2004, a obinut Medalia de Argint la Olimpiada Balcanic de
Matematic Plevna - Bulgaria i Medalia de Argint la Olimpiada Internaional de
Matematic Atena - Grecia succes rsuntor pentru prima participare la olimpiada
internaional i pentru vrsta fraged a lui Andrei.
n clasa a XI-a, anul 2005, situaia se complic ntruct curriculum-ul pentru
olimpiada naional este altul dect cel pentru internaionale i Andrei reuete n paralel s le
parcurg pe amndou la cote maxime.
Premiul I la Olimpiada Naional de Matematic, calificarea n lotul Romniei urmate
apoi de rezultatele internaionale: Medalia de Bronz la Olimpiada Balcanic de Matematic
Iai- Romnia i n sfrit mplinirea visului din copilrie, succes i lacrimi de bucurie
Medalia de Aur la Olimpiada Internaional de Matematic escaladarea olimpic

-5-

EDITORIAL
ajuns la apogeu, Andrei a reuit s transforme visul n realitate.Si cum istoria se
repeta,acelasi succes se reediteaza si in 2006- Medalia de Aur la Olimpiada Internaional
de Matematic-Slovenia.
Andrei a obinut primul loc i la concursurile americane American Mathematics
Competitions i American Invitational Mathematics Examination (echivalentele olimpiadelor
locale i judeene din Romnia), fiind singurul calificat pentru faza final U.S.A.M.O.
(United States American Mathematics Olympiad). Punctajul foarte bun obinut l-a remarcat
n S.U.A. drept pentru care a primit o burs pentru olimpici cu durata de un an din partea unei
instituii americane de selecie a performerilor: Exeter Academy New Hampshire SUA.
Andrei nu a onorat ns aceast burs ntruct a considerat c pregtirea n Romnia l va
propulsa foarte bine n ierarhia valorilor mondiale.
Elev eminent la toate disciplinele, Andrei a avut din clasa a V-a pn n prezent,
media general 10, a participat i la olimpiadele judeene de fizic i informatic n clasa a
VIII-a i a IX-a, obinnd numeroase premii I.
Colegiul Naional Traian a fost dintotdeauna o coal a elevilor, a performanelor, a
olimpicilor iar succesele obinute de Andrei Ungureanu ne determin s-l declarm ca fiind
cel mai bun elev al acestuia de la nceputuri i pn n prezent i totodat cel mai valoros
olimpic al tuturor timpurilor pentru coala matematic mehedinean.
Elev fiind, am fost i eu olimpic la rndul meu (premiul I, II, III la olimpiadele
judeene, naionale i interjudeene), dar nu am beneficiat de o atenie aparte n pregtirea
mea. Cnd mi-am ales s fiu profesor, am tiut c voi sprijini cu orice pre i sacrificii elevii
cu nclinaii deosebite cci numai aa succesele pot fi maxime. Am creat mai muli olimpici
naionali dar din prima clip cnd l-am ntlnit pe Andrei (n clasa a VIII-a) am tiut c
aceasta este o ans unic n via.
mi amintesc de prima impresie a primei ntlniri: rapiditatea, ingeniozitatea i
naturaleea cu care rezolva problemele (n demersul a dou ore ajunsesem deja la olimpiadele
naionale de clasa a X-a n mod firesc) mi-au umplut sufletul de bucurie i efortul era cu att
mai mare cu ct nu puteam s-i art acest lucru pentru c eram abia la nceput de drum.
A urmat apoi o munc de apostolat fr ezitri i renunri, leciile noastre durau 4-5
ore n fiecare duminic, le pregteam cu eforturi (dificultate, concentrare, nvare, timp)
fiind destul de greu s ne ridicam la standardele impuse de ambitiile noastre.
S-a depus mult suflet n acest vis, satisfaciile sunt pe msur i desi Andrei studiaza
acum in polul universitar al inteligentelor mondiale, satisfactiile noastre, ale celor care l-am
creat si format, parinti si profesori, nu se opresc aici.Vom mai auzi de Andrei Ungureanu,ne
vom mandri cu el peste ani si, cand inceputurile lui vor fi fost date deja uitarii, vom putea
spune cu modestie: l-am cunoscut candva.
Si nu pot sa nu imi amintesc spusele istoricului, scriind despre menirea omului potrivit
la locul potrivit si despre opera unei vointe:
poate s-ar fi dezvoltat el si fara aceasta vointa, insa cine stie cum si cine stie cand .
Un exemplu, un mentor, o vointa, nu putem exprima in cuvinte ceea ce simtim cu totii
atunci cand ne gandim la Andrei si la performantele lui.
Manuela Prajea

-6-

Petre Sergescu
A doua ediie a Concursului interjudeean
Petre Sergescu
-Drobeta Tr. Severin20 ianuarie 2006

SSM
H

Asa cum anuntasem in numarul 5 al revistei noastre, pe 20.01.2006 a avut


loc la Colegiul National Traian a doua editie a Concursului Interjudetean de
matematica Petre Sergescu.La concurs au participat peste 600 de elevi din
judetele Mehedinti, Dolj, Gorj, Caras-Severin, Timis.La Simpozionul National
dedicat acestui eveniment au participat un numar mare de cadre didactice,
sustinandu-se un numar de 38 de Referate,dupa cum reiese din programul de
mai jos.

Programul Simpozionului National


PETRE SERGESCU 20.01.2006

1. Dezvoltarea creativitatii in cadrul orelor de matematica

Inspector General Doinita Mariana Chircu, Lector universitar doctorand asociat

2. Combinatorica, o perspectiva moderna in sfera concursurilor de matematica

Prof.doctorand Prajea Manuela CNT

3. Consideratii metodice asupra unor probleme date la olimpiadele din SUA

Profesor dr.Cainiceanu Gheorghe CNT

4.

Bertrand Russel matematician si filosof

Prof.Stoica Rodica, Director adjunct al Colegiului National Traian

5. Viata si opera lui gheorghe titeica

Prof.Falon Florica Liceul de Marina Orsova

6. Generator de probleme

Prof. Bejenaru Laviniu CNT

7. Caracteristica unui inel

Prof. Dan Nedeianu Lic. D-l Tudor Drobeta Tr.Severin

8. Asupra unor identitati trigonometrice

Prof.Lucian Dragomir Liceul Otelul Rosu

9. Problematicul in probleme

Prof. Gh. Calafeteanu Sc.Gen. 9 DrobetaTurnu Severin

10. Opera matematica a lui Grigore Moisil

Prof.Florin Ghiocel CNT

11. Proprietati geometrice ala conicelor

Prof.Gimoiu Iuliana CNT.

12. Spectrul unei matrici

prof.Paponiu Dana CNT

13. Metode si tehnici de rezolvare a problemelor de aritmetica

Institutor Teis Alina, Lic.Ped.Stefan Odobleja.

14. Conditii pentru echivalenta unor clase de functii elementare


15. Problema lui Titeica

Prof.Adi Lupu, Grup Scolar Decebal


Prof.Adriana Patrutescu Moclea, Sc.Gen.11

16. Viata si opera lui David Emmanuel

Prof.Doru Presneanu, Liceul Pedagogic Stefan Odobleja

17. Algebra in ajutorul algebrei

Prof. Farago Alexandru Liceul TL Orsova, Farago Gabriela si Raescu Gheorghita ,Sc.Gen.Orsova

18. Teorema lui rolle si aplicatii

Prof.Dan Nanuti ISJ, Prof.Bizdoaca

19. tefan Odobleja si psihologia consonantista

Claudia Colegiul Economic

Prof.Emanuela Busoi Colegiul National Traian

20. Profesori de prestigiu ai invatamantului mehedintean

-7-

Petre Sergescu
Prof.Stefan Marica, pensionar

21. Metode generale de rezolvare a problemelor de aritmetica

prof. Hinoveanu Sorin, Sc.Gen.6

22. Demonstratii ale teoremei lui ieica

Prof. Tache Oana ,Sc.Gen Craguesti

23. Transformari geometrice si aplicatii

Prof.Drula Ileana,Strehaia

24. Academician Gheorghe Calugareanu 30 de ani de la moarte

Prof. Roman Florentina,Lic.Matei Basarab, Strehaia

25. Probleme de geometrie...inselatoare

Prof. Leonard Giugiuc, CNT

26. Matrici speciale

Prof. Giugiuc Constantin, Lic.Domnul Tudor

27. Definirea functiilor elementare cu ajutorul sirurilor


28. Aplicatii ale seriilor in studiul functiilor elementare

Prof.Raducan Emilia, Grupul Scolar Industrial Decebal


Prof. Pupaza Ecaterina, Grupul Scolar Industrial Decebal

29. Probabilitati geometrice


30. Algoritmii - puntea dintre matematica si informatica

Prof.Popescu Tudor, Colegiul F.Nitti, Timisoara


Prof.Cristina Draghescu, Grupul Scolar Industrial Decebal

31. Marele matematician Gheorghe ieica

Prof. Draga Tatucu Mariana ,Liceul Gheorghe Titeica

32. Problema celor 12 bile

Prof.Stoican Victor, Grup Sc.Ind.Baia de Arama

33. Idei antice, lectii contemporane

Prof.Baloi Valeria, St.Odobleja.

34. Viata si opera marelui matematician Gheorghe ieica

Profesori Adriana Nanuti Liceul Gh.Titeica, Prof.Dana Mosor ISJ

35. Probleme de aritmetica, tehnici si metode de rezolvarte


36. iruri definite prin relatii de recurenta de ordinul II
37. Asupra primitivelor unor functii compuse

Prof.Victor Saceanu Sc.Generala Nr.11


Prof. Buzatu Carmen Liceul Domnul Tudor.
Prof. Bacila Alin Colegiul George Cosbuc Motru

38. Complemente de geometrie sintetica


39. Testul grila, avantaje si dezavantaje

Prof.Antonie Rodica CNT


Prof.Popescu Eleodor CNT

Chairman
Lector univ. dr. Vasile Tomi
Centrul Universitar Drobeta Turnu Severin
In cadrul concursului adresat elevilor, asa cum se stie deja ca o noutate au fost invitati si
elevii clasei a IV-a. Numarul mare de participanti la aceasta clasa (peste 100) ne arata ca
initiativa introducerii in concurs a acestei grupe de varsta a fost buna.
Concursul a fost organizat ca si la prima editie de catedra de matematica din Colegiul
National Traian, in colaborare cu Filiala Mehedinti a Societatii de Stiinte Matematice.
Presdintele comisiei de organizare a fost Prof.drd.Prajea Manuela, director al Colegiului
National Traian, vicepresedinte a fost Prof.dr.Cainiceanu Gheorghe, presedintele Filialei
Mehedinti a SSMR, secretar fiind Prof. Dan Daniel de la Liceul Gh.Titeica. Subiectele au fost
propuse de colegi din 14 scoli ale judetului, comisiile pe clase fiind conduse de profesorii: Pi
Rada Marica, Sc.gen.5 la clasa a IV-a, Antonie Rodica CNT, la clasa a V-a, Gimoiu Iuliana,
CNT, la clasa a VI-a, Stretcu Daniel, Gh.Titeica, la clasa a VII-a, Popescu Eleodor CNT, la
clasa a VIII-a, Paponiu Dana, CNT, clasa a IX-a, Nedeianu Dan, D-l Tudor, la clasa a X-a,
Sitaru Daniel, Colegiul Economic, la clasa a XI-a, Giugiuc Leonard, CNT, la clasa a XII-a.

-8-

Petre Sergescu
Concursul Interjudeean de Matematic
PETRE SERGESCU
Ediia a II-a, 20 ianuarie 2006, Drobeta Turnu-Severin

SSM
H

SUBIECTE
Clasa a IV - a
1) Un elev citete o carte care are 385 pagini.
a) Ce pagini citete elevul , dac suma numerelor celor dou pagini este 413?
b) Cte cifre s-au folosit pentru numerotarea paginilor acestei cri?

***

2) Un biat are 11 ani.


a) Determinai vrsta tatlui acestui biat tiind c atunci cnd biatul va avea vrsta tatlui,
tatl va avea 59 ani.
b) Peste ci ani vrsta tatlui va fi egal cu dublul vrstei fiului?
Prof. Marica Pi-Rada

3) Un balaur s-a nscut cu 11 capete. n primii 13 ani de via i-a crescut n fiecare an cte un cap.
n urmtorii 15 ani , i-au mai crescut n fiecare an cte 2 capete. Apoi balaurul s-a mbolnvit i
a nceput s piard n fiecare an cte 3 capete.
a) Care a fost numrul maxim de capete i la ce vrst le-a avut?
b) Ci ani a trit balaurul?
c) La ce vrst a avut balaurul 30 capete?

Prof.Ionel-Vasile Pi-Rada

Clasa a V-a
1) a) Determinati numerele naturale x,y,z,t,u, x<y<z<t<u astfel incat 2x + 2y +2z + 2t + 2u = 248.
b) Determinati ultima cifra a numarului 22002 + 2 2003 + 22004 +22005 + 22006.
2006

Prof.Antonie Rodica

2) Sa se scrie numarul 5
ca o suma de cinci numere naturale a,b,c,d,e astfel incat resturile
impartirii fiecaruia dintre numerele a,b,c,d,e la 5 sa fie distincte.

Prof.Nedeianu Dan

3) Se considera multimea M={nN* | n 2006}.


a) Stabiliti daca se poate scrie multimea M ca reuniune de submultimi disjuncte Mk astfel incat
in fiecare multime Mk sa existe un element egal cu suma celorlalte.
b) Sa se demonstreze ca din oricare 1005 elemente din M se pot alege doua elemente diferite
astfel incat suma lor sa fie 2006.

Prof. Paponiu Dana

Clasa a VI-a
1) Se considera multimea M={b a(1+a+a2)| a, b N}
a)Sa se arate ca M contine o infinitate de patrate perfecte;
b)Pentru b numar prim, determinati elementele lui M ce pot fi scrise ca suma a cinci numere
naturale consecutive.
Prof.Dana Paponiu

2) Daca n
notam cu Pn produsul divizorilor naturali ai numarului n. Sa se determine n cu
proprietatea ca Pn=10450.
N* ,

Prof.Manuela Prajea

3) Fie [OX, [OY doua semidrepte situate in interiorul unghiului propriu <AOB, [OX [OY,
m(<AOX)=a; m(<BOX)=b,m(<AOY)=c si m(<BOY)=d. Daca ac=bd, atunci sa se arate ca:
a) ab+cd=2bd
b) unghiurile <AOB si <XOY au aceeasi bisectoare.
Prof.Iuliana Gimoiu

-9-

Petre Sergescu
Clasa a VII-a
1) a) Sa se arate ca : ab5

= m25 , unde a,b N si m= ab ( ab +1).

b) Sa se calculeze suma cifrelor numarului N , unde N = 3 + 33 + 333 + .... + 33...3 .


2006 ori

Prof.Gorun Sanda, Prof.Prajea Manuela

2) Sa se determine numarul de solutii numere intregi pozitive ale ecuatiei:


1 1
1
+ =
.
x y 2006
Prof.Stretcu Daniel

3) In trapezul ABCD cu baza mare AB, diagonala CA este bisectoarea unghiului


< BCD.Daca M este mijlocul bazei mici CD, iar BMAD = {N}, sa se afle masura unghiului
<ACN.

Prof.Dan Nedeianu

Clasa a VIII-a
1) Determinati numerele intregi a, b, c care verifica simultan inegalitatile:
a2 +b +3 <5a, b 2 +c +3 <5b si c2 +a +3 <5c.
Prof.Dan Nedeianu

2) Fie ABCDABCD un cub cu muchia de 8 cm si E[AD], F[CD] astfel


incat DE = DF = 2cm. Paralela prin E la DB intersecteaza (ABB) in H, paralela prin F la
DB intersecteaza (BCC) in I, iar planul determinat de dreptele EH si FI intersecteaza [BB] in
O. Sa se determine Aria[EHOIF].

Prof.Gheorghe Cainiceanu

3) O societate comerciala a cumparat trei feluri de aparate cu preturile de 23 lei, 45 lei si 67 lei ,
platind pentru toate suma de 854 lei.Care este numarul total al aparatelor cumparate?

Prof.Eleodor Popescu

Clasa a IX-a
x y 1 1 5
1) Sa se rezolve in Z* ecuatia: + + + =
y x x y 2

Profesori Dana Paponiu, Stefan Marica

2) Fie poligonul A1A2An inscris in cercul C de centru O (n,n3). Daca punctele P1,P2,,Pn
situate pe segmentele (A1A2), (A2A3),,(AnA1) impart segmentele orientate in acelasi raport si
OP1=OP2==OPn , sa se arate ca A1A2An este poligon regulat.
Prof. Manuela Prajea

3) Demonstrati ca are loc relatia:


1
2
3
+
+
+ ... +
x +1 x + 3 x + 6

n
p
n(n + 1)
x+
2

n
, n N * si x (0, ).
x
Prof. Dana Paponiu

Clasa a X-a
1) Fie a, b (0,1) (1,) si funcia f: R(0,) cu proprietile:
(i) x1 , x 2 (0,) cu x1 x 2 implic f( x1 ) f( x 2 ).
(ii) funcia g:(0,)R, g( x )=f( log a x ) f( log b x ) este constant.
S se calculeze produsul ab i demonstrai c exist funcii f care satisfac condiiile din enun.
2) Fie z =

[ 3(1 + i ) + 1 i ]

2006

[ 3(1 i ) + 1 + i ]

2006

Prof.Dan Nedeianu

numr complex.

S se arate c z este un numr real din intervalul ( 2 3010 ,2 3009 ).


Prof. Eleodor

- 10 -

Popescu

Petre Sergescu

SSM
H

2006 + x + x + 8 + x 3 = 50 , x R.
2
3
b) S se arate c: x + 4
4 , x , .
2x 3
2

3) a) S se rezolve ecuaia:

Prof. Florentina Roman

Clasa a XI-a
1) Fie A = (aij )1i , j n ; B = (bij )1i , j n ; C = (cij )1i , j n ; ai , j = max(i, j ) ; bi , j = min(i, j )
ci , j

i j; i j

=
. S se arate c det[( AB C )( BC A)(CA B )] 0 .
2i 1, i = j

Prof. Daniel Sitaru

2) Fie ( x n )n0 , x0=a, x1=b, x2=c, x3=d, a, b, c,d R i x n = x n1 x n 2 + xn 3 x n 4 , ( )n 5 .

a) Artai c pentru orice valori ale constantelor a, b, c, d irul ( x n ) are subiruri convergente.
b) S se determine inf xn i sup xn .
c) S se determine a, b c, d astfel irul ( xn ) s fie convergent.
Prof. Gheorghe Ciniceanu

3) S se determine funciile continue f: RR care satisfac relaia:


f (x )
f x 2006 =
; ( )x R { 1} .
2
1 + x + x + ........... + x 2005

Prof.. Manuela Prajea

Clasa a XII-a
1) a) Fie m R* si f,g:I R, f ( x) =

cos x
sin x
, g ( x) =
,
sin 2 x + m(sin x + cos x)
sin 2 x + m(sin x + cos x)

unde I R este un interval pe care f si g sunt definite. Calculati

f ( x)dx g ( x)dx.

x 1
1 cos x, 2 2
x

b) Fie F:RR,F(x) = 4 +
. Aratati ca F derivabila si calculati F(x).
2

x
1


3 + cos x, >

2 2
Prof.Giugiuc Constantin

2) Fie p N, p 2 si G = {A1 , A2 ..., A p }, M 2 ( R) un grup cu operatia de inmultire a matricelor.


p

Se stie ca

Tr ( A ) = 0. (Tr(Ak) = suma elementelor pe diagonala principala a matricei Ak).


k =1

cos kt sin kt

a) Sa se arate ca multimea G =
t = , k = 1.8 satisface cerintele din enunt.
4

sin kt cos kt
p

b) Sa se demonstreze ca

A
k =1

= O2 (matricea nula de ordinul 2).


Prof. Gheorghe Cainiceanu

3) Fie ( G,o ) un grup abelian cu 4n+2 elemente, n N*. Presupunem cunoscut faptul ca ecuatia
x2n+1=e are 2n+1 solutii.
a) Aratati ca ecuatia x2=e are 2 solutii.
b) Daca grupul ( G,o ) este neabelian ramane adevarata cerinta a) ?

Prof. Leonard Giugiuc

- 11 -

Petre Sergescu
Concursul Interjudeean de Matematic PETRE SERGESCU
ediia a II-a, 20 ianuarie 2006, Drobeta Turnu-Severin

REZULTATE
Clasa a IV-a
Numele i prenumele
Ciobanu Diana
Medele Miruna
Osmani Amil
Mitroi Laura
Lupulei Alexandru
ru Vlad
Clbureanu Diana
Guin Diana Alexandra
Iona Ioana
Mema erban
Neagu Drago
Teodorescu Alin
Caot Andreea
Mija Ctlina
Petcu Oana
Trancot Diana
Piorescu Alexandru
Chicet Carina
Puna Denisa Nicoleta
Puican Tiberiu
Crstov Gabriela
Marculescu Teodora
Simion Claudia
Munteanu Floarea
Borco tefan
Mogoanu Cosmin
Benga Andrei
Ciuciu Laureniu
Bobii Ionu Adelin
Michidu Alexandru
Voicu Robert
Geic Gabriela
Oancea Mihaela
Olaru Alin

Scoala
Odobleja
Carol Craiova
Carol Craiova
Carol Craiova
Odobleja
Carol Craiova
Carol Craiova
Odobleja
c Paulian
Odobleja
Odobleja
Carol Craiova
Carol Craiova
Carol Craiova
Carol Craiova
c 6
Carol Craiova
c 6
Odobleja
c 2
Carol Craiova
c 6
Odobleja
Odobleja
c 3
c Paulian
c 2
c 2
Odobleja
Carol Craiova
Carol Craiova
Carol Craiova
Carol Craiova
c 6

Premiul
I
II
II
II
III
III
meniune
meniune
meniune
meniune
meniune
meniune
meniune
meniune
meniune
meniune
meniune
meniune
meniune
meniune
meniune
meniune
meniune
meniune
meniune
meniune
meniune
meniune
meniune
meniune
meniune
meniune
meniune
meniune

Numele i prenumele
Soare Daniela
Ciut Cora
Lu Robert
Bloi Mihnea
Dragomir Andrei
Du Anidora
Mrzoca Cezara
Rescu Adelina Larisa
Spnu Antoanela
Bloi Elisabeta
Gheorghe Bogdan
Gherghe Diana
Mircea Andrei
Popescu Alin
Beloineanu Andrei
Ciocan Andrei
Gegea Georgiana
Popa Mihaela
Tudor Georgiana
Borug Alin
Marcu Bogdan
Mendea Andreea
Ene Bogdan
Stanciu Vladimir
Marghescu Luminia
Truc Alina
Ciotrl Cristian
Liliac Ramona Denisa
Semen Alexandra
Vladu Ctlin
Brbulescu Ramona
Budecan Cosmin
Munteanu Enia Maria

Scoala
c 14
Odobleja
Carol Craiova
c Paulian
Odobleja
c 3
c Paulian
c 11
P Sergescu
Sc 2
c 2
c 3
Sc 3
c 11
Carol Craiova
c 3
Odobleja
Odobleja
c Paulian
c 2
Carol Craiova
c 3
c 6
c 6
c 11
c 6
c Negreanu
Odobleja
c 2
c Negreanu
Odobleja
c 11
Odobleja

Premiul
meniune
meniune
meniune
meniune
meniune
meniune
meniune
meniune
meniune
meniune
meniune
meniune
meniune
meniune
meniune
meniune
meniune
meniune
meniune
meniune
meniune
meniune
meniune
meniune
meniune
meniune
meniune
meniune
meniune
meniune
meniune
meniune
meniune

Scoala
CNT
CNT
CNT
CNT
CNT
CNT
CNT
Odobleja
CNT
CNT
CNT
CNT
Motru
CNT
CNT
CNT
Odobleja

Premiul
I
II
III
meniune
meniune
meniune
meniune
meniune
meniune
meniune
meniune
meniune
meniune
meniune
meniune
meniune
meniune

Numele i prenumele
Raicu Maria
Badea Beatrice
Pegulescu Vlad Daniel
Nicola Dorina
Lacatusu Maria
Tnasie Denisa
Anghel Cristian
Cazacu Cosmin
Mohora Eduard
Oprea Alida
Nu Flavius Andrei
Diaconu Paula Luiza
Surdulescu Simona
Calafeteanu Liviu
Catanescu Raluca
Grbovan Paul
Idita Marina

Scoala
c 11
CNT
Vnju Mare
Dl Tudor
Sc.6
CNT
Vnju Mare
CNT
Odobleja
CNT
CNT
Odobleja
c 9
c 9
Sc.2
CNT
c 9

Premiul
meniune
meniune
meniune
meniune
meniune
meniune
meniune
meniune
meniune
meniune
meniune
meniune
meniune
meniune
meniune
meniune
meniune

Clasa a V-a
Numele i prenumele
tefan Andrei
Voicu Rzvan
Sceanu Andrei
Nicolae Andrei
Nicola Silviu
Chiri Alexandra
Vlcu Andrei
Brbulescu Cristian
Blu Adrian
Faer Eleonora
Gimoiu Ruxandra
Marghescu Andreea
Aldea Loredana
Viian Sorina
Filip Radu
Georgescu Ana
Nistor Ioana Andreea

- 12 -

Petre Sergescu
Blaa Ciprian
Raicu Marian
Turcu Izabela
Ania Georgiana
Asproniu Andreea
Crstea Roxana
Pristoleanu Narcis

SSM

CNT
Motru
Lic ieica
CNT
CNT
CNT
CNT

meniune
meniune
meniune
meniune
meniune
meniune
meniune

Lcraru Alexandru
Stng Adrian
Trlui Valeria
Oancea Madalina
Tintos Adela
Truc Antonia Lidia
Zoril Mihai

Odobleja
c 11
Lic ieica
Motru
Sc.6
Odobleja
CNT

meniuneH
meniune
meniune
meniune
meniune
meniune
meniune

Scoala
CNT
CNT
CNT
c 14
CNT
Motru
CNT
Motru
CNT
CNT
CNT
Sc.2
c 14
Lic ieica
Sc.2
c 11

Premiul
I
II
III
III
meniune
meniune
meniune
meniune
meniune
meniune
meniune
meniune
meniune
meniune
meniune
meniune

Numele i prenumele
Guran Maria
Lupiu Gabriela
Pnescu Drago
into Mdlina
Arbnai Cornel
Boescu Anca
Costea Alexandra
Drguin Cornel
Olaru Mihaela
Prvulescu Eleodor
Popescu Denisa
Rdulescu Silvia
Sirbu Adrian Mihai
Tufaru Danusia
Vian Luana
Cojocaru Alex

Scoala
Sc.2
CNT
CNT
CNT
Lic ieica
CNT
CNT
c 5
CNT
CNT
c 14
c 6
Lic.Titeica
c 6
CNT
Sc 2

Premiul
meniune
meniune
meniune
meniune
meniune
meniune
meniune
meniune
meniune
meniune
meniune
meniune
meniune
meniune
meniune
meniune

Scoala
Carol Craiova
Lic ieica
c 5
Carol Craiova
Motru
CNT
CNT
Carol Craiova
CNT
CNT
CNT
CNT
CNT

Premiul
I
II
III
meniune
meniune
meniune
meniune
meniune
meniune
meniune
meniune
meniune
meniune

Numele i prenumele
Nicolescu Adelina
Ti Andreea
Papa Andreea
Agape Mihai
Cheredi Ion
Popescu Eduard
Surugiu Mdlina
Andreca Mihai
Leonescu Larisa
Popa Bogdan
Prunar Victor
Zanfir Cristian

Scoala
Carol Craiova
CNT
CNT
CNT
Lic ieica
CNT
Motru
CNT
c 14
c 14
Loga Carans.
Doda Carans.

Premiul
meniune
meniune
meniune
meniune
meniune
meniune
meniune
meniune
meniune
meniune
meniune
meniune

Scoala
Loga Carans.
CNT
CNT
Loga Carans.
CNT
Loga Carans.
Loga Carans.
Loga Carans.

Premiul
I
II
III
meniune
meniune
meniune
meniune
meniune

Numele i prenumele
eitan Mihaela
Croitoru Rzvan
Milcu Roxana
Orbu Alexandra
Tudor Mihaela
Alex. Crivac Cristina
Duicu Vlad
Ionacu Luana

Scoala
CNT
CNT
Loga Carans.
Lic ieica
Motru
Lic ieica
Motru
Motru

Premiul
meniune
meniune
meniune
meniune
meniune
meniune
meniune
meniune

Scoala
CNT
Carol Craiova
Loga Carans.
CNT
CNT
Motru
CNT

Premiul
I
II
III
meniune
meniune
meniune
meniune

Numele i prenumele
Cruan Bogdan
Drgan Alin
Eftinoiu Diana
Staicu Alin
Gliga Anemona
Catan Nicu
Chilom Radu

Scoala
CNT
Oelu Rou
CNT
Motru
CNT
Motru
CNT

Premiul
meniune
meniune
meniune
meniune
meniune
meniune
meniune

Clasa a VI-a
Numele i prenumele
Zugravu Rozalia
Grosu Vlad
Limbosu Maria
Nistor Mihaela Adriana
Cilioaica Rebeca
Dogaru Marian
Sndulescu Adela
Bleanu Andrei
Drghici Rzvan
Opria Geanina
Pavel Cristian
Toader Simona
Ghinea Andrei Bogdan
Grecu Anda Teodora
Croitoru Andra
Gavril Costinel

Clasa a VII-a
Numele i prenumele
Tiucu Andreea
Teil Bianca
Milici Alina
Matei Diana
Sibinescu Ioana
Furcu Ioana
Papa Florin
Sandu Loredana
Stnioar Sarah
Budu Anca
Drghia Miruna
uculanu Andreea
Zorocliu Anda

Clasa a VIII-a
Numele i prenumele
Timofte Andrei
Carapencea C-tin
Du Adrian
Vlad Adina
Pupz Elena
Megan Ligia
Moatr Alexandra
Plotinaru Diana

Clasa a IX-a
Numele i prenumele
Trocan Irina
Pufu Cristian
Kremer Emanuela
Boblc Oana
Oprea Radu
Sibinescu Corina
Ciouca Eugen

- 13 -

Petre Sergescu
Lincan Dan
tefnoiu Anca
Voicu Andreea
Gurgu Caius
Rogobete Roxana
Sandu Andreea
Mema Alexandra
Mercioniu Marina
Rducu Alexandru
Bechir Adriana
Cerga Alina
Ciorobea Mihai
Nica Flavius
Lungu Viorel Costin
Zaharia Mihaela

CNT
CNT
CNT
Loga Carans.
CNT
Lalescu Rei
CNT
Orova
CNT
CNT
CNT
CNT
CNT
CNT
CNT

meniune
meniune
meniune
meniune
meniune
meniune
meniune
meniune
meniune
meniune
meniune
meniune
meniune
meniune
meniune

Nicolescu Alexandra
Robu Alex
Rou Maria
Tuce Andreea
Dunreanu Lidia
Ungura Drago
Tic Mihaela
Picu Vulpin Bianca
Svoiu Aurel
Mrinic Andreea
Vasilache Mdlina
Ursachi Alexandru
Papal Doriana
Pasre Andra
Vldoiu Alina

CNT
CNT
CNT
Lic ieica
Lic ieica
Oelu Rou
Lalescu Rei
CNT
CNT
Carol Craiova
CNT
Vnju Mare
CNT
Lalescu Rei
CNT

meniune
meniune
meniune
meniune
meniune
meniune
meniune
meniune
meniune
meniune
meniune
meniune
meniune
meniune
meniune

Scoala
CNT
CNT
CNT
CNT
Lic ieica
CNT
CNT
CNT
Lalescu Rei
CNT
CNT
CNT
CNT
Doda Carans.
CNT
Doda Carans.

Premiul
I
I
II
II
III
III
meniune
meniune
meniune
meniune
meniune
meniune
meniune
meniune
meniune
meniune

Numele i prenumele
Popescu Anca
Puia Constantin
Nistor Alexandru
Jiplea Bogdan
Ciocea Marina
Belbu Loredana
Abagiu Aurel
Ghinea Constantin
Ghiga Octavian
Mariescu Radu
Punescu Georgiana
Hllu Rodica
Prvnescu Izabela
Preneanu Ruxandra
Vladu Margareta
Caragea Ctlin

Scoala
Odobleja
Carol Craiova
CNT
CNT
CNT
CNT
CNT
Dl Tudor
CNT
CNT
CNT
CNT
Lic ieica
CNT
CNT
Motru

Premiul
meniune
meniune
meniune
meniune
meniune
meniune
meniune
meniune
meniune
meniune
meniune
meniune
meniune
meniune
meniune
meniune

Scoala
CNT
CNT
Lic ieica
CNT
CNT
CNT
Lalescu Rei
Strehaia
Lalescu Rei
CNT
CNT
Motru
CNT
Vnju Mare

Premiul
I
II
II
II
II
meniune
meniune
meniune
meniune
meniune
meniune
meniune
meniune
meniune

Numele i prenumele
Surdulescu Vlad
Busuioc Lucian
Boar Andra
Frumosu Sveta
Moi Delia
Asproniu Gheorghe
Dan Lorena
Enache Bianca
Glavici Iasmina
Pascu Liviu
Posniciuc Iulian
Svoiu Lorena
Ungureanu Irina

Scoala
CNT
CNT
CNT
Orova
CNT
CNT
CNT
Doda Carans.
CNT
CNT
CNT
CNT
CNT

Premiul
meniune
meniune
meniune
meniune
meniune
meniune
meniune
meniune
meniune
meniune
meniune
meniune
meniune

Scoala
CNT
Lic ieica
CNT
CNT
CNT
CNT
Orova
Lalescu Rei
CNT

Premiul
I
II
III
meniune
meniune
meniune
meniune
meniune
meniune

Numele i prenumele
Crciunescu Mirela
Jianu Amalia
Trea Gabriela
Chilom George
Nef Andrei
Creescu Oana
Mare Flavia
Purdescu Monica
Vjaic Raluca

Scoala
CNT
CNT
CNT
CNT
CNT
CNT
CNT
CNT
CNT

Premiul
meniune
meniune
meniune
meniune
meniune
meniune
meniune
meniune
meniune

Clasa a X-a
Numele i prenumele
Mituc Atena
Tigora Andrei
Rveanu Ioana
uelea Robert
Gogoloiu Gabriela
Botea Silvia
Tu Leontin
Coand Oana
Popovici Doru
Creu Andrei
Hinoveanu Ctlin
Prundeanu Andreea
Ciniceanu Andrei
Dochin Luminia
Rachieru Adrian
Iacob Alexandra

Clasa a XI-a
Numele i prenumele
Rapcea Mihai
Bzdoac Mihai
Bobii Ruxandra
Dgdi Monica
Rou tefan
Marcu George
Cucu Silviu
Pleu Maria
Buna Mdlina
Nstase Liliana
Turturea Roxana
Aanici Simona
Petria Andrada
Ghintaru Silviu

Clasa a XII-a
Numele i prenumele
Ungureanu Andrei
Bzvan Eduard
Mirea Teodor
Stoica George
Bcrin Dan
Paov Iulia
Buoniu Andra
Chi Andrei
Ciocea Diana

- 14 -

Petre Sergescu
Simpozionul Naional
PETRE SERGESCU

SSM
H

REFERATE

I. O generalizare a problemei celor 12 bile


prof. Stoican Victor
Grup colar C.Brncoveanu
Baia de Arama
Este cunoscut problema celor 12 bile:
Se dau 12 bile, dintre care 11 au aceiai greutate iar a 12-a este de greutate diferit
(nu se tie dac este mai grea sau mai uoar fa de celelalte). Din trei msurri cu balana s
se afle care este bila defect si cum este fa de celelalte.
Problema general pe care ne-o propunem, este urmtoarea:
Avnd o balan i n msurri, care este numrul maxim de bile m=f(n) dintre care m-1
au aceiai greutate iar a m-a bil diferit de celelalte, astfel ca prin n msurri s putem afla
care este bila defect.
Introducem funciile f:NN, g:NN i h:NN unde:
f(n) numrul maxim de bile pentru care din n ncercri putem afla bila defect,
g(n) numrul maxim de bile pentru care din n ncercri s se poat afla bila defect
considernd c naintea msurrilor ni se mai d o bil normal.
Introducem soluia x>y (sau x<y) care nseamn c am pus x bile cu poteniale defecte pe primul
taler i y pe al doilea i completm volumul cu bile mai puine astfel ca s fie acelai numr de bile
pe ambele talere i talerul cu cele y bile se va ridica( fiind mai uor).
h(n) numrul maxim m = x + y astfel nct avnd la ndemn o relaie de tipul x>y avem
o metod sigur de a afle care este bila defect din cele x + y bile.
Vrem s aflm deci, care este numrul maxim de bile m = f(n) din care putem gsii sigur
bila defect avnd voie s facem n cntriri. La prima cntrire punem A bile pe fiecare taler, B
fiind numrul bilelor nc nefolosite, i ne mai rmn n-1 cntriri.
f(n) = m = 2A + B
Presupunem h(n) = x + y unde x<y. Fie x = x1+x2+x3 i y = y1+y2+y3. Deci,
x<y x1+x2+x3 < y1+y2+y3. Fie
x1 bilele care raman pe primul taler

x = x2 bilele pe care le trecem pe al doilea taler


x bilele nefolosite la prima cntnt
3
y1 bilele care rm pe al doilea taler

y = y 2 bilele pe care le trecem pe primul taler


y bilele nefolosite
3
La prima cntrire vom pune pe primul taler x1 + y2 iar pe al doilea x2 + y1 bile.
Avem trei posibiliti:
a)
x1 + y2 < x2 + y1
x1 + x2 + x3 < y1 + y2 + y3
Evident, bila defect nu poate fi printre cele x3 i y3 dar nici printre x2 i y2 deoarece dac ar
fi de exemplu printre cele x2, atunci din x1+y2<x2+y1 ar rezulta c este bila grea iar din relaia
x1+x2+x3<y1+y2+y3 c este bila uoar. Deci, bila defect poate fi ori n x1 i este uoar ori n y1 i
este bila grea, deci
x1 < y1 (1)
b)
x1 + y2 > x2 + y1
x1+x2+x3< y1+y2+y3 de unde analog rezult
x2<y2 (2)

- 15 -

Petre Sergescu
c)

x1 + y2 = x2 + y1
x1+x2+x3 < y1+y2+y3 de unde rezult

x3< y3 (3)

Contopind cazurile a), b) i c) de mai sus, ajungem la concluzia c de la o relaie de tipul


x<y printr-o msurare ajungem la una din relaiile (1), (2), (3).
h(n) = x + y , dar din n-1 cntriri trebuie s gsim bila defect n oricare din cele trei
cazuri. Deci, putem considera ca x+y s fie h(n), adic maximul astfel nct din x<y s putem afla
bila defect din n cntriri. Deci, x1+y1, x2+y2 , x3+y3 trebuie s fie maximele astfel nct din
cazurile x1<y1, x2<y2, x3<y3 s putem gsii sigur bila defect din n-1 cntriri. Deci,
x1+y1 = x2+y2 = x3+y3 = h(n-1). Atunci,
h(n) = x+y = x1 + x2 + x3 + y1 + y2 + y3 = 3h(n-1)
Din h(1) = 3 i h(n) = 3h(n-1) se verific prin inducie c h(n) = 3n.
Cum 2A = A+Ah(n-1) = 3n-1 i cum 2A trebuie s fie numrul par maxim mai mic dect 3n-1,
rezult c 2A = 3 n-1-1. Deci
f(n) = 2A+B = 3 n-1-1 + g(n-1).
Pentru a afla o relaie pentru g(n), procedm analog cu deosebirea c trebuie s inem cont
de faptul c trebuie s avem n plus o bil normal. Deci
g(n) = 2A + 1 + g(n-1) = 3 n-1 + g(n-1).
n-1
Din g(1) = 1 i g(n) = 3 + g(n-1) avem:
g(1) = 1
g(2) = 3 + g(1)
g(3) = 32 + g(2)
.
g(n-1) = 3 n-2 + g(n-2)
g(n) = 3 n-1 + g(n-1),
adunnd egalitile, obinem:
3n 1
3n 1
3n 3
g(n) = 1+3 +3 2 + 33 + = 3 n-1 =
. Deci,
g(n) =
i f(n) =
.
2
2
2
n particular, Problema celor 12 bile avnd f(3) = 12, poate fi rezolvat prin 3 cntriri.
32 1
Din 12 = F(3) = 2A + B avem
2A = 3n-1 1 A = 4, B = g(2) =
=4
2
Deci, la prima cntrire se vor pune cte 4 bile numerotate a1, a2, a3, a4, i respectiv b1, b2,
b3b b4 pe cele dou talere. Bilele normale le notm cu P.
Avem cazurile:
1. Dac a1+a2+a3+a4 = b1+b 2+b 3+b4 atunci bila defect nu poate fi dect n cele patru bile rmase,
c1, c2, c3, c4. Punem pe talere pe c1+c2 i respectiv pe c3+P i putem avea cazurile:
1a.
c1+c2 = c3+P i rezult c c4 este bila defect pe care o comparm prin ultima
cntrire ( a treia) cu o bil normal.
1b.
c1+c2 > c3+P rezult c la ultima cntrire vom pune pe talere bilele c1 i c2.
2. a1 + a2 + a3 + a4 > b1 + b 2 + b3 + b4
Conform cazului general, mprim cele 8 bile n 3 grupe n mod ct mai echilibrate; 3,3,2.
La a doua cntriei punem pe cele dou talere a1+a2+b3 i respectiv b 1+a3+P. Putem avea cazurile:
2a.
a1+a2+b3 = b1+a3+P a1, a2, a3, b1, b3 sunt normale i a4+P<b2+b4. La ultima
cntrire punem pe talere pe b2 i b4.
2b.
a1+a2+b3>b 1+a3+P,
a1+a2+a3+a4>b1+b 2+b 3+b4 (*)
Presupunem b3 bil defect b3 este bila grea i din (*) rezult c b3 este bila grea , deci b 3
=P(bil bun). Analog se arat c a3 =P i relaiile 2b devin:
a1+a2+P=b 1+P+P,
a1+a2>b1+P
Deci, la ultima cntrire punem pe talere pe a1 i a2.
2c.
a1+a2+b3<b 1+a3+P,
a1+a2+a3+a4>b1+b 2+b 3+b4
Analog cazurilor precedente rezult c a1=P, a2=P, b1=P i din P+P+b3<P+a3+P tezult c
b3<a3. La ultima cntrire vom pune pe talerele balanei pe b3 i o bil normal P.

- 16 -

Petre Sergescu
II. Elemente de probabiliti geometrice

SSM
H

prof Popescu Octavian Tudor


Colegiul Economic F.S.Nitti Timisoara
Introducere
Reamintim definiia clasic a probabilitii:
Se consider o urn U care conine n bile, dintre care m albe i n-m negre Se extrage la ntmplare
o bil. Fie evenimentul bila extras este alb.
Definiie:
Se numete probabilitatea evenimentului A raportul dintre numrul cazurilor
favorabile realizrii evenimentului A i numrul cazurilor egal posibile.
m
Deci: P( A) = .
n
Aceast definiie se poate folosi numai n experimentele cu evenimente elementare egal
posibile. (cu probabiliti egale).
Plecnd de la definiia clasic a probabilitii putem rezolva ( intuitiv deocamdat)
probleme apelnd la probabilitile geometrice. Spre exemplu:
La o int circular de raz R se arunc cu o sgeat, la ntmplare. S se determine:
a) probabilitatea ca sgeata s se nfig n interiorul unui ptrat nscris n cercul de raz R .
b) Probabilitatea ca sgeata s se nfig ntrun triunghi echilateral nscris n cerc. Se tie c
toate sgeile ating inta i poziiile punctelor de nfigere a sgeilor pe int sunt egal
posibile.
a) Avem evenimentul
A : sgeata se nfige n ptratul nscris n cercul de raz R
nr. caz posibile toate punctele ptratulu Aria ptra 2 R 2 2
=
=
=
=
P( A) =
n caz favorabile toate punctele cercului
Aria cerc. R 2
b) A: sgeata se nfige n triungiul nscris n cercul de raz R .
3
R23
Aria triunghi
2 =3 3.
P ( A) =
=
2
Aria cerc.
R
4
Rezolvarea acestei probleme a plecat de la principiul definiiei clasice a probabilitii.
Vom furniza n cele ce urmeaz suportul matematic riguros care ne ndrum la rezolvarea
acestor tipuri de probleme.
Probabiliti geometrice
Orice familie de figuri geometrice F din spaiul euclidian r este dat de ecuaii de forma:
(1)

F (x1 ,..., xr ; 1 ,..., q ) = 0 ,

1 j r l , 1 l r 1

unde 1 ,..., q sunt parametri. Dac dm valori concrete parametrilor 1 ,..., q atunci obinem o
figur din respectiva familie. Deci exist o corespondern biunivoc ntre familia de figuri F i o
mulime D0 Bq a spaiului q cu elemente de forma ( 1 ,..., q ) .
Cu alte cuvinte o discuie asupra familiei de figuri F din Spaiul Euclidian, se reduce la o
discuie asupra mulimii D 0 a parametrilor ( 1 ,..., q ).
Iar dac F1 este o submulime a lui F atunci lui F1 i corespunde n q un domeniu D D 0 .
Unei familii de figuri F i se poate asocia un grup de invarian i anume un grup de
transformri care transform orice figur din F ntr-o alt figur din F (n cazul n care acest grup
exist).
Ex: Grupul de invarian al familiei dreptelor din plan este grupul transformrilor ortogonale.

- 17 -

Petre Sergescu
n spaiul r o familie de transformri este dat de funcii de forma:

a = (a1 ,..., a s ) cu legea funciei dat de

f i = f i (x1 ,..., xr ; a1 ,..., a s )

(2)

a : r r ;

1 i r

unde a = (a1 ,..., as ) sunt parametrii transformrii.

Deci Gr = {a : r r a = (a1 ,..., a s ) s }

a, b s a o b s (c s a. .a o b = c )
Dac punem condiia ca Gr s fie grup invariant al familiei F, acest lucru nseamn
(conform definiiei grupului invariant i innd cont de (1) i (2))
F j (x1 ,..., xr ;1 ,..., q ) = F j ( y1 ,..., yr ; 1 ,..., 2 ) ;
1 j r l
1 l r 1
Gr este grup dac

unde i = i ( 1 ,..., q ; a1 ,..., a s ) . ( 1 sunt gsii n funcie de parametrii familiei 1 ,..., q i


parametrii transformrii a1 ,..., a s )
Cu alte cuvinte unui grup invariant de transformri din r corespunztori unei familii de
figuri F i corespunde biuivoc un alt grup de transformri ale paramtrilor n spaiul q . Acest nou
grup o s-l notm s deoarece depinde de s parametrii (a1 ,..., a s ) s .
Adic s = { a : q q a = (a1 ,..., a s ) s }

s este grupul transformrilor care transform parametrii unei figuri din F n parametrii
altei figuri din F.
Urmrind ca la o deplasare ale figurilor din familia F rezultatul calculelor s nu se schimbe,
putem s dm urmtoarea definiie:
Definiie: O integral = ... (1 ,..., q )d1 ...d q se numete invariant integral al
D

grupului de transformri s dac

... (1 ,..., q )d 1...d q = ... (1 ,..., q )d1...d q


D

D'

unde D Bq este domeniul parametrilor

( ,..., )
1

corespunztor unei subfamilii F,iar

D=domeniul parametrilor (1 ,..., q )obinut prin aplicarea transformrilor (2). (deplasarea lui D)
Se poate demonstra c o condiie necesar i suficient ca integrala I s fie invariant integral
grupului s este ca
(3)

x ( ) = 0
2

k =1

kt

1 t s

unde kt sunt coeficienii transformrilor infinitezimale ale grupului s .


Acest sistem de ecuaii cu derivate pariale se numete sistemul lui Deitheil.
n continuare ne va interesa situaia cnd grupurile de transformri admit un invariant
integral unic, adic atunci cnd sistemul lui Deitheil admite o soluie unic , abstracie fcnd de
un factor constant.
Putem presupune c (soluia sistemului lui Deitheil) este nenegativ, cci dac
furnizeaz un invariant integral, atunci i furnizeaz un invariant integral.
Considerm D0 B q i : P(D0 ) B q R+ (D ) = ... (1 ,..., q )d1...d q

este o msur pozitiv.

Pentru c ntre submulimi de figuri ale lui F i submulimile domeniului D0 exist o


coresponden biunivoc, putem defini msura unei submulimi F1 a familiei F
(F1 ) ca fiind (D ) iar (F ) = (D0 )

- 18 -

Petre Sergescu

Fcnd o schimbare de variabile k = k 1 ,..., q

1 k q
se observ c
D( 1 ,..., q )
... (1 ,..., q )d 1 ...d q = ... 1 ,..., q
d 1 ...d q
D
D'
D 1 ,..., q

( )

) (

SSM
H

adic (D ) = D = (F1 ) msura unei mulimi de figuri F1 ale unei familii F nu depinde de
invariantul integral cu care se construiete msura respectiv. (la o deplasare a figurii rezultatul
calculelor nu se schimb).
Dac ( 1 ,..., q ) = 1 atunci (D ) = ... d1...d q = volumul domeniului D .
D

Dac F1 F este o submulime de figuri ale familiei F creia n spaiul parametrilor i corespunde
domeniul D , atunci probabilitatea ca un element luat la ntmplare din F s fie din F1 este
(4)

P = P(F1 ) =

( 1 ,..., q )d 1 ...d q
(F1 ) (D ) ...
=
= D
(F) (D0 ) ... ( 1 ,..., q )d 1 ...d q
D0

Prin aceast construcie se obine cmpul de probabilitate (D0 , D0 B q , P ) .


Cu alte cuvinte pentru rezolvarea unei probleme de probabiliti geometrice referitoare la o
familie de figuri date, se determin invariantul integral corespunztor grupului de transformri
considerat, apoi se determin submulimea din spaiul parametrilor ale crei puncte genereaz
submulimea de figuri a mulimii de figuri date i apoi se calculeaz probabilitatea cerut cu
formula (4).
Aplicaie
Sisteme de puncte din plan.
Fie n planul euclidian de coordonate carteziene ortogonale (x, y) familia de figuri format
din mulimea punctelor M (1 , 2 ) ; deci ecuaiile familiei sunt:
(5)
x = 1 , y = 2
cu 1 , 2 parametri. Grupul de invarian al acestei familii este grupul transformrilor din plan de
ecuaii:
x = x' cos y' sin + a
y = x' sin + y ' cos + b
Aplicnd acest grup familiei ecuaiile ( ) devin:
x ' = 1 , y ' = 2
unde:
(6)
1 = 1 cos 2 sin + a
2 = 1 sin + 2 cos + b
Acesta e grupul H 2(1 , 2 ) ataat grupului de invarian fa de familia (5 ).
innd seama de dezvoltrile n serie:
3 5
2 3
sin =
+ ...; cos = 1
+ ...
3! 5!
2! 3!
relaiile (6) devin:
1 = 1 2 0 + a + ...
2 = 2 + 1 0 + b + ...
coeficienii transformrilor infinitezimale ale acestui grup sunt:
11 = 1, 12 = 0 , 13 = 2 , 21 = 0, 22 = 1, 23 = 1
Sistemul lui Detheil ( ) se scrie:

- 19 -

Petre Sergescu

=0


=0

+ 1
=0
2
2
2

i are soluia = const. Avnd n vedere (4) putem lua = 1.


Deci msura mulimii F de puncte din planul xOy ai crei parametrii variaz ntr-o
mulimee mrginit D0 2 este

(F) = D0 d1d 2
sau, dac inem seama de (5)
(F ) = D dxdy = S (D0 )
0

unde S (D0 ) reprezint aria domeniului D0 din plan.


Exemple:
1. Pe segmentul OA de lungime l situat pe axa Ox se alege la ntmplare punctul B. S se determine
l
probabilitatea ca cel mai mic dintre segmentele OB i BA s aib lungimea mai mare ca .
3
l
Fie evenimentul A: cel mai mic dintre segmentele OB i OA are o lungime mai mare ca .
3
notm cu
x = min{OB , OA }
l
l
l
l l
reprezint domeniul D0 = 0, i x > ceea ce nseamn D = ,
2
3
2
3 2
l l

v (D ) 2 3 1
deci P( A) =
=
=
l
v ( D0 )
3
2

atunci x

2. Pe un segment AB de lungime l sunt fixate la ntmplare punctele C i D. Care este probabilitatea


ca punctul C s fie situat mai aproape de D dect de A? Poziiile punctelor C, D sunt egal posibile.
Fie punctele C, D n interiorul segmentului AB, unde A(0,0), B(l,0) cu l 0 , C(x,0), D(y,0).
Considerm evenimentul
E: punctul C s fie situat mai aproape de D dect de A
avem
x [0, l ], y [0, l ] .
deci
D0 = [0, l ] [0, l ]
iar x i y mai trebuie sa satisfac condiia
adic
deci

- 20 -

x> x y

D = {( x, y ) [0, l ] [0, l ] : x > x y }

l
l + l
v(D ) AriaTrapezului 2 3
P (E ) =
=
=
=
v(D0 ) AriaPatratului
2l 2
4

Petre Sergescu

SSM
3. Un indicator de semnale se compune din dou dispozitive. Fiecare din cele dou semnale esteH
egal posibil n orice interval de timp T. Indicatorul acioneaz dac diferena ntre momentele de
sosire ale semnalelor este mai mic ca t (t < T ) . S se determine probabilitatea ca indicatorul s
acioneze n timpul T, dac oricare din cele dou dispozitive trimite un semnal.
Considerm evenimentul
E: indicatorul acioneaz n timpul T
Notm cu x momentul sosirii semnalului de la dispozitivul nr.1 i cu y momentul sosirii semnalului
de la dispozitivul nr.2.
atunci
x (0,T ) i y (0,T )
adic
D0 = [0, T ] [0, T ]
pentru producerea evenimentului E se impune condiia
D = {( x, y ) [0, T ] [0, T ] : x y < t}

adic

P (E ) =

v(D ) AriaPatrulateruluiHasurat
=
=
v( D0 )
AriaPatratului

x y <t

T2 2

(T t )2

t (2T t )
T2

4. Se aleg la ntmplare dou numere pozitive x i y mai mici sau egale cu 2. s se determine
y
probabilitatea ca produsul lor s nu depeasc 1, iar
s nu depeasc pe 2.
x
y
E: produsul numerelor xy nu depete 1, iar
nu depete 2.
x
avem
x [0,2] , y [0,2]
rezult c
D0 = [0,2] [0,2]

x
D = (x, y ) [0,2] [0,2] : xy 1, 2
y

v (D )
AriaHasurata 1 + 3 ln 2
P (E ) =
=
=
v(D0 ) AriaPatratului
8

iar
deci

5. S se determine probabilitatea ca lungimea unei coarde ntr-un cerc de raz R s fie cuprins
ntre dou numere a i b.
Considerm ntr-un cerc de raz R, o coard aleas arbitrar, de lungime l.
atunci
l [0,2 R]
Vom restrnge domeniul ns, la un semicerc determinat n felul de mai jos, tocmai pentru a pstra
invariantul egal cu 1. n semicercul de diametru paralel cu coarda considerat, exist doua coarde
paralele deasemenea cu diametrul i coarda considerat, ce au lungimile a respectiv b.Mai
considerm i raza cercului ce trece perpendicular pe coardele considerate i care le intersecteaz n
punctele M i N.
D0 = [0, R ] iar D = segmentulMN
Deci

P(a l b ) =

v(segmentuluiMN ) 1
a2
b2
=
R2
R2
R
R
4
4

- 21 -

Petre Sergescu
III. Asupra unor identiti trigonometrice condiionate
Lucian Dragomir
Grup colar Industrial
Oelu-Rou, Cara-Severin
Propunem n nota de fa obinerea unor cunoscute identiti trigonometrice pe cale
nonstandard , folosind proprieti geometrice ale triunghiului . Este vorba despre :
A
B
C
( 1 ) sin A + sin B + sin C = 4 cos cos cos
;
2
2
2
( 2 ) sin 2 A + sin 2 B + sin 2C = 4 sin A sin B sin C ;
( 3 ) tgA + tgB + tgC = tgA tgB tgC ,
egaliti adevrate n orice triunghi ABC (deci A + B + C = ). O bun cunoatere a formulelor
trigonometrice i deprinderea de a le aplica conduce la obinerea acestor identiti pe cale
standard, aa cum sunt prezentate, de exemplu, n [1], pag. 205 sau n [3], pag. 196 i 228 .
n cele ce urmeaz vom folosi binecunoscuta proprietate geometric :
Dac M este un punct n interiorul unui triunghi ABC , iar x , y , z sunt distanele de la M la
laturile acestuia ( care au lungimile a , b , respectiv c ) , atunci
(*)
ax + by + cz = 2 S ,
unde S este aria triunghiului .
Particulariznd poziia punctului M n relaia precedent vom obine , pe rnd , identitile anunate
( i ) Dac M = I (adic M este centrul cercului nscris), folosind notaiile uzuale, avem evident :
A
B
C
x = y = z = r = 4 R sin sin sin
2
2
2
Egalitatea ( * ) devine succesiv , folosind i teorema sinusurilor :
(a + b + c) r = 2 2 R 2 sin A sin B sin C
A
B
C
A
B
C
A
B
C
8 R 2 (sin A + sin B + sin C ) sin sin sin = 32 R 2 sin sin sin cos cos cos
2
2
2
2
2
2
2
2
2
Doar o simplificare conduce acum la identitatea ( 1 ) .
( ii ) Dac M = O ( centrul cercului circumscris ) , avem :
x = R cos A, y = R cos B, z = R cos C
Egalitatea ( * ) conduce astfel la :
R (a cos A + b cos B + c cos C ) = 4 R 2 sin A sin B sin C ,
de unde , cu teorema sinusurilor , obinem imediat identitatea ( 2 ) .
( iii ) Dac M = H (ortocentrul triunghiului ABC) i D, E, F sunt picioarele nlimilor din A, B,
respectiv C, avem :
x = DH = 2 R cos B cos C ,
y = EH = 2 R cos C cos A ,
z = FH = 2 R cos A cos B .
Aceeai relaie ( * ) conduce la :
sin A cos B cos C + sin B cos C cos A + sin C cos A cos B = sin A sin B sin C
De unde , prin mprire cu cos A cos B cos C 0 ajungem la
tgA + tgB + tgC = tgA tgB tgC , adic ( 3 ) .
Bibliografie:
[1] Becheanu Mircea , Enescu Bogdan Manual pentru clasa a X a , Ed. Teora , 1999
[2] Lalescu Traian Geometria triunghiului , Ed.Tineretului , 1958
[3] Panaitopol Laureniu , Blun Mihai , Enescu Bogdan Manual pentru clasa a X a , Ed. Gil , 2000
[4] Vod Viorel Gh. Vraja geometriei demodate , Ed. Albatros , 1983

- 22 -

Petre Sergescu

SSM
H

IV. Generator de probleme

drd. Laviniu Bejenaru


C.N. TRAIAN
Deschiznd mai multe numere din revista dedicat elevilor (serioi i capabili) Gazeta de
matematic, prescurtat n aceast prezentare prin GM, se pot distinge o serie de probleme
comune. Deoarece generaiile de elevi se schimb, iar ei trebuie s nvee cam aceleai noiuni, este
cumva normal s se cam repete o serie de probleme scrise, desigur, sub alte forme. Att pentru
liceu ct i pentru coala general.
M voi opri asupra unor probleme pentru gimnaziu ct i asupra unor aspecte legate de
categoria de probleme discutat, legate de raionament. O categorie lucrativ pentru muli elevi este
tipul problemelor n care se d un numr scris sub forma unei expresii barate format cu cifre
zecimale gen abcd , unde a, b, c, d aparin bazei zece, desigur, deci sunt cifre ntre 0 i 9, cu a 0.
Dndu-se anumite cerine sau restricii asupra cifrelor, se cere s se determine care sunt aceste
numere sau cte sunt cele care verific proprietile impuse. Desigur, numrul de 4 cifre nu este
semnificativ, putnd fi mai multe sau mai puine; ns sub aspectul calculului matematic adresat
elevilor nu prea multe.
Pot interveni i probleme n care apar i alte elemente, cum ar fi puterile. n acest caz, nu
mai apare restricia ca prima dintre cifre s fie nenul, n lipsa altor precizri. Una din aceste
probleme este problema E:12649 din GM Anul CVIII, nr.12/2003. Enunul acestei probleme este
urmtorul:
E:12649. Determinai a, b, c, d N, tiind c
2 a + 3 b + 4 c + 5 d = 37.
Rezolvarea normal pentru un elev de gimnaziu este dat de urmtoarea ide de baz: se
observ c toate elementele din suma stng sunt pozitive, se determin valorile puterilor a, b, c, d
maxime dincolo de care suma din stanga ar depi valoarea din dreapta i se trece la o serie de
vcerificri pentru egalitate. Valorile maxime sunt: a maxim este 5, b maxim este 3, c maxim este 2,
d maxim este 2. S-a avut n vedere i faptul c celelalte elemente din sum au cel puin valoarea 1.
Soluia de verificare, computerizat, este prezentat n fiierul E _12649.cpp de mai jos. Rezolvnd
problema direct i/sau computerizat, obinem soluiile (0,3,1,1); (1,2,0,2); (2,3,0,1); (3,1,0,2);
(3,3,0,0); (5,1,0,0). Aceast problem se poate adapta, genernd noi probleme. Propun urmtoarea
problem:
(***) Determinai a, b, c, d, e N, tiind c
2 a + 3 b + 4 c + 5 d + 7 e = 43
Soluiile acestei probleme obinute foarte uor sunt n numr de 7 i anume: (0,2,0,2,1);
(2,1,1,2,1); (2,2,1,2,0); (2,3,1,0,1); (4,1,2,0,1); (4,2,2,0,0); (5,0,1,1,0) .
O alt problem generalizabil este E:13031 din GM Anul CX, nr.9/2005, numr aniversar,
110 ani de Gazeta Matematic. La muli ani! Enunul ei este
E:13031. Determinai elementele mulimii xyz | ( x y ) ( y z ) ( z x ) = 128 .
Prezentat spre rezolvare elevilor de gimnaziu, acetia trebuie s verifice relaia folosind
anumite abiliti, cum ar fi observaia c numrul din dreapta este par, deci produsul de cifre din
stnga trebuie s fie par, sau mai mult, c este o putere a lui 2. Aceste observaii mai reduc numrul
de cifre x, y, z care trebuie verificate n relaia de mai sus. Se obin soluiile: (1,5,9); (4,8,0);
(5,9,1); (8,0,4); (9,1,5). Corectitudinea poate fi verificat imediat folosind calculatorul printr-un
program corespunztor, prezentat n fiierul E_13031.cpp.
Putem genera foarte uor multe alte probleme cu soluii cu tot fr efort, computerizat. Voi
propune o nou problem:
(***) Determinai numrul de soluii i soluiile mulimii
xyzt | ( x y ) ( y z ) ( z t ) (t x) = 1024

- 23 -

Petre Sergescu
Utilitatea verificrii este important, mai ales c soluiile corecte se obin instantaneu.
Acestea sunt n numr de 13 i anume: (1,5,1,9); (1,9,1,5); (1,9,5,9); (4,0,8,0); (4,8,0,8); (5,1,9,1);
(5,9,1,9); (8,0,4,0); (8,0,8,4); (8,4,8,0); (9,1,5,1); (9,1,9,5); (9,5,9,1).
Utilitatea considerrii unor astfel de probleme este att la nivelul profesorilor ct i la
nivelul elevilor. La nivelul profesorilor, obinerea de probleme variate, orientate pe o anumit tem,
este o necesitate, cu att mai mult cu ct obinerea soluiilor corecte este imediat. Din punctul de
vedere al elevilor, variaia enunurilor n anumite limite sau exactitatea soluiilor nu sunt elemente
care i las indifereni pe cei care doresc s se perfecioneze.
Concluzia este c se poate verifica corectitudinea soluiilor multor probleme i se pot genera
o serie de probleme noi, n tiparele existente sau n tipare noi, destul de uor de obinut, la nivel
gimnazial. i pentru liceu, aceste aspecte se pot discuta.
Nu n ultimul rnd, a putea prezenta i alte probleme care pot fi generalizabile i pot avea
soluii calculate computerizat, n ideea c att elevii ct i profesorii i pot uura activitatea i pot
varia coninutul n anumite limite.
ab + cd 9
E:12869. S se afle toate numerele de forma abcd , tiind c
= .
ab cd 7
E:12997. Aflai numerele de forma abc pentru care are loc egalitatea: abc = 2 bc ba .
E_12649.cpp

E_13031.cpp

#include<iostream.h>
#include<conio.h>

#include<iostream.h>
#include<conio.h>

long a,b,c,d,nrsol,p1,p2,p3,p4;
long Putere(long m,long n) //returneaza m^n
{
long p,k;
if(n==0) return 1;
else
{
p=1;
for(k=1;k<=n;k++)
p=p*m;
return p;
}
}
void main(void)
{
for(a=0;a<=5;a++)
for(b=0;b<=3;b++)
for(c=0;c<=2;c++)
for(d=0;d<=2;d++)
{
p1=Putere(2,a); p2=Putere(3,b);
p3=Putere(4,c); p4=Putere(5,d);
if((p1+p2+p3+p4)==37)
{
cout<<Solutia <<++nrsol<< : ;
cout<<(;
cout<<a<<","<<b<<","<<c<<","<<d;
cout<<);
cout<<endl;
}
}
getch();
}

long x,y,z,nrsol,ValExp;

Soluii E_12649
(0,3,1,1); (1,2,0,2); (2,3,0,1); (3,1,0,2);
(3,3,0,0); (5,1,0,0)

- 24 -

void main(void)
{
for(x=0;x<=9;x++)
for(y=0;y<=9;y++)
for(z=0;z<=9;z++)
{
ValExp=(x-y)*(y-z)*(z-x);
if( (ValExp == 128) && (x != 0) )
{
cout<<"Solutia "<<++nrsol<<" : ";
cout<<"(" ;
cout<<x<<","<<y<<"," <<z;
cout<<")";
cout<<endl;
}
}
getch();
}

Soluii E_13031
(1,5,9); (4,8,0); (5,9,1); (8,0,4); (9,1,5)
Soluii E_12869
(1,6,0,2); (2,4,0,3); (3,2,0,4); (4,0,0,5);:
(4,8,0,6); (5,6,0,7); (6,4,0,8); (7,2,0,9);
(8,0,1,0); (8,8,1,1); (9,6,1,2)
Soluii E_12997
(3,1,2)

Petre Sergescu
V. PRIMUL CONCURS JUDETEAN DE GEOMETRIE
GHEORGHE TITEICA

SSM
H

Profesor pensionar STEFAN MARICA


Cu prilejul implinirii a 100 de ani de la nasterea marelui matematician
severinean Gheorghe Titeica (nascut la 4 octombrie 1873) ,Inspectoratul Scolar
Mehedinti ,Filiala Mehedinti a Societatii de Stiinte Matematice din Romania si
catedra de matematica a Liceului Traian ,au initiat in 1973 Concursul de
geometrie Gheorghe iteica pentru elevii claselor VII-XII.
Din comisia de organizare a concursului au facut parte urmatoarele cadre
didactice:
Profesor Mircea Chivu-Inspector General Adjunct
Profesor Dorin Radoslav-Inspector de matematica
Profesor Stefan Marica-Responsabil al concursului pentru clasele VII-VIII
Profesor Paunescu Alexandru-Responsabil al concursului pentru clasele IX-XII.
Datele concursului
Pentru clasele VII-VIII a doua duminica a lunii octombrie la Scoala Generala
Nr.1, unde a fost elev Gheorghe Titeica
Pentru clasele IX-XII a treia duminica a lunii octombrie la Liceul Traian.
Cateva din prevederile regulamentului acestui concurs:
1) Pot participa la concurs elevii scolilor din judetul Mehedinti care au aparut ca
rezolvitori sau propunatori de probleme in revistele de specialitate, cu deosebire
in Gazeta Matematica.
2) In scopul antrenarii elevilor pentru studierea si aprofundarea cunostiintelor
matematice, elevii vor expedia comisiei de organizare si desfasurare a
concursului cel putin un exercitiu sau problema originala.
3) Deoarece concursul are loc la inceputul anului scolar, programa de concurs
este cea studiata in anii scolari precedenti.
4) Fiecare concurent va avea de rezolvat 3 probleme si anume: o problema din
Gazeta Matematica publicata in ultimii doi ani, o problema sau exercitiu propus
de un profesor de matematica din Mehedinti, o problema din culegerea Gheorghe
Titeica, sau din manualele scolare.
5) Lucrarile scrise vor fi corectate de cate doi profesori, iar cele mai bune vor fi
revizuite de o alta comisie (aceasta in scopul unei evaluari corecte si pe acesta
baza stabilirea castigatorilor)
Experienta concursului a determinat comisia de organizare si desfasurare
sa modifice unele cerinte ale regulamentului.
Deoarece problemele propuse de concurenti nu erau dintre cele mai reusite
s-a renuntat la aceasta prevedere.
Din cauza ca la Scoala Generala Nr.1 nu erau conditii normale pentru
desfasurarea competitiei, intreg concursul s-a desfasurat in liceul Traian.
Incepand cu anul scolar 1978-1979, Facultatea de Matematica a
Universitatii din Craiova a organizat Concursul Interjudetean de Matematica
Gheorghe Titeica. Din aceasta cauza comisia judeteana a hotarat incetarea
concursului propriu incepand cu 1980.
Avem satisfactia ca am fost cei dintai care am organizat un concurs de
geometrie care a purtat numele unui matematician de talie mondiala.

- 25 -

Petre Sergescu
In cele ce urmeaza prezentam rezultatul primului concurs de geometriE
Gheorghe Titeica desfasurat pentru elevii din clasele VII-VIII.
Extras din Procesul Verbal 14-Octombrie 1973:
PREMIUL I
Balu Cristina Sc.Gen.3,Profesor I.Gaban
Glavici Marius Lic.Traian,Profesor St.Marica
Constantinescu Violeta Sc.Gen.14, Profesor Baluta Poganu
PREMIUL II
Stelescu Stefan, Sc.Gen.5, Profesor C.Covrig
Gurbina Mircea Sc.Gen.Nr.2, Profesor R.Bazacov
Ivaschescu Livia Liceul Traian, Profesor St.Marica
Buse Iustin ,Sc.Gen.9, Profesor El.Vicea
PREMIUL III
Serbanescu Cristian, Liceul Traian,Profesor Fl.Nica
Dragomir Nicolae, Sc.Gen.9, profesorA.Gugu
Preotesoiu Pompiliu, Sc.Gen.4,Profesor Ana Vladica
Balu Marinela,Sc.gen.Nr.6, Profesor El.Gimanca
Serbulescu Mihaela,Sc.Gen.6, Profesor el.Gimanca
MENTIUNI
Rotaru Dan, Liceul Traian, Profesor St.Marica
Vaduva Cristian,Sc.Gen.9, Profesor El.Vicea
Derjac Simona,Sc.Gen.6, Profesor Palina Iorga.
Ceilalti 55 de concurenti au obtinut un punctaj inferior.....

- 26 -

FOTI ELEVI MEHEDINENI

SSM
H

De la Severin la Rutgers
Nu stiu altii cum sunt, dar eu, cand ma gandesc la casa si familia in
care am crescut si la educatia pe care am primit-o, realizez cat de mult mau influentat in alegerea unei directii pe plan professional. In momentul de
fata lucrez la obtinerea unui doctorat in Mathematics Education (Stiintele
Educatiei in Matematica) in cadrul universitatii Rutgers din New Jersey,
Statele Unite. Adeseori ma intreb ce m-a adus pe aceste meleaguri, si
inevitabil ma uit la trecut.
Am crescut intr-o familie de matematicieni (atat ambii parinti cat si
unul dintre bunici fiind profesori de matematica de liceu), alaturi de o sora
geamana (Simona) si o sora mai mica (Speranta). Tot ce imi aduc aminte
despre experientele cu matematica in scoala primara si gimnaziala este ca
intelegeam conceptele si rezolvam problemele cu relativa usurinta. Sa fie
acest lucru datorat, in parte, unei mosteniri genetice? N-am nici cea mai
vaga idee. Cert este ca si surorile mele au manifestat aceeasi abilitate
matematica, in conditiile in care nu lucram la matematica aproape deloc
inafara scolii. Nu am facut niciodata meditatii, dar prezenta a doi profesori
de matematica in propria casa a ajutat cu siguranta. Ajutorul dat de parintii
nostri consta in indicatii subtile oferite atunci cand ne impotmoleam la
vreo problema din tema. Aceste indicatii erau destul cat sa ne scoata din
blocaj, dar nu contineau niciodata intreaga solutie. Uitandu-ma inapoi, imi
dau seama ca formularea acestor indicatii subtile este o arta, si numai
cineva cu ani de experienta in predarea matematicii o poate stapani.
Avand in vedere plenitudinea de resurse matematice avute la
indemana in copilarie, este curious ca nici una dintre noi, cele trei fete, nu a
manifestat un interes deosebit fata de matematica in clasele 1-12 (facute la
Scoala Generala Nr. 2, Liceul Gheorghe Titeica si Liecul Traian). Zambesc si
acum cand imi amintesc ca singurul fel in care am folosit maldarele de
culegeri de matematica prezente in casa noastra a fost ca suport pentru
cartile de joc. Au existat ambitii temporare de a obtine rezultate bune la
olimpiade de matematica si fizica (Simona si Speranta participand chiar la
fazele nationale ale olimpiadelor de matematica si respectiv fizica); eu nu am
avut ambitii in aceasta directie, ducandu-ma la olimpiade municipale si
judetene mai mult ca sa multumesc profesorii si parintii. Am avut parte de
profesori exceptionali (prof. Coada Carmen in gimnaziu si prof. Cainiceanu
George in liceu), si simteam ca se asteptau ca noi, elevii buni ai clasei, sa
reprezentam scoala la aceste concursuri. S-a intamplat sa nu am nici o
motivatie personala sa-mi compar cunostintele matematice cu altii, dar ce
consider ca am invatat in anii de gimnaziu si liceu de la acesti profesori este
ca matematica este un domeniu al gandirii logice si al cunoasterii prin
explorare.
Anii de facultate (Universitatea Bucuresti, Facultatea de Matematica)
nu au consolidat aceasta perceptie a matematicii. Majoritatea cursurilor
luate acolo m-au tarat intr-o lume abstracta in care nu mai stiam de ce
studiez acest domeniu. Consider ca am vegetat acei 4 ani m-am lasat
dusa de curentul facultatii, pregatindu-ma pentru examene mai mult din

- 27 -

FOTI ELEVI MEHEDINENI


obisnuinta decat din interes. In ultimul an de facultate am inceput sa fiu
interesata de studii in strainatate, si m-am hotarat sa fac un master in
Romania pentru a avea mai multe sanse sa fiu acceptata la o universitate
buna in Statele Unite, si mai ales sa primesc ajutor financiar. In timpul
acestui masterat am aflat accidental (datorita Simonei, sora geamana)
despre o pozitie de instructor de matematica la un colegiu de arte din
Vermont, Statele Unite. Am aplicat in ultimul minut si m-au angajat! Au
urmat doi ani de imersiune intr-o cultura total straina mie pana in acel
moment: m-am trezit brusc in pozitia de profesoara, cu care nu eram
obisnuita, intr-o tara unde nu cunoasteam pe nimeni. Asa am descoperit ca
imi priesc provocarile, si ca ma ajuta sa cresc atat pe plan profesional cat
si personal. Experienta acelor doi ani m-a ajutat in decizia de a-mi continua
studiile la nivel de doctorat, in domeniul stiintelor educatiei in matematica.
Asa am ajuns la Rutgers University, in New Jersey (la aproximativ o
ora de New York). In Statele Unite se pune mult accent pe constructivism
in educatie ideea principala este ca elevii trebuie sa-si construiasca
propriile idei ca sa priceapa matematica cu adevarat. Din acest punct de
vedere, rolul profesorului este acela de guide on the side (ghidul de pe
margine) si nu de sage on the stage (inteleptul de pe scena). Cu alte
cuvinte, profesorul trebuie sa creeze situatii in care elevul este lasat sa
exploreze o problema noua sau un concept nou, intervenind doar cand e
necesar. Departamentul de educatie de la Rutgers primeste fonduri de
milioane de dolari de la diverse organizatii pentru a investiga cum se pot
aplica diferite versiuni ale constructivismului in scoli. De ce? Pentru ca teste
internationale arata ca elevii americani sunt sub nivelul mediu la
matematica (clasele 8 si 12) al tarilor industrializate. Momentan, nu exista
consens in mediile academice in privinta modului in care sistemul
educational american poate fi imbunatatit.
Asadar, treburile sunt complicate la nivel macro intr-un sistem
educational necentralizat. In studiile mele, eu ma concentrez pe educatie la
nivel micro cum afecteaza diferitele tipuri de interactiune dintre elev si
profesor procesul de invatare, ce fel de activitati trezesc interesul elevilor,
etc. Inca nu am decis care va fi subiectul tezei mele de doctorat: poate fi pe
teme legate de intrebarile mentionate mai sus, sau pe teme privind calitatea
programelor de calificare a profesorilor. Cu putin noroc, voi absolvi in 2008.
Nu m-am decis inca ce voi face dupa terminarea doctoratului. Acum ca am
vazut cum ti se poate schimba cursul vietii in cateva zile (vezi episodul cu
Vermont), nu ma grabesc sa fac planuri pe termen lung!
Iuliana Radu
Graduate School of Education
10 Seminary Place
New Brunswick, NJ 08901
USA
tenis@rci.rutgers.edu

- 28 -

CERCUL DE MATEMATICA
Tem pentru grupa de performan la clasa a- V -a

SSM
H

Metode de rezolvare a problemelor de matematica


Prof. Victor Saceanu
Rezolvarea unei probleme de aritmetica, daca nu se apeleaza la metodele elgebrice, devine
dificila la nivelul claselor IV-VI, in cazul cand nu se cunoaste metoda sau metodele prin care
aceasta poate fi rezolvata.
De aceea in cele ce urmeaza, voi cauta sa prezint metodele de baza ce se pot folosi in rezolvarea
problemelor de aritmetica, urmate de 1-2 exemple concrete
Consider ca este necesar acest lucru, deoarece atat programa cat si manualele de matematica de la
nivelul claselor IV-VI, prezinta succinct sau nici nu amintesc de aceste metode.
1.Metoda figurative (grafica) se aplica in rezolvarea problemelor in care se cunosc:
a) Suma si diferenta a doua marimi:
O barac merge in sensul curentului unei ape cu 16km/h, iar contra curentului cu 12km/h.Care este
viteza barcii in apa statatoare si care este viteza apei?
Solutie: Fie:
------------- - viteza barcii in apa statatoare =Vb
-------- - viteza apa = Va
------------------ = 16 km/h
-----------De unde deducem ca 2xVb=16+12Vb=14km/hVa=2km/h
Obs: In acest tip de probleme daca deducem suma cu diferenta obtinem de doua ori marimea mai
mare, iar daca le scadem obtinem de doua ori marimea mai mica.
b) Suma sau diferenta si raportul celor doua marimi
Ce ora este acum, daca partea ramasa din zi este de 6x1/2 ori mai mica decat partea care a
trecut ?
Solutie: Fie:
--------- = partea ramasa = 1p
----------------------------------- = partea trecuta
Atunci 1p+6x1/2p=7x1/2 parti
24:7x1/2=3x1/5 ore
24-3x1/5=20x4/5 ore
Este ora 20 si 48 de minute

=24h

Intr-o clasa, numarul elevilor absenti este de 1/6 din numarul celor prezenti . Cati elevi
sunt in clasa, daca numarul celor prezenti este cu 25 mai mare decat numarul celor absenti ?
Solutie: Fie:
------ -elevi absenti

------------------------------ = elevi prezenti diferenta = 25 de elevi


1p-1/6p=5/6p
25:5/6=30 elevi sunt in clasa
c) Suma mai multor marimi si diferentele sau rapoartele ce exista intre ele.
Suam a trei numere naturale a,b,c este 900. Aflati numerele stiind ca b este de 3 ori mai
mare ca a si cu 25 mai mic decat c.

- 29 -

CERCUL DE MATEMATICA
Solutie: Fie:
------ = a

S = 900
--------------------- = b
------------------------ = c
Avem 7p+25=9007p=8751p=125
Deci numerele sunt a = 125; b = 375; c=400
2) Metoda comparatiei ( aducerii la acelasi termen de comparative )
Aceasta metoda consta in:
a) Eliminarea unei marimi prin scadere:
Exp: Daca 20 de caiete si 35 de creioane costa 54lei noi, iar 15 caiete si 9 creioane costa 26,70 lei,
sa se afle pretul uniu caiet si al unui creion.
Solutie : Avem asezarea:
20caiete----------35creioane--------54lei
15caiete-----------9creioane---------26,70lei
Facem ca marimea caiete, de exemplu, sa aiba aceiasi valoare in ambele cazuri si atunci luam prima
data de 3ori mai mult si a doua oara de 4ori mai mult si obtinem:
60caiete-----------105creioane---------162lei
60caiete-----------36creioane----------106,80lei
Prin scadere obtinem ca: 69creioane costa 55,20lei, de unde 1 creion costa 0,80lei . Inlocuind in
una din situatii creioanele obtinem ca un caiet costa 1,30lei.
d) Eliminarea unei necunoscute prin inlocuirea ei:
Exp: Un costum de haine costa cat doua uniforme scolare, iar o uniforma costa cat cinci camasi.
Sa se afle pretul fiecaruia din aceste obiecte, stiind ca 40de costume, 150 uniforme si 120de camasi
costa impreuna 81280lei.
Solutie: Avem ca 40costume costa cat 40x2=80uniforme
Uniforme avem acum : 80+150=230 care ele costa atat : 230x5=1150camasi. Deci in final avem
: 1150+ 120=1270 camasi care costa 81280lei, de unde o camasa costa 64lei, o uniforma costa
5x64=320lei si un costum costa 2x320=640lei.
3. Metoda falsei ipoteze consta in aceea ca ambele marimi sunt presupuse la fel si apoi se deduce
diferenta care apare, di ferenta din care se poate afla marimea care a ramas.
Exp: Intrebat fiind, cati porumbei si cati iepuri are, un elev a raspuns: In total sunt 51de capete si
132de picioare.Cati prumbei si cati iepuri are elevul?
Solutie: Presupunem ca ar avea numai prumbei. Atunci ar fi in total 51x2=102 picioare, deci mai
raman in plus 132-102=30picioare. Acestea provin din faptul ca un iepure are in plus
4-2=2picioare. Atunci cele 30picioare le impartim in grupe de cate 2 picioare si obtinem 15iepuri si
51-15=36porumbri
Exp: Intr-un bloc sunt 39apartamente cu 4 si 3 camere. Cate apartamente de fiecare fel sunt, daca
numarul total al camerelor este 141 ?
Solutie : Presupunem ca toate apartamentele sunt cu 3camere si am avea 39x3=117camere, deci
mai raman in plus 141-117=24camere .Acestea provin din faptul ca avem si apartamente cu 4
camere, deci cu una in plus. Cele 24camere ne dau si cele 24apartamenete cu 4camere iar 39-24=15
apartamente cu 3camere.

- 30 -

CERCUL DE MATEMATICA

SSM
4. Metoda drumului invers(retrograda) consta in aceea, ca in rezolvarea problemelor, o luam cu H
rationamentul de la datele din finalul lor, pana atunci ajungem la inceput, adica la cele cerute in
problema.

Exp: O taranca a vandut oua la 3cumparatori astfel: primului jumatate din oua ce le avea si inca o
jumatatede ou la al doilea, jumatate din oua ramase si inca o jumatate de ou, iar la al treilea
jumatate din oua cu care mai ramansese si inca o jumatate de ou si astfel le-a vandut pe toate. Cate
oua a vandut taranca ?
Solutie: Daca al 3-lea comparator nu lua jumatate de ou atunci el ar fi cumparat jumatate din
cantitatea care era si ar fi ramas in cos o cantitate de oua egala cu aceea cumparata de el, adica o
jumatate de ou si ouale sau terminat, inseamna ca el a cumparat 1 ou, pe care l-a gasit in cos dup
ace a cumparat al 2-lea. Deci al doilea a gasit in cos 1x1/2+1x1/2=3 oua si el a cumparat
1x1/2+1/2= 2 oua. Atunci primul a gasit in cos 3x1/2 + 3x1/2 =7 oua si a cumparat
3x1/2 +1/2=4oua. Deci taranca a avut 7oua
Exp: La un centru de vanzare a painii inainte de inchidere erau 4cumparatori care au cumpart toata
cantitatea de paine.Sa se afle cate paini au fost,daca fiecare comparator a acumparat jumatate din
painea care se mai gasea in momentul in care i-a venit randul si inca cate o jumatate de paine
Solutie: Daca al 4-lea comparator a terminat painea ce mai ramansese, inseamna ca el a cumparat
1/2 +1/2 =1 paine acre ramansese de la al 3-lea. Acesta a gasit 1x1/2 +1x1/2=3paini ce ramasesera
de la al 2-lea si el a cumparat 1x1/2+1/2=2paini. Al 2-lea inseamna ca a gasit 3x1/2+3x1/2=7paini
si a cumparat 3x1/2+1/2= 4 paini. Daca primul i-a lasat celui de-al 2-lea 7paini inseamna ca el a
gasit 7x1/2+7x1/2=15 paini , si a cumparat 7x1/2+1/2 =8paini. Deci au fost 15 paini.
Obs: Rezolvarea unei probleme de aritmetica se poate face, fie aplicand una din metodele expuse,
dar si combinate,deoarece problemele de aritmetica nu pot fi clasificate nici dupa datele lor nici
dupa al cerintelor lor.
Prin cele expuse in acest material, consider ca nu au adus o noutate in rezolvarea problemelor de
aritmetica ci am dorit sa reamintesc cele mai importante metode re rezolvare,spre a fi folosite la
clasa sau la pregatirea loturilor olimpice, evitandu-se metodele algebrice.
Bibliografie:
1. Metodica de predare a matematicii E.D.P 1954
2. Probleme de matematica pentru gimnaziu I.Petrica s.a
3. Culegere de probleme de matematica Bucuresti 1987
4. Gazeta matematica 1980 2005.

- 31 -

CERCUL DE MATEMATICA
Tema pentru grupa de performanta la clasa a- VII -a
Formule de calcul prescurtat
Angela Nioiu & Manuela Opria
Colegiul Tehnic Decebal
1. an bn= (a b )(an-1 + an-2b + + abn-2 + bn-1 )
Demonstraie
Pornim de la membrul drept i ajungem la membrul stng :
(a b )(an-1 + an-2b + + abn-2 + bn-1 ) = an + an-1b + an-2b2 + +a2bn-2 + ab n-1- an-1b
- an-2b2 - - a2bn-2 - abn-1 bn = an bn
2. a2k+1 + b2k+1 = (a + b)(a2k a2k-1b + a2k-2b2 + + b2k)
Demonstraie
Pornim de la dreapta la stnga i nmulim termen cu termen obinem:
(a + b)(a2k a2k-1b + a2k-2b2 + + b2k) = a2k+1 a2kb + a2k-1b2 + +ab 2k + a2kb + a2k-2b3 + - ab2k
+ b2k+1 = a2k+1 + b2k+1
3. (a + b)0 = 1
(a + b)1 = a + b
(a + b)2 = (a + b)(a + b) = a2 + 2ab + b2
(a + b)3 = (a + b)2(a + b) = (a2 + 2ab + b 2)(a + b) = a3 + 3a2b + 3ab2 + b3
(a + b)4 = (a + b)3(a + b)=(a3 + 3a2b + 3ab2 + b3)(a + b)=a4 + 4a3b + 6a2b2 + 4ab 3 + b4
(a +b)5 = (a + b)4(a + b)=( a4 + 4a3b + 6a2b 2 + 4ab3 + b 4) (a + b)
= a5 + 5a4b + 10a3b3 + 10a2b3 + 5ab 4 + b5
(a + b)6 = (a + b)5(a + b)=( a5 + 5a4b + 10a3b3 + 10a2b3 + 5ab4 + b5)(a + b)
=a6 + 6a5b + 15a4b2 + 20a3b3 + 15a2b4 + 6ab5 + b 6
Coeficienii acestor binoame ( coeficieni binomiali) se pot calcula din triunghiul lui Pascal dup
cum urmeaz:
1
n0
1
1
n1
1 2 1
n2
1 3 3 1
n3
1 4 6 4
1
n4
1 5 10 10 5
1
n5
1 6 15 20 15 6 1
n6
.
Exerciiu: Calculai: (a + b)7 .
Rezolvare:
Se observ pe baza exemplelor calculate anterior i din triunghiul lui Pascal urmtoarele:
(a + b)7 = (a + b)6(a + b)=( a6 + 6a5b + 15a4b2 + 20a3b3 + 15a2b4 + 6ab5 + b 6)(a + b)
= a7 + 7a6b + 21a5b2 + 35a4b3 + 35a3b4 + 21a2b5 +7ab 6 + b7
4. Identitatea lui Lagrange :
(i)
(a2 + b2)(c2 + d2) = (ac + bd)2 + (ad bc)2
(ii)
(a2 + b2 + c2)(m2 + n2 + p2) = (am + bn + cp)2 + (an bm)2 + (ap cm)2 + (bp cn)2
(iii)

Generalizare
Aplicaii:

- 32 -

a
b

a i bi +

i -1
i -1
i -1

2
i

2
i

(a b

1 i j n

a j bi )

CERCUL DE MATEMATICA
1. Artai c:
a) (2005 2006 1) M12

SSM
H

b) (4 2005 + 5 2005 ) M 9

2. S se calculeze folosind triunghiul lui Pascal:

a) (2 x 3)4

b) 5xy x 2

1 2
1
1
1
1
, a + 2 , a 3 + 3 , a 4 + 4 ,..., a n + n .
a
a
a
a
a

3. Dac

a+

4. Dac

a
b
a6 + b6
.
+
= 2 , calculai
b
a
a 3b 3

= 3 , calculai : a +

Bibliografie:
[1]
C. Nstsescu, C. Ni .a., Exerciii i probleme de algebr, EDP , Bucureti, 1981
[2]
Revista de matematic Minimath, Nr.1 /2005, Editura Reprograph

Tema pentru grupa de performanta la clasa a- VIII a


Probleme de algebr cu soluii geometrice
prof. Mihai Octavian Ungureanu
P.1.

Determinai numerele reale strict pozitive x i y care au suma S i produsul P.

Soluie BD = x, DC = y, BD + DC = x + y = S
AD BC
T .I .
2
^
AD = BD DC = xy = P
o
ABC , m( A) = 90
Triunghiul ABC poate fi construit dac i numai dac
paralela la BC intersecteaz semicercul de diametru S.
S = x + y 2 xy = 2 P
BC S
AO =
= , AD = BD DC = P
2
2
S 2 4P
DO 2 = AO 2 AD 2 DO =
2
x = BD = BO DO =

S S 2 4P
2

y = DC = DO + CO =

S + S 2 4P
2

- 33 -

CERCUL DE MATEMATICA
P.2.

Determinai numerele reale strict pozitive x i y care au diferena d i produsul p.

Soluie presupunem x > y , x y = d ,


Punctele M, A, B din figur sunt coliniare
MA = x, MB = y, AB = x y

MT 2 = MA MB = xy = p MT =
OT =

xy = P

AB x y d
=
=
2
2
2
^

T .P.

MOT , m(T ) = 90 MO =
o

y = MB = MO OB =

d 2 + 4p
2

d 2 + 4p d
2

d2 + 4p + d
2
Determinai numerele reale strict pozitive x i y care au suma S i diferena d.

x = MA = MO + OA =
P.3.

Soluie Fie x y, S = x + y, d = x y
A, M, B, C coliniare, M mijlocul [AC]
AM = x, MB = y
AB = AM + MB = x + y = S
BC = MC MB = x y = d
x = AM = AC/2 = (AB+BC)/2 = (S+d)/2
y = MB = MC - BC = (S+d)/2 d = (S - d)/2
P.4.

Determinai numerele reale strict pozitive x i y care au produsul p i raportul k.

BD x
= =k.
DC y
BD BC
AB
=
=k
= k
CD BC
AC
AB
BD = AD
AC

Soluie BD = x, DC = y, BD DC = xy = p,

AB 2
AC 2
BD AB
DBA ~ DAC
=
AD AC
AB
x = BD = AD
= p k = pk
AC
AB AD
AC
DBA ~ DAC
=
DC = AD
AC DC
AB
AC
1
p
y = DC = AD
= p
=
AB
k
k
AD = BD DC =

P.5.

p;

Determinai numerele reale strict pozitive x i y care au suma S i raportul k.


AB 2
= k (aceeai figur ca la problema 4)
AC 2
AB 2
AB 2
AB 2
k
S
= BC
=
BC

=S
y=Sx=
2
2
2
BC
k +1
k +1
BC
AB + AC

Soluie BD = x, DC = y, BC = x + y = S ,
T .catetei

x = BD =

- 34 -

CERCUL DE MATEMATICA
P.6.

SSM
H

Determinai numerele reale strict pozitive x i y care au diferena d i raportul k.

Soluie PA = x, PB = y, AB = PA PB = x y = d
d
x+ y
OT = , PO =
, PT = xy ( problema 2)
2
2
xy ( x y )
PT TO
POT TD =
=
PO
x+ y
2

xy( x y) 2
x y
TDO DO = TO TD =

( x + y) 2
2
( x y) 2
DO =
2( x + y )
y( x y ) DA x
DA TA 2
k
DA
TA 2
TA 2
DB = BO DO =
= =k ;
=

=
=
=

x + y DB y
DB TB 2
k + 1 DA + DB TA 2 + TB 2 AB 2

x( x y )
k
DA = DO + OA =
TA 2 =
d2

x+ y
k +1
2

TB 2
2
2
TA DB
AB = TA = kd ; DO = DA OA = d (k 1) ; PO = TO = d (k + 1)
DA =
=
AB k + 1
2(k + 1)
DO 2(k 1)
TB 2
TB 2
d
kd
y = PB = PO OB =
; x = PA = PB + AB =
k 1
k 1
2

TA 2

Tema pentru grupa de performanta la clasa a-IX-a


ALGORITMUL PLICULUI
prof. DRAGA-TATUCU MARIANA
prof. DRAGA-TATUCU PORFIREL
Se consider n 3, n N , n este impar, ansamblul cresctor al numerelor reale
A = {a1 < a 2 < ... < a n2 }, ale crei elemente sunt n progresie aritmetic cu r > 0 .
Se cere proiectarea unui algoritm pentru generarea cu elementele ansamblului A a unei matrici
ptratice A = (aij )n n , care s verifice simultan egalitile:

j =1

i =1

i =1

i =1

aij = aij = aii = ai ,ni +1 = S n ,

unde Sn este o valoare ce se va preciza.


Descriere algoritm.
Elementele ansamblului A precizat fiind n progresie aritmetic, rezult

A=

(a1 + a n2 ) n 2

.
2
Deoarece elementele matricei A = ( aij ) n n care se cere generat, verific condiiile din
enun, rezult S n =

(a1 + an 2 ) n
1
.Orice matrice ptratic A care verific condiiile
A; S n =

n
2

- 35 -

CERCUL DE MATEMATICA
precizate n enun se numete ptrat magic de ordin n, avnd suma magic S n care depinde de
elementele ansamblului A.
Dac n este par, generarea unui ptrat magic A cu elementele unui ansamblu A se face cu
ajutorul algoritmului permutrilor.
Pentru cazul n 3 , numr natural impar, pentru generarea unei matrici A cu elementele
ansamblului A care s verifice condiiile precizate n enun, vom utiliza metoda teraselor datorit
lei Bachet (sau metoda plicului).
Metoda teraselor const n urmtoarele: se consider un caroiaj ptratic de dimensiuni
(2 n 1) (2n 1) de tipul urmtor:
n caroiajul considerat, se numete origine a
1..n....2n-1
acestuia, mijlocul oricrei laturi frontiere al lui.
1
Deci, exist patru origini posibile.
Se definete sens de terasare (de generare)
.
.
sensul considerat de la originea aleas spre
mijlocul uneia din laturile frontier cea mai
n
.
apropiat. Deci, din fiecare origine pot fi
.
considerate dou sensuri de terasare.
.
Pentru a face o alegere, vom considera c
mijlocul laturii frontier cea mai din stnga este
2n-1
originea acestuia, iar sensul de la originea aleas
spre mijlocul laturii frontier cea mai de sus este sensul de terasare sau sensul de generare n
caroiaj.
Odat fixat originea i sensul, pe tot parcursul terasrii, sensul de terasare trebuie meninut.
Terasarea elementelor ansamblului n caroiaj se va face n ordinea cresctoare a valorilor lor, n n
terase paralele, fiecare teras avnd n elemente distincte din ansamblu.
Terasarea se va face astfel:
n urma terasrii tuturor
elementelor ansamblului A, n caroiajul
considerat se obine un romb valoric de
forma:

an
a 2n

.
.

a2
a1

an 2

a n+2
a n+1

a n2 n + 2
a n 2 n+1

2
4
1

2
4

Unind mijloacele laturilor rombului valoric i ducnd diagonalele ptratului obinut rezult
patru triunghiuri valorice exterioare ptratului i patru triunghiuri valorice interioare.
Triunghiurile valorice obinute au urmtoarele proprieti: oricare dou triunghiuri valorice,
unul exterior i unul interior ptratului, sunt complementare valoric, n sensul c dac o poziie
precizat dintr-un triunghi valoric exterior este ocupat de un element al ansamblului A, poziia
corespunztoare din oricare triunghi valoric, interior ptratului, este vid. Aceasta nseamn c,
dac un triunghi valoric exterior este suprapus pe oricare triunghi valoric interior se va obine un
triunghi valoric plin, adic un triunghi n care toate poziiile sale sunt ocupate de elemente distincte
ale ansamblului A considerat.

- 36 -

CERCUL DE MATEMATICA

SSM
un ptratH

Dac se numeroteaz cu 1 4 triunghiurile valorice exterioare pentru a obine


valoric plin ale crui elemente s reprezinte o matrice A = ( aij ) n n , care s verifice condiiile din
enun, suprapunerea triunghiului exterior peste triunghiul valoric interior trebuie s se fac astfel:
1 1 din dreapta
2 2 de jos
3 3 din stnga
4 4 de sus
Fiind dat ansamblul A cu n 3 numr natural impar, cu ajutorul metodei teraselor pot fi
generate 8 matrici distincte avnd aceeai sum magic Sn.

Caz particular.
S se exemplifice algoritmul
proiectat pentru urmtorul caz:
A = {1,3,5,7,...,49}, n = 5 .
Rezolvare:
(1 + 49) 5
S5 =
= 125
2
Matricea cutat este:
5 31 17 43 29

3
39 15 41 27
A = 13 49 25 1
37

47 23 9 35 11

21 7 33 19 45

3
1
11

19

31

17

43

29

39

15

41

27

13

49

25

37

47

23

35

11

21

33

19

45

31

39
49
47

43
41

- 37 -

CERCUL DE MATEMATICA
Tema pentru grupa de performanta la clasa a-IX-a
TEOREMA LUI CASEY
eleva Otilia Stretcu,
Liceul Teoretic ,,Gh. ieica
Pentru a prezenta subiectul acestei teoreme este necesar prezentarea urmtoarelor leme:
LEMA 1
Se dau cercul C(O,R) i cercurile C(O1,R1), C (O2,R2) tangente interioare cercului C(O,R), n
punctele A, respectiv B. S se arate c distana dintre cercurile C (O1,R1) i C (O2,R2) (lungimea
tangentei exterioare comune) este :
dAB = AB (R r1 )(R r2 ) .
R
Demonstraie:
Din trapezul dreptunghic O1O2B1A1 avem:
d 2AB =A1B12 =O1O22 (r1-r2)2.
(1)
Din O1OO2 obinem:
O1O 22 = OO12 +OO22 - 2 OO1 OO 2 cos =
=(R-r1)2 +(R-r2)2- 2 (R r1 ) (R - r2 ) cos .

(2)

2R AB
. (3)
2R 2
nlocuim (3) n (2) i apoi (2) n (1) i obinem:
AB
dAB =
(R r1 )(R r2 ) .
R
LEMA 2
Se dau cercul C (O,R) i cercurile C (O1,R1), C
(O2,R2) tangente exterioare cercului C (O,R), n punctele
A, respectiv B. S se arate c distana dintre cercurile C
(O1,R1) i C (O2,R2) este :
dAB = AB (R + r1 )(R + r2 )
R
Demonstraie:
Analog cu Lema 1, dar (2) se nlocuiete cu
(2)
O1 O 22 = ( R + r1 ) 2 + ( R + r2 ) 2 2( R + r1 ) ( R + r2 ) cos
obinndu-se n final relaia dAB = AB (R + r1 )(R + r2 )
(5)
R
Cum AB 2 = 2 R 2 2 R 2 cos cos =

TEOREMA LUI CASEY


Dac cercurile C (O1,R1), C (O2,R2), C (O3,R3), C (O4,R4) sunt tangente interior
cercului
C (O,R) (sau exterior) n punctele A,B,C,D C (O,R) astfel ca ABCD s fie
patrulater convex i dac notm cu dAB distana dintre cercurile C (O1,R1), C (O2,R2)
i analoagele, atunci avem:
d AB d CD + d BC d AD = d AC d BD .
Demonstraie:
Se exprim d AB , d CD , d BC , d AD , d AC , d BD conform formulei (4) (sau (5)) i se nlocuiesc n relaia:

- 38 -

CERCUL DE MATEMATICA

SSM
H
obinndu-se relaia lui Ptolemeu.

d AB d CD + d BC d AD = d AC d BD
Observaie:
1) Dac toate cele patru cercuri degenereaz n puncte gsim chiar teorema lui Ptolemeu, iar din
aceasta n cazul patrulaterului dreptunghic obinem teorema lui Pitagora.
2) Din a doua teorem a lui Ptolemeu, care afirm c dac ABCD este patrulater inscriptibil,
AC AB AD + CB CD
atunci
=
se poate demonstra o generalizare a acestei relaii i anume:
BD BA BC + DA DC
d AC d AB d AD + d CB d CD
=
.
d BD d BA d BC + d DA d BC
APLICAII:
1) Fie triunghiul ABC nscris n cercul O (O,R) i un punct D pe latura [BC]. Se consider
cercurile C i D tangente la O i AD; primul cerc este tangent i la BD iar al doilea la DC. S
se arate c cercurile C i D sunt tangente dac i numai dac m( BAD )=m( CAD ).

(D. Brnzei)

Rezolvare:
Se consider punctul E, intersecia dintre AD i cercul O. Fie M punctul de tangent al
cercului C cu BD, iar N punctul de tangen al cercului D cu DC. Aplicm teorema Casey pentru
cercurile degenerate B, C, E i C i avem: BE CM + CE BM = BC EP , unde P este punctul de
tangen al cercului C cu AD. Din teorema Casey pentru cercurile degenerate B, C, E i D obinem,
BE CN + CE BN = BC EQ , unde Q este punctul de tangen al cercului D cu AD.
Cercurile C i D dac i numai dac P=Q. Scznd relaiile precedente rezult:
BE (CM - CN) = CE ( BN BM) sau BE = CE , adic m( BAE )=m( CAE ).
2) Fie cercul circumscris unui triunghi ABC i I centru cercului nscris n triunghi. Se consider
cercul tangent laturilor AC, BC, respectiv n D,E i tangent cercului . S se arate c I este
mijlocul segmentului DE.
Rezolvare:
Este suficient s demonstrm c I [DE]. Notnd BE=x i AD=y, din teorema lui Casey
aplicat punctelor A, B, C i cercului rezult: xb +za =(a-x)c.
(6)
Din CE=ED a x = b y z = b a + x.
(7)
a(p b)
b(p a)
Din (6) i (7) obinem: x=
i y =
.
(8)
p
p
BE
AD
C' I
Din teorema transversalei I [DE] AC'
(9)
+ BC'
= AB
EC
DC
IC
unde CI AB = {C' } .
C' I
c
cb
x
ca
y
c2
=
, nlocuim n (9)

=
IC a + b
b+a ax a +b by a +b
ba(p b)
ba(p a)
+
= c , evident adevrat, dup ce am folosit (8).

pa ap + ab pb bp + ba

Din teorema bisectoarei

Observaie: Pentru a exersa aplicarea teoremei lui Casey se recomand rezolvarea


problemelor din bibliografie.
[1]
[2]
[3]
[4]

BIBLIOGRAFIE:

M. Ganga, ,,Probleme elementare de matematic pentru gimnaziu i liceu, Ed.Mathpress 2003.


L. Nicolescu i V. Boskff, ,,Probleme practice de geometrie, Editura Tehnic, Bucureti 1990.
Gheorghe ieica, ,,Probleme de geometrie, Editura Tehnic, Bucureti, 1982.
V. Vornicu, ,,Olimpiada de matematic - de la provocare la experien, Editura Gil, 2003

- 39 -

CERCUL DE MATEMATICA
Tema pentru grupa de performanta la clasa a-X-a
COMENTARII METODICE ASUPRA UNOR PROBLEME
DATE LA OLIMPIADELE SUA
Prof.dr.Gheorghe Cainiceanu
C.N.Traian, Drobeta Turnu Severin
1 Introducere
De-a lungul timpului am fost martorii mai multor tipuri de filosofii; de la credinta ca noi
suntem cei mai grozavi de pe pamant pana la paguboasa insinuare ca suntem mici si saraci si
neimportanti. Scopul acestui articol este sa arate cum pe o structura ce imita metodele romanesti de
selectie ,o organizare superioara si o treaba bine facuta pot in timp da rezultate remarcabile... Sa nu
uitam ca in urma cu 15 ani SUA nu prea conta in lupta pentru medalii la Olimpiada Internationala
de Matematica iar in prezent intr-un clasament cumulat pe anii 1997-2001, SUA se afla pe locul
doi, Romania situindu-se pe locul 8.
In SUA selectia lotului national se desfasoara dupa urmatorul algoritm:
American Mathematics Competitions AMC 10/12 (Olimpiada locala- prin corespondenta,
subiecte unice, concurs deschis unui numar mare de scoli agreate din lume)
American Invitational Mathematics Competitions AIME (Olimpiada judeteana- prin
corespondenta, participa primii aproximativ 1% din concurentii de la AMC 10 si 5% din
concurentii de la AMC 12).
USAMO (Olimpiada Nationala- participa doar elevii americani care au intrunit un punctaj cumulat
de peste 200p din maximul de 300p realizabil).
Test Selection (baraje).
MOSP (Mathematical Olimpiad Summer Program- Pregatirea lotului).
In articolul de fata ma voi opri asupra a doua probleme din faza AIME, alese oarecum la cei
doi poli de dificultate. Mentionam ca regulamentul concursului prevede ca elevii sa scrie pe fisa de
concurs doar un rezultat numeric de trei cifre la fiecare din cele 15 probleme ce treebuiesc rezolvate
in trei ore.
2 Problema 1 AIME 1998
Pentru cate valori ale lui k numarul 1212 este cmmmc al numerelor 6 6 , 88 si k ?
Comentariu Este de remarcat faptul ca acesta problema este practic accesibila unui elev
chiar de clasa a V-a. De ce o astfel de problema? Iata ce scrie d-l Steven dunbar Directorul AMC
intr-o scrisoare adresata parintilor elevilor participanti la AMC: ...problemele sunt in afara
tiparelor (outside of the box)...chiar daca fiul sau fiica dumneavoastra rezolva o singura problema
sau doua din concurs trebuie sa fiti mandrii de el caci aceste probleme sunt altfel decat problemele
obisnuite de la clasa...
Solutie 6 6=26-36; 8 8=2 24 ;12 12=224-312. desigur vom lua k=312-2 p cu p lund valori de la 0 la
24 deci raspunsul este 025
3 Problema 13 AIME 1998
Daca {a1,a2,...a n} este o multime de numere reale cu elementele ordonate strict crescator
vom denumi CPS a sa numarul
a1i + a2i2 + a3i3 +....anin.
Fie Sn suma CPS corespunzatoare tuturor submultimilor nevide ale ale multimii {1,2,n}. Daca S8
=-176- 64i si S9 = p+qi determinati |p| +|q|.

- 40 -

CERCUL DE MATEMATICA

SSM
Se observa deja un nivel de dificultate sporit ,in primul rand pentru intelegereaH

Comentariu
corecta a enuntului. Se simte si se va confirma pe parcursul rezolvarii ca problema se adreseaza
unui elev de cel putin clasa a X-a, fiind necesare cunostinte de numere complexe, dar si de
combinatorica pentru numararea submultimilor.
Solutie Analizam posibilitatea calculului lui S9.Multimea {1,2,...9} are 29 1 submultimi
nevide.Sumele care apar din cauza submultimilor de un element le vom nota cu T1(1) , T1(2) , ....T1(9)
si desigur

T
k =1

(k )
1

= 45i. Sumele datorate submultimilor de doua elemente le vom nota T2(1,2) , T2(1,3)

T
Evident S9 = T1 + T2 + T3 +.
,.....,T2

(8,9)

. De exemplu T2(k,l) = ki+li2 si

( k ,l )
2

1k <l 9

(ki l).

1 k <l 9

T8 + T9.

Apar doua intrebari

" Nu este prea complicat sa calculam aceste sume ? "


"De ce am primit in enunt S8? "
Esta firesc chiar sa ne intrebam daca se poate generaliza problema. Se naste astfel ideea de a
incerca gasirea unei relatii de recurenta intre Sn+1 si Sn.
Sa urmarim din cine este formata Sn+1. Submultimile lui {1,2,....,n,n+1} sunt toate submultimile lui
{1,2,...,n} si apoi toate submultimile lui {1,2,...,n} la care se adauga n+1. Vom evita submultimea
vida care nu are o CPS.
Pentru inceput constatam ca Sn+1 = Sn + Un+1. Sa ne apucam de calculul lui Un+1. O submultime ce
contine n+1, si are k+1 elemente este de forma a1<a2<...<ak<n+1.
CPS = a1i + a2i2 + ....+akik + (n+1)ik+1,
Deci Un+1 este formata din CPS-uri date de toate submultimile nevide ale lui {1,2,...n}si la fiecare
din ele (n+1)ik+1 in numar egal cu Cnk. Deci
Un+1=(n+1)i+Sn+(n+1)i2 Cn1+(n+1)i3 Cn2+....+(n+1)in Cnn-1+ (n+1)in+1Cnn =
Sn+(n+1)i(1+i)n.
In concluzie
Sn+1 = 2Sn + (n+1)i(1+i)n ,deci
S9 = 2 S8 + 9 i(1+i)8 = - 352 +16i, si deducem ca |p| + |q| = 368.
Rezolvarea problemei s-a incheiat dar ea a deschis urmatoarea intrebare: putem gasi formula
termenului general pentru Sn din moment ce avem o relatie de recurenta?
Raspunsul este da, iar calea de urmat este urmatoarea:
Sn = 2 Sn-1 + i n (1+i)n-1
2 Sn-1= 22 Sn-2 + 2 i (n-1) (1+i)n-2
....................
2n-2 S2 = 2n-1 S1 + 2 n-2 i- 2 -(1+i)1
prin adunare vom obtine:
Sn = 2n-1S1 + i[20n(1+i)n-1 + 21(n-1)(1+i)n-2 + ....+2n-2-2(1+i)1]
Cum S1=i deducem:
Sn = i[20n(1+i)n-1 + 21(n-1) (1+i)n-2 + ....+2 n-2-2(1+i)1 + 2n-1-1-(1+i)0]=
1 + i n-1
1 + i n-2
1+ i 0
=i-2 n-1[n(
) + (n-1) (
) + ....+ 1-(
) ].
2
2
2
1+ i
Cu notatia z=
avem ca Sn = 2n-1-i (1+2z+3z2+...+nzn-1) i prin inmultire cu 1-z
2
1 zn
(1-z) Sn = 2 n-1i [1+z+....+zn-1 nzn] = 2n-1i [
- nzn].
1 z
Dupa o noua inmultire cu (1-z)-1 = 1+i , obtinem
1+ i n
1+ i n
Sn = 2n-1i [2i(1-(
) ) n(1+i)(
) ]= ....=(n+1+i)(1+i)n-1 2n.
2
2
Sa verificam acum rezultatul obtinut pentru cazul n=8:
S8 = (9+i)(1+i)7 256 = - 176 64i.

- 41 -

CERCUL DE MATEMATICA
Sa remarcam in finalul acestei prezentari frumusetea si abordabilitatea problemelor, naturaletea lor,
deschiderea lor spre generalizari si incitarea elevului spre o cercetare mai profunda dupa ce a
obtinut rezolvarea.
Mai este de remarcat faptul ca unele probleme pot avea abordari informatice, lucru care apropie
elevii, cunoscuta fiind pasiunea crescanda a lor pentru calculator.
BIBLIOGRAFIE

[1]
[2]

American Invitational Mathematical Examination AIME 1998


Titu Andreescu, Zuming Feng USA and International Mathematical Olympiads 2001
Published and distributed by The Mathematical Association of America 2001

Tema pentru grupa de performanta la clasa a-X-a


Tetraedrul i capcanele lui
Prof: Ion Chilea
Elev: Aida Giurcan
Motto: ,,Geometria tetraedrului are strlucirea i duritatea diamantului
Definiia 1 Se numete ,,nlime a unui tetraedru perpendiculara dus dintr-un vrf al
tetraedrului pe planul feei opuse.
Notaii
Vom nota tetraedrul de referin (A)= A1 A2 A3 A4 . Planul feei opuse vrfului Ai , i= 1,4 va fi notat

(A ) = (A A A ) ,
i

( Ai ) = A j Ah Ak

{i,j,k}={1,2,3,4}. Faa opus vrfului Ai este un triunghi ce se va nota cu

( )

( )

i aria sa cu S i , i = 1,4 . Unghiul dintre planele Ai i A j va fi notat cu ij .

Lungimea muchiei Ai A j se va nota cu a ij , iar piciorul nlimii duse din vrful Ai va fi notat cu

H i , i = 1,4 . Lungimea segmentului Ai H i va fi notat cu hi , i = 1,4 .


Vom enuna n continuare cteva teoreme referitoare la volumul tetraedrului.
Teorema 1 nlimile unui tetraedru sunt invers proporionale cu ariile feelor
corespunztoare, adic
S1h1 = S 2 h2 = S 3 h3 = S 4 h4
Demonstraia o lsm pe seama cititorului.
Definiia 2 Prin volumul tetraedrului (A) nelegem numrul notat V=v ( A1 A2 A3 A4 ) i
Sh
definit prin
V= i i , i = 1,4 .
3
Teorema 2. Fiind dat tetraedrul (A) i punctele Bi A4 Ai , i = 1,3 are loc relaia:
v( A1 A2 A3 A4 ) a1 a 2 a 3
=
unde am notat a i = A4 Ai i cu bi = A4 Bi , i = 1,3.
v( A4 B1 B2 B3 ) b1 b2 b3
Teorema 3. (Dostor- 1967). Volumul V al tetraedrului (A) este dat de formula:
2 S i S j sin ij
V=
, {i,j,h,k}= {1,2,3,4}.
3a hk
Demonstraie. Cu notaiile din figura 1, avem:
a A P 2 sin 12 2 S 2 sin 12
h1 = A1 P1 sin 12 = 34 1 1
=
de unde, n virtutea definiiei 2, rezult:
2
a 34
a34
Sh
2 S S sin 12
V= 1 1 = 1 2
.
3
3a 34

- 42 -

CERCUL DE MATEMATICA

SSM
H

Consecine:
1. n tetraedrul (A), are loc relaia:
a12 a34
a13 a 24
a14 a 23
4S S S S
=
=
= 1 2 23 4 (teorema sinusurilor n tetraedru)
sin 12 sin 34 sin 13 sin 24 sin 14 sin 23
9 V
Demonstraie. Din teorema 3, avem:
2 S S sin 12 2 S 3 S 4 sin 34
4 S S S S sin 12 sin 34
V= 1 2
=
de unde V 2 = 1 2 3 4
din care
3a34
3a12
9a 34 a12
a12 a34
4S S S S
obinem:
= 1 2 23 4 .
sin 12 sin 34
9V

Teorema cosinusului.
Teorema 4 (a proieciilor).ntr-un tetraedru (A), are loc relaia
S i = S j cos ij + S h cos ih + S k cos ik , {i,j,k,h}= {1,2,3,4}.
Demonstraie. Dac H 1 este proiecia lui A1 pe faa S1 avem:

) (

) (

S1 = s H 1 A3 A4 + s H 1 A2 A4 + s H 1 A2 A3 = S 2 cos 12 + S 3 cos 13 + S 4 cos 14 .

Definiia 3 Vectorul notat cu S i , avnd direcia i sensul vectorului Ai H i i modulul S i ,

poart numele de vector al feei ( Ai ), i = 1,4 .


Teorema 5 (a ,,ariciului).
S1 + S 2 + S 3 + S 4 = 0 .

Demonstraie. Notm aij = Ai A j . Avem 2 S 4 = a12 a13 , 2 S 3 = a14 a12 , 2 S 4 = a13 a14

) (

2 S1 = a 24 a13 = a14 a12 a13 a12 = a14 a13 a14 a12 a12 a13 .

Din aceste relaii avem:

2 S1 + S 2 + S 3 + S 4 =

= a14 a13 a14 a12 a12 a13 + a14 a12 + a13 a14 + a12 a13 = a14 a13 a14 a13 = 0
Teorema cosinusului n tetraedrul (A) au loc relaiile:
(a) S 42 = S12 + S 22 + S 32 2 S1 S 2 cos 12 2 S1 S 3 cos 13 2 S 2 S 3 cos 23

(Carnot)
(b) S + S 2 S1 S 2 cos 12 = S + S 2 S 3 S 4 cos 34 .
Demonstraie:
(a) Se folosete teorema ariciului pentru S 4 , i apoi calculm S 42 = S 4 S 4
(b) Din teorema ariciului avem:
S1 + S 2 = S 3 + S 4 apoi ridicm la ptrat.
2
1

2
2

3
3

2
4

Teorema 6 (Schwering). Dac n tetraedrul (A) notm cu S =

S1 + S 2 + S 3 + S 4
, atunci are
2

loc relaia:
34
9 2
2
V (a12 a 3 a 4 ) = (S S1 )(S S 2 )(S S 3 )(S S 4 ) S1 S 2 S 3 S 4 cos 2 12
.
16
2
2 S S sin 12 2 S 3 S 4 sin 34
Demonstraie. Avem conform teoremei 2, V = 1 2
=
.
3a 34
3a12

- 43 -

CERCUL DE MATEMATICA
Tema pentru grupa de performanta la clasa a-XI-a
Asupra teoremei lui Knaster
Elev Rapcea Mihai
Clasa a XII-a CNT
n acest articol ne propunem s analizm o problem de punct fix dat ca teorem n
lucrarea Manual pentru grupele de performan, clas a XI-a. Vom ncepe prin a da definiia unui
punct fix al funciei.
Fie A R o mulime, f : A A o funcie. x 0 A se numete punct fix al funciei dac

f ( x0 ) = x0 .
Observaie: o funcie f are cel puin un punct fix dac i numai dac graficul funciei f
intersecteaz prima bisectoare.
n lucrarea mai sus menionat, teorema lui Knaster este enunat n felul urmtor:
Fie f : [a, b] [a , b] o funcie monoton. Atunci exist c [a, b ] astfel nct f(c)=c.
Demonstrm acest rezultat pentru o funcie monoton cresctoare.
Fie B= {x A : f ( x) x} . Cum f (a ) a rezulta ca a B si deci B . Fie c=supB.
Deoarece c x, x B si f crescatoare, rezulta ca f (c) f ( x), x B, deci f (c) x, x B,
adica f (c) sup B = c . Atunci f ( f (c)) f (c), deci f (c) B f (c) sup B = c . Din relatiile
f (c) c, f (c) c rezulta ca f (c) = c.
Cu toate acestea, teorema este enuntata gresit deoarece aceasta nu se verifica pentru functii
monoton descrescatoare. In acest sens, oferim un comtraexemplu mai jos folosind metoda grafica.

O functie care ilustreaza contraexemplul de mai sus, data prin lege de asociere este:

f : [ 0 , 2 ] [ 0 , 2 ], f ( x ) =

- 44 -

1
2
3
2

x + 2 , x [ 0 ,1]
x + 3 , x (1, 2 ]

CERCUL DE MATEMATICA
Tema pentru grupa de performanta la clasa a-XII-a

SSM
H

Caracteristica unui inel

prof. Nedeianu Dan,


Grupul Scolar Domnul Tudor

n cele ce urmeaz ne propunem s aprofundm noiunea de caracteristic a unui inel


(ntlnit n algebra superioar) prin unele probleme propuse la diverse concursuri colare.
Definiie Fie (A ,+,) un inel, cu elementul unitate 1A i elementul nul 0 A .
1. Dac exist n * cu n 1A = 0 A (adic 1A + 1A + ... + 1A + 1A = 0 A ), atunci cel mai mic n cu
144424443
de n ori

aceast proprietate se numete caracteristica inelului A.


2. n caz contrar, adic n 1A 0 A , n * , spunem c inelul A are caracteristica zero.
Notaie Dac inelul A are caracteristica n * , notm char(A)=n.
Observaii:
1) Dac char(A)=n1, atunci n este ordinul elementului 1A n grupul (A,+).
2) Inelul Z are caracteristica zero, cci n10, n * .
3) Pentru n2, inelul n are caracteristica n, cci grupul ( n ,+) este ciclic generat de 1 , i deci
ordinul elementului 1 n acest grup este n.
Propoziia 1
Un inel integru are caracteristica zero sau un numr prim.
Demonstraie: Artm c n cazul cnd caracteristica este nenul, aceasta este un numr prim. Fie
deci (A,+,) un inel integru cu char(A)=n1. Presupunem prin absurd c n nu este prim
n= n1 n2 cu n1 , n2 i 1< n1 < n, 1< n2 < n; atunci avem c:
n 1A = 0 A , (n1 n2 ) 1A = 0 A 1A + 1A + ... + 1A + 1A = 0 A ( 1A + 1A + ... + 1A + 1A )
144424443
144424443
de n 1 n 2 ori

( 1A + 1A + ... + 1A + 1A )= 0 A
144424443

de n 1 ori

( n1 1A ) ( n2 1A )= 0 A . Cum A este inel integru (adic nu are divizori

de n 2 ori

ai lui zero), deducem c ( n1 1A )= 0 A sau ( n2 1A )= 0 A ,ceea ce contrazice faptul c n * este cel


mai mic cu proprietatea n 1A = 0 A .
Consecin: Caracteristica unui corp este zero sau un numr prim.
Demonstraie: evident, cci un corp este i inel integru.
Propoziia 2
Fie A un inel comutativ de caracteristic p numr prim. Artai c:
a) Funcia f:A A, f(x)= x p este un endomofism al inelului A (endomorfismul lui Frobenius)
b) Pentru n * i orice x, y A avem egalitatea: ( x + y ) = x p + y p .
Demonstraie:
p
a) Avem f(xy)= ( xy ) = x p y p = f ( x ) f ( y ), x, y A si f(1A)=1 A.
pn

f ( x + y ) = ( x + y ) = x p + C 1p x p 1 y + ... + C pp 2 x 2 y p 2 + C pp 1 xy p 1 + y p
p

Dar p prim divide pe C1p , C 2p ,..., C pp 1 C 1p x p 1 y = C p2 x p 2 y 2 = ... = C pp 1 xy p 1 = 0 A i atunci

f ( x + y ) = x p + y p = f ( x ) + f ( y ), x, y A , deci f este endomorfism.

- 45 -

CERCUL DE MATEMATICA
def
n
g ( x ) = f o f o ...o f ( x ) = x p , care este tot endomorfism, i atunci

n ori

g ( x + y ) = g ( x ) + g ( y ), x, y A, adic exact relaia cerut.

b) Se observ c

Propoziia 3
Fie K un corp comutativ de caracteristic p * . S se arate c exist un singur morfism de
grupuri de la grupul aditiv (K,+) la grupul multiplicativ (K * ,)
Demonstraie
S-a vzut c
(x + y ) p = x p + y p , x, y K (din propoziia precedent). Atunci

(x y ) p = x p + ( 1 )p y p ; pentru p3, p fiind prim (x y ) p


(x y )2 = x 2 + y 2 = x 2 y 2 (cci 2 y 2 = 0 K )
p
Cu alte cuvinte am dedus c ( x y ) = x p y p , x, y K .
Fie f:(K,+)

= x p y p , iar pentru p=2

( K * , ) un eventual morfism. Artm c este cel constant. ntr-adevr, pentru x K ,

avem px= 0 K i deci 1K =f( 0 K )=f(px)=f(x) p 0 K =f(x) p - 1K =(f(x)- 1K ) p , de unde f(x)=1K


x K .
Propoziia 4
Fie A un inel finit cu cel puin dou elemente. Atunci char(A) divide card(A), card(A) fiind
numrul de elemente din A.
Demonstraie
Presupunem c card(A)=n2, iar char(A)=m2, m, n * . Se tie c notnd cu H subgrupul ciclic
generat de elementul 1K A (al grupului (A,+)), din teorema lui Lagrange: card(H)|card(A)
Dar card(H)= char(A) , cci H={ 1A , 1A + 1A , , 1A + 1A + ... + 1A + 1A }, deci m|n.
14442444
3
=m
m -1 ori

Propoziia 5
Dac (A,+,) este inel finit cu p elemente, p prim, atunci A este corp comutativ cu char(A)=p.
Demonstraie:
Fie n=char(A), deci elementul 1A A are ordinul n n grupul (A,+); fie H{ 1A , 1A + 1A , ,
1A + 1A + ... + 1A + 1A }, subgrupul generat de 1A , n grupul (A,+) atunci card(H)|card(A), deci n|p i
14442444
3
n -1 ori

cum p prim n=p, deci char(A)=p A=H={ 1A , 1A + 1A , , 1A + 1A + ... + 1A + 1A }.


14442444
3
n -1 ori

()

Definim f: Z p A , f i = 1A + 1A + ... + 1A , evident izomorfism de inele A corp comutativ.


1442443
i ori

Propoziia 6
Fie A inel, 0 A 1A , cu proprietatea c grupul elementelor inversabile ale inelului are un numr
impar de elemente. Atunci char(A)=2.
Demonstraie:
Dac (U(A,)) este grupul elementelor inversabile din A, din enun card(U(A))=2n+1, n * ; avem
2 n +1
c - 1A U ( A) ( 1A )
= 1 -1A = 1A
1A + 1A =0 char(A)=2.
S-a folosit c ntr-un grup(G,) finit cu n elemente i e element neutru, avem c x n = e, x G .
Propoziia 7
Fie A inel cu propietatea c ()n N * fixat astfel ca x 2n = x, x A . Atunci char(A)=2.
Demonstraie:

- 46 -

CERCUL DE MATEMATICA

SSM
H

Avem (- 1A ) = -1A 1A = -1A 1A + 1A = 0 char(A)=2.


Observaie:
n particular orice inel boolean (inel A n care x 2 = x, x A ) este de caracteristic 2.
2
Mai mult, un inel boolean este i comutativ, pentru c ( x + y ) = x + y implic
x 2 + xy + yx + y 2 = x + y , deci xy+yx=0 x, y A i cum xy+xy=0, rezult xy=yx x, y A .
n continuare concretizm noiunea de caracteristic a unui inel i proprietile sale, prin unele
aplicaii, formulate sub form de probleme de concurs (olimpiade locale, judeene, concursuri
colare interjudeene, etc.).
2n

Exemplul 1
Fie (A,+,) inel cu 1A 0 A , cu proprietatea c exist A unic astfel ca 2 + 1A = 0 A (*)
S se arate c:
a) char A=3
b) x 2 p = 0 A x p = 0 A , p * , x A (OLM Vaslui 2000)
Soluia:
2
a) Avem (1A ) (1A ) + 1A = 1A + 2 1A + + 1A = 2 + 1A = 0 i din unicitatea
(*) = 1A + = 1A .
Dar cum 1A = 2 1A = (1A ) si 1A = (1A ) este element inversabil al inelului cu
1 = 1 A .
(*)

n fine din + = 1 A 1 1 A + 1 A = 1 = 1 A , adica = 1 A 1 A + 1 A + 1 A = 0 A c.c.t.d.


b) Presupunem x 2 p = 0 , x A, p *

Avem 1A x p

) ( 1 x ) + 1 = 1 + x + x
+ 1 ) + (x ) + (x + x + x ) = 0
2

+ x 2 p + 1A + x p + 1A =
pentru c 1K + 1K + 1K = 0 K .

2p
p
p
p
= (1A + 1A
A
Din unicitatea (*), avem c 1 A x p = = 1A x p = 0 A
Reciproc este evident.

Exemplul 2
Fie (A,+,) inel cu proprietatea c x 4 = x 2 , x A . S se arate c 1 A + 1A + ... + 1 A + 1 A =0.
14442444
3
12 ori

( x + 1A )

(OLM Galai 2005)

= ( x + 1 A ) , x A , deci x + 4 x + 6 x + 4 x + 1A = x + 2 x + 1 A i deci
4 x + 6 x + 2 x = 0 A , x A ; pentru x= 1A 41A +6 1A +2 1A =0, adic 1 A + 1A + ... + 1 A + 1 A = 0 A .
14442444
3

Soluie: Avem

12 ori

(amintim c 4 x 3 = x3 + x 3 + x 3 + x 3 ; 6 x 2 = x 2 + x 2 + x 2 + x 2 + x 2 + x 2 ;2 x = x + x etc)
Exemplul 3
Fie (A,+,) un inel n care 1A + 1A este inversabil. Demonstrai c cel puin una dintre urmtoarele
afirmaii este adevrat:
a) char A= n *
b) A conine o infinitate de elemente inversabile
(OLM Arge 2000)

Soluie: Presupunem c a) este fals i artm c b) este adevrat, artnd inductiv (dup n * )
c 2 n 1A este element inversabil.
Pentru n=1 avem 1 A + 1 A = 21 1 A este inversabil ()u A a.. ( 1 A + 1 A )u= 1 A , adic u+u= 1 A (*)

- 47 -

CERCUL DE MATEMATICA
Presupunem c pentru k * , elementul 2 k 1A este inversabil i artm c elementul 2 k +1 1A este
inversabil:
Fie x A astfel c ( 2 k 1A )x=x( 2k 1A )= 1A . Atunci ( 2k x)= (x 2 k )=1 A (**)
i deci

2k xu=u; adun u

(*)

2 k xu+u=u+u = 1A

( 2 k -1)xu+xu+u= 1 A

( 2k -

(**)

1)(xu)+(x+ 1A )u= 1A ( 2k -1)xu+(x+ 2 k x)u= 1A 2 k +1 xu= 1A ( 2 k +1 1A )(xu)= 1A .


Analog se arat c (xu) ( 2 k +1 u)= 1A , deci 2 k +1 1A este element inversabil al lui A.
Deci am artat c 2 n 1A este element inversabil al lui A, n * i cum a) este fals
2 m 1A 2 n 1A , m, n * , mn, deci mulimea U={2 n 1A | n * } are toate
elementele distincte dou cte dou i conine o infinitate de elemente, toate inversabile.
Exemplu 4
Dac (A,+,) inel de caracterisric zero i x,y,z
x 2 = x, y 2 = y, z 2 = z , x + y + z = 0 A , s se arat c x=y=z= 0 A .

A,

cu

proprietatea

(IMC 2000)

Soluie
Avem

2 x + 2 y + xy + yx = 0 A ;
2 x + 3 xy + xyx = 0 A (1).
x+y=-z

(x + y )2 = z 2 = z = x y, deci x 2 + xy + yx + y 2 = x y
nmulesc

cu

la

stnga:

2 x + 2 xy + x y + xyx = 0 A
2

astfel

nmulind cu x la dreapta 2 x 2 + 3 xyx + xyx 2 = 0 A , adic 2 x + 4 xyx = 0 A (2).


Din (1) i (2) xy=xyx i folosind (2) 2x+4xy= 0 A .
char A=0 x+2xy= 0 A 2xy=-x
4xyxy= x 2 =x;

Atunci

dar 4xyxy=4xyy=4xy=-2x,

deci x=-2x

x+x+x= 0 A

CharA= 0

( 1A + 1A + 1A )x= 0 A x= 0 A .
Analog avem i c y=z= 0 A . Se observ c este suficient ca char A {2,3}.
Exemplu 5
Fie K un corp de caracteristic p, cu p 1(mod 4)
a) Demonstrai c - 1K este ptratul unui element din K
b) Artai c orice element nenul din K se scrie ca suma a trei ptrate nenule de elemente din K.
c) Rmne, n general, aceast scriere adevrat pentru elementul nul din K ?
Soluie
a) Fie m * , p=4m+1
Din teorema Wilson, p|(p-1)!+1, deci (p-1)!+ 1K = 0 K n K, deci (4m)!=- 1K n K.
Apoi 1K =-4m, 2=-(4m- 1K ), ..., 2m=-(2m+ 1K ) n K nmulite:
1A 23...(2m)=(4m)(4m-1K )...(2m+ 1K ) i nmulim cu membrul drept
(4m)!=[(4m)(4m- 1K )(2m+ 1K )] 2 , adic - 1K =[(4m)(4m-1K )(2m+ 1K )] 2 ,
deci -1K este ptrat perfect n K
b) Fie b=2 1 , deci ( 1K + 1K )b= 1K , b+b= 1K (*) i avem: b-b; atunci a i b comut cci din (*)
nmulind cu a la stnga, respectiv la dreapta avem c ab+ab=a=ba+ba
( 1K + 1K )ab=(1K + 1K )ba
ab=ba, a K * variabil i 2ab=a.
Dac a-b a+b 0 K i a=2ab=(a+b) 2 -a 2 -b 2 =(a+b) 2 +a 1 2 +b 1 2 , cci- 1K este ptrat perfect n K.
Dac a=-b ab i a=2ab+b 2 -(a-b) 2 =a 2 +b 2 +c 2 , cci - 1K este ptrat perfect n K.
c) n Z , 0 nu se poate scrie ca suma a trei ptrate perfecte nenule.
5

- 48 -

CERCUL DE MATEMATICA

SSM
H

n ncheiere, expunem cititorului un set de probleme propuse (PP) legat de tema acestui articol.
PP 1) Fie (A,+,) inel cu 6 elemente i A={n N * 1 A + 1 A + ... + 1 A = 0 A }
1442443
de n ori

a)

S se arate c min n = 6

b)

A este un inel comutativ cu divizori ai lui zero.

n A

(OLM Prahova 2003)


PP 2) Spunem c inelul A are proprietatea P dac pentru orice x A , exist un nx impar, cu
proprietatea x nx {0 A ,1A }
a) S se arate c un inel cu proprietatea P este corp.
b) S se arate c orice corp finit n care 1A + 1A = 0 A are proprietatea P.
c) S se dea exemplu de corp infinit n care 1A + 1A = 0 A i care nu are proprietatea P.

(Concurs Gh. Lazr, 2003)

PP 3) Fie (A,+,) inel, a A, n, k , n 2, k 2 astfel ca 1 A + 1 A + ... + 1 A =0, a k = a + 1A .


1442443
de n ori

S se arate c:
a) oricare ar fi S N * , exist P0 , P1 ,..., Pk +1 astfel c:

a S = P0 + P1a + ... + Pk 1a k 1
b) exist m N * astfel nct a m = 1 A
(OJM 2002)
Bibliografie
[1] M.ena Manual pentru clasa a XII-a, Editura Gil Zalu 2003
[2] C.Nstsescu, C.Nit .a. Manual pentru clasa a XII-a, E.D.P. Bucureti 2002
[3] M.Antronache, M.ena .a. Olimpiadele de matematic 2003 (cls. XI-XII) Editura Gil 2003
[4] A.Izmail, G.Kreindler .a. Olimpiadele de matematic 2004 (cls. XI-XII) Editura Gil 2004.
[5] D.Brnzei, C.Stoica .a. Matematic n concursurile colare, Ed. Paralela 45, Piteti 2000.

Tema pentru grupa de performanta la clasa a-XII-a


TEOREMELE LUI SYLOW
prof. Dana Paponiu CNT
Fie G un grup finit. Din teorema lui Lagrange, ordinul unui subgrup al lui G divide ordinul
grupului G. Reciproca teoremei lui Lagrange este falsa, adica daca n este un divizor al lui G nu
rezulta neaparat ca G are un subgrup de ordin n.
Teoremele lui Sylow permit sa se stabileasca existenta subgrupurilor de ordin n daca n este
o putere a unui numar prim si dau informatii importante despre aceste subgrupuri.
Fie p un numar natural prim. Daca G = p m r ,unde m si r sunt numere naturale nenule si

p nu divide r , vom numi p-subgrup Sylow al lui G orice subgrup de ordin p m al sau.
T1. Pentru orice grup finit G si orice numar prim p ce divide G exista un
p-subgrup Sylow al lui G.

- 49 -

CERCUL DE MATEMATICA
T2. Fie G un grup finit si p un numar prim ce divide G . Daca H este un p-subgrup Sylow
al lui G, iar J un p-subgrup al lui G, atunci exista g G astfel incat J g 1 Hg .
T3. Fie n p numarul p-subgrupurilor Sylow distincte ale unui grup finit G. Atunci
n p = G : N G ( H ) , unde H este un p-subgrup Sylow particular al lui G, n p divide G : H si
n p 1(mod p ) .

Aplicatii.
A1. Fie G un grup de 6009 elemente. Aratati ca G are un singur subgrup de ordin 2003.(V.
Vornicu, lista scurta O.N.M. 2003)
Solutie.Teorema 3 a lui Sylow ne spune ca numarul 2003-subgrupurilor lui G este congruent cu 1
(mod 2003), deci el nu poate fi decat 1.
A2. Fie p un numar prim, G un p-grup cu G = p m , m N. Sa se demonstreze ca pentru
fiecare i {0,1,2,..., m} exista un subgrup normal Gi al lui G astfel incat Gi = p i
si 1 = G0 < G1 < ... < G m = G .
A3. Fie p un numar prim, iar G un grup finit astfel incat p m G , m N. Sa se demonstreze
ca G are un subgrup de ordin p m .
Solutie. Fie H un p-subgrup Sylow al lui G cu H = p t ; atunci m t si exista (conform A2) J H
astfel incat J = p m .
Ca urmare a acestui exercitiu putem trage concluzia ca pentru p-grupurile finite este adevarata
reciproca teoremei lui Lagrange.
A4. Fie G un grup finit, p un divisor al lui G , iar H un p-subgrup Sylow al lui G.
Sa se demonstreze ca daca H = p , atunci numarul elementelor de ordin p din G este egal cu
n p ( p 1) .

A5. Aratati ca orice grup de ordin 255 este ciclic.


Solutie. Fie G astfel incat G = pq, P un p-subgrup Sylow al lui G si Q un q-subgrup Sylow al lui
G. Deoarece P = p si Q = q , deducem ca P si Q sunt ciclice, adica P =< x >, Q =< y > cu
ord(x)=p si ord(y)=q. Conform teoremei lui Lagrange, deducem ca P Q = 1 . Avem n p q si
n p 1(mod p ) . Deoarece q 1(mod p ) rezulta ca n p = 1 si P G ; analog Q G . Din P G

obtinem ca xyx 1 y 1 = x( yx 1 y 1 ) P si din Q G obtinem ca ( xyx 1 ) y 1 Q , deci


xyx 1 y 1 P Q . Rezulta succesiv ca xy = yx ,
ord ( xy) = pq , G =< xy > , G este grup ciclic.
Bibliografie:
[1] Gh.Vraciu Elemente de teoria grupurilor finite
[2] D.Busneag Probleme de algebra

- 50 -

CERCUL DE MATEMATICA
Tema pentru grupa de performanta la clasa a-XII-a

SSM
H

Asupra polinomului caracteristic


prof. Daniel Sitaru,
CN Economic Th. Costescu,
Fie V o mulime nevid, ale crei elemente le vom nota cu a, b, c . i (K,+, *) un corp
comutativ cu elementele notate , , , .. (exceptnd elementul nul i elementul neutru pe care
le vom nota cu 0, respectiv 1). Presupunem c pe V este definit relaia de egalitate a elementelor
sale.
Definiia 1 Spunem c mulimea V are structur de spaiu vectorial (liniar) peste corpul K
dac este nzestrat cu dou legi de compoziie:
I ) O lege de compoziie intern, + : V x V V, numit adunare, n raport cu care V este grup
II ) O lege de compoziie extern * : K x V V, numit nmulire cu scalari, care verific
axiomele:
1. ( ) a = ( a )
2. ( a + b ) = a + b
3. ( + ) a = a + a
4. 1* a = a , oricare ar fi a , b V i , K.
Elementele unui spaiu vectorial se numesc vectori; elementele corpului K se numesc scalari
Elementul neutru al grupului (V, +) se numete vectorul nul (i l notm cu 0 ) al spaiului
vectorial V.
Fie V,W dou spaii vectoriale peste acelai corp K.
Definia 2 Aplicaia f: V W se numete aplicaie liniar (homomorfism liniar, operator
liniar) dac:
a) f este aditiv, adic f( x + y ) = f ( x ) + f ( y ), oricare ar fi x , y V.
b) f este omogen, adic f ( x ) = f( x ), oricare ar fi K ; x V.
Dac W = V spunem c f este un endomorfism liniar sau operator liniar al spaiului vectorial
V. Notm End(V) mulimea endomorfismelor spaiului vectorial V.
Definiia 3 Vectorul nenul x V se numete vector propriu al operatorului liniar f dac
exist un scalar K astfel nct f ( x ) = x . Un scalar K se numete valoare proprie a
operatorului liniar f dac exist un vector x V \ { 0 } pentru care f ( x ) = x .
Proprietatea 1
a) Dac x V\{ 0 } este vector propriu al lui f, atunci exist un unic scalar K pentru care
f( x )= x . Cu alte cuvinte, oricrui vector propriu i corespunde o singur valoare proprie
b) Oricrei valori proprii i corespund o infinitate de vectori proprii.
Definiia 4 Fie B1 = { a1 , a 2 , ., a n } i B2 = { b1 , b2 , , bn } baze n spaiile vectoriale
V, respectiv W i f: V W o aplicaie liniar. Scriind vectorii f ( a1 ); f ( a 2 );; f ( a n ) n raport
cu baza B2 avem:
f ( a i ) = ai1 b1 + ai2 b2 + + aim bm ; i = 1,2, , n.
Matricea A ale crei coloane sunt coordonatele imaginilor prin f ale vectorilor bazei B1 n
raport cu baza B2 se numete matricea aplicaiei liniare f n raport cu bazele B1 i B2.
Vom avea:

- 51 -

CERCUL DE MATEMATICA
a11
2
a1
A=
m
a1

a 12
a 22
a 2m

a 1n

a n2

a nm

Definiia 5 Fie B = { a1 , a 2 , ., a n } o baz a spaiului vectorial V i f End (V). Atunci


oricare ar fi i = 1, 2, , n avem f ( ai ) = aik a k .
Prin matricea operatorului liniar f n raport cu baza B vom nelege matricea:
a11 a 12
a 1n
2

2
a n2
a1 a 2
A=

n
a nn
a1 a 2
Proprietatea 2 Fie A Mn(k) matricea operatorului liniar f End (V) n raport cu baza
B = { a1 , a 2 , ., a n }a lui V. Au loc urmtoarele afirmaii:
a) Un scalar K este valoarea proprie a lui f dac i numai dac det ( A In ) = 0 .
b) Un vector x 0 = x0i a i V este vector propriu al lui f, corespunztor valorii proprii ,
dac i numai dac vectorul coloan ~
x = ( x01, x02, , x0n)t format cu coordonatele sale,
0

este o soluie nenul a sistemului liniar i omogen :


~
(A - In) ~
x = 0
Proprietatea 3 Fie f End (V) i A, B matricile lui f n raport cu bazele B = { a1 , a 2 , .,
a n }, respective B1 = { b1 , b2 , , bn } ale lui V. Atunci det (A - In) = det (B - In).
Definiia 6 Polinomul Pf () = det (A - In) se numete polinom caracteristic al operatorului
liniar f. Ecuaia Pf () = 0 se numete ecuaia caracteristic a lui f.
Proprietatea 4 Un scalar K este valoarea proprie a lui f dac i numai dac este o
rdcin (n K) a ecuaiei caracteristice Pf () = 0.
Practic, pentru aflarea valorilor proprii i a vectorilor proprii pentru un operator liniar
f End (V), cu V finit dimensional, se procedeaz astfel:
1) Se fixeaz o baz B = { a1 , a 2 , ., a n } n V
2) Se scrie matricea lui f n raport cu baza B.
3) Se calculeaz polinomul caracteristic Pf () = det (A - In)
4) Se rezolv n K ecuaia caracteristic Pf () = 0, adic se determin valorile proprii 1,
2, , m; m n (egalitate pentru K = C).
5) Pentru fiecare valoare proprie i, i = 1, 2, , m se determin vectorii proprii, adic
~
vectorii nenuli xi prin rezolvarea sistemului liniar i omogen (A i In) ~
x = 0

Aplicaia 1
Fie f End (R2) care n raport cu baza canonic e1 = (1,0); e2 = (0,1) a lui R2 este definit
prin: f ( x ) = ( x1 2 x2, 2 x1 4 x2), pentru x = (x1,x2) R2. Se cer valoriile proprii i vectorii
proprii pentru f.
Soluie: f ( e1 ) = (1,2); f ( e2 ) = (-2,-4). Matricea lui f n raport cu baza { e1 , e2 }este:
1 2

A =
2 4

- 52 -

CERCUL DE MATEMATICA

SSM
H

Polinomul caracteristic al lui f este :


2
1
Pf () = det
= 2 + 3
2

Ecuaia caracteristic este 2 + 3 = 0. Valorile proprii sunt soluiile acestei ecuaii: 1 = -3;
2 = 0.

Determinarea unui vector propriu pentru 1 = -3;


4 2 x1 0

2 = , adic: 2x1 x2 = 0.
2 1 x 0

Alegem x1 = necunoscut secundar i rezult x2 = 2; R. Un vector propriu este


x 1 =(1,2). Orice alt vector propriu este de forma x = x 1 ; R*. Analog, pentru 2 = 0 avem
x 2 =(2,1). Orice alt vector propriu este de forma x = x 2 ; R*.
Aplicaia 2
Fie f End (R3) care n raport cu baza canonic din R3 are matricea:
1 0 0

A = 0 1 1 . Se cer valorile i vectorii proprii.


0 1 1

Soluie: Singura rdcin real a polinomului caractristic:


1
0
0
Pf () = 0
1
1 =( 1 )[(1 )2 +1] este = 1. Rezult c f are valoarea proprie =1.
0

1
0 0 0 x 0

2
Formm sistemul 0 0 1 x = 0 .
0 1 0 x3 0


1
Se observ c x este necunoscut secundar, iar x2 = x3 = 0. Deci, un vector propriu ataat
valorii proprii = 1 este a = (1,0,0), ceilali vectori proprii fiind de forma x = a ; R*.

Aplicaia 3
Fie V un spaiu vectorial peste R i f End (V) care n raport cu baza B = { a1 , a 2 } a lui V
2 5

are matricea A =
4 2
ntruct polinomul caracteristic Pf () =

2
4

5
= (2- )2 +20 nu are rdcini n R
2

rezult c f nu are valori proprii i vectori proprii.


Aplicaia 4
Fie V un spaiu vectorial peste R i f End (V) care n raport cu baza B = { a1 , a 2 , a 3 , a 4 }
2 1
1 0

4 2
0 1
are matricea A =
.
2 1 0
1

2 1 1 2

Se cer valorile i vectorii proprii.

- 53 -

CERCUL DE MATEMATICA
1
0
0
1

2
4

1
2

= (-1)4
2
1
1
2
1 1 2
Rezult c f admite valoarea proprie = 1, multipl de ordinul 4. Sistemul care d
coordonatele x1, x2, x3, x4 ale vectorilor proprii ( n raport cu baza B) ataai valorii proprii = 1
este:

2x3 x 4 = 0
(S) 1
. Alegnd x1, x2 ca necunoscute secundare i x3, x4 principale,
2
3
4
2 x x x + x = 0
obimem:

x1

x2 = x2

cu x1,x2 R.
3
1
2
x = 2 x + x
x 4 = 4 x 1 + 2 x 2
Soluie: Polinomul caracteristic al lui f este : Pf () =

Prin urmare, un sistem fundamental de soluii ale sistemului (S) este: a1 = (1,0,-2,-4);

a 2 =(0,1,1,2). Vectorii a1 , a 2 sunt vectori proprii ai lui f, ceilali vectori proprii fiind de forma
a = a1 + a 2 , unde , R nct a 0 .

Teorema Cayley Hamilton


Fie Pf () polinomul caracteristic al lui f End (V). Dac n raport cu o baz oarecare f are
matrice A Mn (C) atunci Pf (An) = 0 n
Aplicaia 5:
a b
matricea unei aplicaii f End (V).
Fie A =
c d
a
b
Pf () =
= (a- )(d- )-bc = ad - a - d + 2 bc = ad bc - (a+d) + 2
c
d
Pf (A) = 0 2 A2 (a+d) A + (ad-bc) I2 = 02 A2 TrA A + detA I2 = 02
Aplicaia 6
a b c

Fie A = d e f matricea unei aplicaii f End (V).


g h i

Pf () = (a ) (e- )(i- ) + bfg + cdh cg(e- ) hf(a- ) bd(i- ) = - 3 + 2(a+e+i)


(ae+ai+ei-cg-hf-bd) + aei + bfg + cdh cge ahf bdi.
a b
Pf () = - 3 + 2 TrA (det A1 + det A2 + det A3) + det A unde det A1 =
; det A2=
d e
e
h

f
a c
; det A3 =
;
i
g i
Pf (A) = 0 3, de unde:
A3 Tr A A2 + (det A1 + det A2 + det A3) A det A I3 = 03

Bibliografie:
[1] I.Vladimirescu Note de curs Reprografia Univ. din Craiova 1987
[2] I.R. afavevici Noiunile fundamentale ale algebrei Ed. Academiei 1989

- 54 -

CERCUL DE MATEMATICA

SSM
H

mATh Studio

...sau despre limitrile matematicii fcute pe hrtie


Boce Bogdan elev clasa a XII-a CNT
A folosi un calculator n rezolvarea unor probleme de matematic este de multe ori
considerat un demers nenecesar, care poate mpiedica o nvare corect a materiei. Cnd
problemele ce apar n domeniile mai avansate ale matematicii se complic, folosirea unei maini de
calcul devine inevitabil. Exist multe cazuri, mai ales n simularea unor procese reale, cnd o
soluie analitic, exact nu mai poate fi obinut; drept urmare, e nevoie de o soluie aproximativ,
numeric.
Acest proiect al meu, mATh Studio, este un set
de instrumente ce permite rezolvarea numeric a unor
astfel de probleme care nu admit soluii exacte. Printre
acestea se pot enumera integrri multiple i derivri
pariale, convoluii, estimri de serii, rezolvri numerice
de ecuaii difereniale i de ecuaii difereniale cu
derivate pariale, transformri Fourier rapide, analiza
(statistic, diferenial) a seturilor de date.
Dat fiind faptul c majoritatea conceptelor aici
prezentate depesc programa de liceu, m voi limita la
a arta aplicaiile lor practice.

Figura 1

1. Propagarea oscilaiilor unui fir semi-elastic,


considernd forma iniial dat, pierderile de energie
i oscilaii definite la capete (figura 1).
2. Propagarea undelor unei membrane semi-elastice,
considernd forma iniial, pierderile de energie i
oscilaii suplimentare definite la margini.
3. Studiul evoluiei sezoniere a populaiilor dintr-un
ecosistem pe baza relaiilor prad-prdtor, folosind
ecuaiile Lotka-Volterra (figura 2).

Figura 2

4. Studiul gradului de fericire al individului ntr-o serie


de circumstane definite, prin msura n care sunt
favorabile sau nefavorabile. (figura 3)
5. Studiul unor probleme din cinematic, cum ar fi
aruncarea pe oblic la viteze foarte mari,
considernd rezistena la naintare proporional cu
ptratul sau cu cubul vitezei.
Figura 3

- 55 -

CERCUL DE MATEMATICA
De asemenea, mATh Studio pune la
dispoziia utilizatorului numeroase module de
vizualizare pentru diferite tipuri de seturi de
data: curbe explicite i parametrice (n plan sau
n spaiu), suprafee explicite sau parametrice
(figura 4) i diagrame de variaie. Toate
modulele de vizualizare pot fi folosite att
pentru seturi de date statice sau animate (care
conin un parametru suplimentar, pentru timp).
O facilitate important a programului
este c permite relaionarea seturilor de date.
Mai exact, datele de ieire obinute folosind
unul dintre modulele de calcul pot fi folosite ca
date de intrare pentru alt modul. Astfel se pot
defini ntregi scheme de calcul, care pot fi
salvate, editate i folosite ulterior.

Figura 4

n concluzie, mATh Studio, e un utilitar ce permite experimentarea cu diverse concepte


matematice ce nu pot fi dezvoltate n absena unui sistem de calcul. Mai mult, programul permite
aplicarea acestor concepte n diverse domenii, de la fizic i astronomie, pn la sociologie i
epidemiologie.
Din partea redactiei
Lucrarea complet a fost prezentat n cadrul CONCURSULUI
INFOEDUCATIE - Glciuc 2006 i a obinut Premiul I.

- 56 -

NATIONAL

PROBLEME PROPUSE
Elevii vor rezolva probleme de la clasa pe care o urmeaz i de la clasa inferioar.
Se pot rezolva i problemele date la Ediia a II-a a concursului Petre Sergescu pagina 9. Soluiile redactate pe foi format A5 se vor preda profesorului ndrumtor.

IV.1.
a)
b)
c)

Aflai numerele naturale a, b, c care ndeplinesc simultan condiiile:


al doilea numr este de trei ori mai mare dect primul,
al treilea numr este jumtatea celui de-al doilea,
suma numerelor este mai mare dect 7 i cel mult egal cu 44.

inst. Maria Ungureanu

IV.2. Vlad are un numr de bile. El observ c le poate grupa cte 6 dar i cte 10,
iar diferena dintre grmezi este 2. Cte bile are Vlad?

inst. Maria Ungureanu

IV.3. Dou veverie consum rezerva de alune n 6 luni. n ct timp vor termina
rezerva trei veverie?

inst. Maria Ungureanu

IV.4. Radu are de 7 ori mai multe timbre dect colegul su. Dac ar fi avut cu 48 de
timbre mai puine, iar colegul su cu 36 de timbre mai multe, atunci Radu ar fi
avut de trei ori mai multe timbre dect colegul su. Cte timbre are fiecare?

inst. Maria Ungureanu

IV.5. La concursul Cangurul pentru fiecare din cele 20 de probleme propuse,


rezolvat corect, se acord 10 puncte, iar pentru fiecare problem nerezolvat se
scad 5 puncte. Ana a obinut 125 puncte. Cte probleme a rezolvat corect ?

inst. Niculina Opria

IV.6. Aflai numerele naturale a, b, c care ndeplinesc simultan condiiile:


a) a + c = 20,
b) a b = 5,
c) b c = 5.

inst. Niculina Opria

IV.7. Diferena a dou numere este 226 iar primul mrit cu 2 este de 8 ori mai mic
dect al doilea. Care sunt numerele?

inst. Niculina Opria

IV.8. Rsturnatul semisumei a dou numere este 24, iar a patra parte din primul
este egal cu a treia parte din al doilea. Care sunt numerele?

inst. Mariana Cornea

IV.9. Tatl are 46 de ani iar fiul are 19 ani. Cu ci ani n urm tatl era de patru ori
mai n vrst dect fiul ?

inst. Mariana Cornea

IV.10.Produsul vrstelor a trei frai este 72, iar suma vrstelor este mai mic dect
15. Ci ani are fiecare, tiind c nu sumt gemeni ?

inst. Mariana Cornea

V.1. S se afle numerele naturale prime n pentru care numerele n+1, n+3, n+5,
n+21 sunt simultan prime.

prof Calafeteanu Gheorghe

V.2. Aflai cel mai mic nr. natural care are exact 20 divizori.

prof. Calafeteanu Gheorghe

V.3. Aflati a,b,c,d,e N care verific a 2 + b 2 + c 2 + d 2 + e 2 =2007


V.4. Artai ca nu exist n N astfel nct n -3 sa fie ptrat perfect

prof. Victor Sceanu

- 56 -

prof. Vctor Sceanu

PROBLEME PROPUSE
a +b

V.5. Determinai a,b,c N tiind ca verific relaia: 3

+4

b +c

+6

c+ a

+6

SSM
H

prof.Victor Sceanu

V.6. Aratati ca daca m este minimul dintre numerele 31338 si 24446, atunci m > 22007.

prof. Alexandru Szoros

V.7. Aflati numerele naturale n ce verifica egalitatea 2n + n5 = n2 + 5n .

prof. Alexandru Szoros

V.8. S se afle restul mpririi numrului n=20052006 + 2007 2006 +2004 la 2006

prof. Marica tefan


propus OJ

V.9. mprind n numere naturale consecutive la n obinem resturi a cror sum


este 55.
a) S se afle n
b) S se afle cele n numere naturale tiind c suma dintre cel mai
mic si cel mai mare este 230

prof. Marica tefan


propus OJ

V.10. Pe un raft al unei biblioteci sunt asezate in ordine 10 carti, astfel ca numarul
de pagini pentru doua carti vecine(alaturate) sa difere cu 1. Pot fi in total, pe raft
2006 pagini? Dar 2005?

prof. Dan Nedeianu


propus OJ

V.11. Se consider mulimea M={n / n 2006}


a) Stabilii dac se poate scrie M ca reuniune de submulimi disjuncte Mk astfe
nct n fiecare mulime Mk s existe un element egal cu suma celorlalte.
b) S se demonstreze c din oricare 1005 elemente din M se pot alege dou
elemente diferite astfel nct suma lor s fie 2006.

prof. Dana Paponiu


propus OJ

2n 2 + 7
2
VI.1. Artai c fracia : 5n + 17 este ireductibil.

prof. Calafeteanu Gheorghe

VI.2. S se arate c oricare ar fi 5 numere biptrate perfecte (adic egale cu puterea


a 4 a unor numere ntregi) exist 2 a cror diferen se divide cu 10.

prof. Calafeteanu Gheorghe

VI.3. Aflai msurile unghiurilor unui triunghi ABC tiind ca :m( A)=[m( C)] si
m( B)=2m( C)

prof. Victor Sceanu

VI.4. Gsii o partiionare a unei suprafee triunghiulare dreptunghice, in 5


suprafee triunghiului isoscel nedistincte.

prof. Victor Sceanu

VI.5. Fie triunghiul ABC cu m( A)=90 , AB=1 , AC= 3 , D-mijlocul lui [BC] si
F (AC)
Artai ca : BF+FD=minim, daca AC=3 AF

prof. Victor Sceanu

VI.6. Fie multimile A = {6n+1| nN* } si B = {6n-1| nN* }. Aratati ca :


1) A B = ;
2)
x,yA xyA ;
3)
x,yB xyA ;
4) Cel putin una din multimile A, B contine o infinitate de numere prime.

prof. Alexandru Szoros

- 57 -

PROBLEME PROPUSE
VI.7. Rezolvati in N ecuaia

VII.1. Rezolvai ecuaia

2
3
2007
+ 3
+ ... + 2007
= 2006 .
n +1 n + 2
n
+ 2006
2

( x 2006)( x 2005)( x 1)
0
x 2 3x + 2

prof. Alexandru Szoros

prof. Calafeteanu Gheorghe

VII.2. In dreptunghiul ABCD cu BC > BC, fie E (AB), F (BC), CE DF={P}, iar MN
mediatoarea lui [EC], N (EF). Daca NC este tangenta cercului circumscris CDP,
artai ca DE EF.

prof. Victor Sceanu

VII.3. Rezolvai in Z X Z ecuaia: 3x y + xy = 2010


VII.4. Aflai x Ndin egalitatea: 3

3 x+1

2 3

2x

prof. Victor Sceanu

2 3 =2007
x

prof. Victor Sceanu

VII.5. Demonstrati ca exista o infinitate de numere naturale a cu proprietatea ca


multimea M = {7a+3, 7a+4, 7a+5, 7a+6 } contine exact un patrat perfect.

prof. Alexandru Szoros

VII.6. Determinati numerele intregi x, y, z ce verifica simultan inegalitatile :


x2 +3y +25 < 11z; y2 +5z 11< 9x;
z2 +7z 3<7y.

prof. Alexandru Szoros

VII.7. Demonstrati ca pentru

aZ expresia a6 +4a4 5a2 este divizibila cu 36.

prof. Alexandru Szoros

VII.8. Sa se afle cifrele x,y,z care satisfac egalitatea


0,x(y1)+0,2y(3z)+0,4z5(yx)=1,430(32).

prof. Eleodor Popescu


propus OJ

VII.9. a) Fie ABCD un trapez cu AB| |CD, AB=b<a=CD i ACBD={0}. Dreapta d


trece prin O i intersecteaz [CD] i [AB] n E respectiv F. Notm CE=x. S se
determine x pentru care aria [FADE]>aria[ECFB].
b) Paralelogramul MNPQ este decupat din interiorul paralelogramului ABCD. S
se determine o dreapt care mparte figura rmas n doua figuri de aceeasi arie.
Justificai rspunsul.

prof. dr. Gh. Ciniceanu


propus OJ

VII.10.
Fie a , b, c Z/ si numerele A = 7a-b+c , B = 5b+a+c.
Demonstrati ca A M 6 B M 6.

VIII.1.

Descompunei n factori 2a2-a-3.

VIII.2.

Calculai maximul expresiei: E(x) = 2005 9x2 + 6x.

(3x + 2)(3x + 3) 2
VIII.3.
Se d fracia F ( x) =
3x(3x + 2) + 1
a) Simplificai fracia
b) Aflai pentru ce valori ntregi ale lui x fracia
c) Calculai F(0,(3))

- 58 -

prof. Dan Nedeianu,


propus OJ

prof. Calafeteanu Gheorghe


prof. Calafeteanu Gheorghe

3x + 4

3x + 1
prof. Calafeteanu Gheorghe

PROBLEME PROPUSE
VIII.4.

Fie a, b > 0 cu

SSM
H

1 1
1
1
1 1
+ Artai ca : 3 + 3 3 + 3 >
a b
a
ab b 4

prof. Victor Sceanu

VIII.5.
In triunghiul ascuitunghic ABC, fie (AA` bisectoarea unghiului A ,
A` (BC). Demonstrai ca I este centrul cercului nscris in triunghiul ABC daca si
DA` p
numai daca
=
EA` a

prof. Victor Sceanu

VIII.6.
Fie triunghiul dreptunghic isoscel ABC cu m( A)=90 , D (AB) , E (AC),
DK BE, AL BE, (K, L (AC) ) si CL = LK. Se duce MB (ABC), MB=BC. Aflai
m( (MC,DC))
0

prof. Victor Sceanu

VIII.7.
In triunghiul ABC, AM mediana, G (AM), GE AB, GF AC, E,
F (BC). Sa se demonstreze ca G este centrul de greutate al triunghiului ABC dac
1
i numai dac aria[GEF]= [ABC]
9
VIII.8.

Rezolvati in R ecuatia

prof. Victor Sceanu

(x2 +x +1)3 + (x2 -x +1)3 = 3x4 +3x2 +2.

prof. Alexandru Szoros

VIII.9.
In trapezul ABCD oarecare cu baza mare CD se dau [AB] =[BC]=a,
m(BCD)=60, M mijlocul lui [CD], BMAD={N}, D[NA]. Pe planul trapezului
(ABCD) se ridica perpendiculara [VA]=a. Sa se calculeze distanta de la punctul V
la dreapta CN.

Prof.Dan Nedeianu
propus OJ

VIII.10.
Aratati,ca daca numarul natural x se scrie ca o suma de trei patrate
perfecte,atunci x n se scrie si el ca o suma de trei patrate perfecte,oricare ar fi n
N.

prof. Victor Saceanu


propus OJ

IX.1. Demonstrati ca in orice triunghi ABC are loc inegalitatea


2

ra rb rc
R
+ + 4 + 1 .
r
r
r
r
b
c
a

IX.2. Sa se arate ca daca x1 ,x2,.,xn (0, ) atunci


2

sin(x1+x2)sin(x2+x3)sin(xn-1+xn)sin(xn+x1)
sin2x1sin2x2 sin2xn
IX.3. Dac x, y, z (1,), s se demonstreze inegalitile:
x2 x +1 y2 y +1 z2 z +1
+
+
9
a)
x 1
y 1
z 1
x
y
z
b)
+
+
6
x 1
y 1
z 1

prof. Alexandru Szoros

prof. Daniel Sitaru


propus OJ

prof. Bloi Valeria propus OJ

- 59 -

PROBLEME PROPUSE
n

IX.4. Se considera functiile f, g:N*N* definite prin:f(n)= (k ) si g(n)=[ n ] unde


k =1

(k) reprezinta numarul divizorilor lui k. Sa se arate ca (-1) f (n ) =(-1) g (n )

prof. Manuela Praja


propus OJ

IX.5. Fie a, b N * Artai c ecuaia ( x a ) 2 + ( x b 2 ) = 2ab 1 nu are rdcini


raionale.
***

Clasa a-X-a
X.1. Aflati a R stiind ca sistemul 2007x +2007-x = y - |x| ;
admite o unica solutie reala.

x2 + 4 + y3 +1 = a
prof. Alexandru Szoros

X.2. Fie z1, z2, z3 numere complexe nenule, distincte cte dou i de module egale.
z z
z z
z z
2
2
2
4
4
4
Dac z1 + 2 3 , z 2 + 1 3 , z 3 + 1 2 sunt numere reale, atunci z1 = z 2 = z 3
z1
z2
z3
sau z1 z 2 z 3 = 1 .
prof. Manuela Prajea
propus OJ

X.3. Daca a, b, c C* sunt numere complexe distincte doua cate doua astfel incat
(a-b)=(b-c)=(c-a), sa se arate ca 2a b c = 2b a c = 2c a b .

prof. Dan Nedeianu


propus OJ

X.4. S se rezolve inecuaia: (3 2 ) x - ( 7 5 2 ) x 2


prof. Eleodor Popescu
propus OJ
n

k =1

k =1

X.5. Demonstrai inegalitatea log a xk log xk a n 2 unde a, x k > 1, k = 1, n, n N *


prof. Ovidiu Ticusi
propus OJ

XI.1. S se determine funciile f:R*+ R continue care satisfac relaia


a a
f ( x) f ( y) + 1 = f f + f ( xy ), ( ) x, y R+* unde a>0, a 1.
x y

prof. Manuela Prajea

XI.2. Fie f : (a,b) R o functie nemarginita. Sa se arate ca exista un sir ( xn )


convergent cu proprietatea ca (f(xn)) sa fie un sir cu limita .

prof. Gheorghe Ciniceanu

XI.3. Sa se arate ca exista o functie bijectiva f : (a,b) R pentru orice a, b numere


reale.

prof. Gheorghe Ciniceanu

XI.4. Se consider irul (x n ) n 1 definit prin x1= 1 si xn+1=xn+xn.


2
1

1
1
S se calculeze
+
+ ... +
.
x
+
1
x
+
1
x
+
1
1
2
n

prof. Manuela Prajea

- 60 -

PROBLEME PROPUSE
XI.5. S se arate c

SSM
H

b
c
a

+
+
| a, b, c > 0 =[ 3 ,2)

b + c a + c a + b
2

prof. Manuela Prajea


n

XI.6. S se calculeze cos2x tiind c lim sin 2n x = 5 .


n

k =0

prof. Manuela Prajea

XII.1. Se consider polinoamele f=8x-6x-1, g=4x-4x i mulimea

M= h(cos ) | h Q[ X ] .
9

a) S se arate c f(cos )=0.


9

b) S se arate ca cos Q.
9

c) Dac h Q[X] i h(cos )=0, s se arate c h este divizibil cu f.


9
d) S se arate c M este spaiu vectorial real n raport cu operaiile uzuale.
prof. Manuela Prajea

XII.2. Se consider irul

(I n )n0

definit prin I n =

4
0

tg2 xdx

1
, ( ) n N*.
2n 1
b) S se arate c sirul (I n )n0 este convergent i s se calculeze lim I n .

a) S se arate c In + In-1 =

1 1
1
(1 + ... + ( 1) n1
).
c) S se calculeze lim
n
3 5
2n 1

prof. Manuela Prajea

cos 2006 x
XII.3. Sa se calculeze:
dx, x (0, ).
sin x

prof. Dana Paponiu

XII.4. Fie f :: R R funcie care admite primitive, i F o primitiv a sa pentru care


sin 2 x
avem relaia F ( x ) f ( x) +
0, ( ) x R . S se arate c nu exist lim F ( x) .
x
1 + cos 2 x

prof. Dana Paponiu

XII.5. Fie A = f : R R f ( x ) 2 f ( x) 4 f ( x) + 1 = 0, x R . Stabiliti:


3

a) cate elemente are multimea A;


b) cate functii din A admit primitive.

prof. Dana Paponiu

- 61 -

Rezultate obtinute de elevii mehedinteni la concursuri de matematica in


anul 2006
OIM
Ungureanu Andrei Bogdan
Bazavan Eduard

medalie de aur
medalie de aur

prof. Prajea Manuela


prof. Stretcu Daniel

medalie de aur

prof. Stretcu Daniel

OBM
Bazavan Eduard

OLIMPIADA NATIONALA
Ungureanu Andrei Bogdan premiul I
prof Prajea Manuela
medalie de aur SSM
Bazavan Eduard
premiul al III-lea
prof. Stretcu Daniel
medalie de aur SSM
Nicolescu Alexandra
mentiune
prof. Gimoiu Iuliana
medalie de argint
Tesila Bianca
medalie de bronz SSM prof. Zaman Irina
Carapencea Constantin
medalie de bronz SSM prof. Paponiu Dana
Tigora Andrei
medalie de bronz SSM prof. Cainiceanu Gheorghe
Pasov Iulia
medalie de bronz SSM prof.Popescu Eleodor
CONCURSUL NATIONAL AL. MYLLER, Iasi
Croitoru Razvan
Carapencea Constantin
Pit-Rada Andrei

premiul al III-lea
mentiune
mentiune

prof. Paponiu Dana


prof. Paponiu Dana
prof. Paponiu Dana

CONCURSUL INTERJUDETEAN GH.TITEICA, Craiova


Pit-Rada Andrei
Tigora Andrei

mentiune
mentiune

prof.Paponiu Dana
prof.Cainiceanu Gheorghe

CONCURSUL INTERJUDETEAN ION CIOLAC , Craiova


Rapcea Mihai

premiul I

prof.Paponiu Dana

CONCURSUL NATIONAL SCOALA CU CEAS, Rm. Vilcea


Proba individuala
Sbarcea Alexandra
Gimoiu Ruxandra
Andreescu Madalina
Mitroi Roxana
Asproniu Robert

premiul al III-lea
premiul al III-lea
premiul al III-lea
mentiune
mentiune

prof. Prajea Manuela


prof. Gimoiu Iuliana
prof. Antonie Rodica
prof. Cainiceanu Gheorghe
prof. Cainiceanu Gheorghe

Nicoara Calin
Seitan Mihaela
Duta Adrian

mentiune
mentiune
mentiune

prof. Paponiu Dana


prof. Paponiu Dana
prof. Paponiu Dana

Proba La ceas
Voicu Razvan
Grosu Vlad
Papa Florin
Nicoara Calin

premiul al IIII-lea
premiul al II-lea
premiul al III-lea
premiul al II-lea

prof. Prajea Manuela


prof. Cainiceanu Gheorghe
prof. Paponiu Dana
prof. Paponiu Dana

BARAJUL NATIONAL CANGURUL (clasele VII-XII), Bucuresti


Bazavan Eduard
Cepesi Cristian
Carapencea Constantin
Sosu Cristian
Croitoru Razvan
Rosu Stefan

premiul I
premiul al II-lea
premiul al II-lea
premiul al II-lea
premiul al III-lea
premiul al III-lea

prof.Stretcu Daniel
prof.Pupaza Ecaterina
prof.Paponiu Dana
prof.Paponiu Dana
prof.Paponiu Dana
prof.Paponiu Dana

BARAJUL JUDETEAN CANGURUL (clasele V-VI)


Clasa a II-a
Popescu Maria
Sc.nr.3
Popescu Cristiana
Lic. I.St.Paulian
Vitus Eduard
Sc.nr 13
Popescu Simona
Sc. Nr 2
Clasa a III-a
Prencea Cassian
Lic. I.St.Paulian
Marghescu Gabriel
Nu a mentionat scoala
Dirpes Iulian
Sc. Nr 2
Marculescu Cristina
Sc. Nr 13
Clasa a IV-a
Dragomir Andrei
Lic. I.St.Paulian
Mema Serban
Lic. I.St.Paulian
Floarea Ionut
Strehaia
Mihutescu Diana
Strehaia
Stoichita Andreea
Strehaia
Ciuta Cora
Lic. I.St.Paulian
Clasa a V-a
Doana Cristina
Fara Mentiune de scoala
Rapcea Lucian
Vinju Mare
Baluta Adrian
C.N.Traian
Stanciu Alexandru
Gen.6
Arbanasi Emil
Gh.Titeica
Nuta Flavius
C.N.Traian
Nicola Silviu
C.N.Traian
Borcean Diana
Severinesti

Ciontea Stefan
Stefan Andrei

Gh.Titeica
C.N.Traian

Clasa a VI-a
Grosu Vlad
Clain Andrei Catalin
Rosca Ioana
Pirvulescu Eleodor
Ciocoteala Alexandru
Gevelica Simon
Tudosie Dorin

C.N.Traian
C.N.Traian
C.N.Traian
C.N.Traian
C.N.Traian
C.N.Traian

Elevi care au primit diplome din partea


American Mathematics Competitions
AMC 12,AIME
Ungureanu Andrei Bogdan
Carapencea Constantin
Pasov Iulia
Mateisescu Alexandru
Meilescu Simona
Sfetcu Adina
Rapcea Mihai
Chilom George
Turturea Roxana
Rosu Stefan
Croitoru Razvan
Golenteanu Magdalena
Achimescu Andreea
Togoe Bogdan
Bazavan Cristina
Bocse Bogdan
Bejinaru Amalia
Dagadita Monica

239p
168p
159,5p
159p
159p
155,5p
155p
152,5p
150,5p
150p
148p
141p
140p
138p
135,5p
134,5p
122p
122p

CNT
CNT
CNT
CNT
CNT
CNT
CNT
CNT
CNT
CNT
CNT
CNT
CNT
CNT
CNT
CNT
CNT
CNT

prof. Prajea Manuela


prof. Paponiu Dana
prof.Popescu Eleodor
prof. Giugiuc Leonard
prof. Paponiu Dana
prof. Paponiu Dana
prof. Paponiu Dana
prof.Popescu Eleodor
prof. Gimoiu Iuliana
prof. Paponiu Dana
prof. Paponiu Dana
prof. Prajea Manuela
prof. Prajea Manuela
prof. Prajea Manuela
prof. Prajea Manuela
prof. Paponiu Dana
prof. Paponiu Dana
prof. Paponiu Dana

210p

CNT

prof. Cainiceanu Gheorghe

21p
17p
17p
16p

CNT
CNT
CNT
CNT

prof. Paponiu Dana


prof. Paponiu Dana
prof. Paponiu Dana
prof. Paponiu Dana

AMC 10, AIME


Tigora Andrei
AMC 8
Carapencea Constantin
Andreca Mihai
Pit-Rada Andrei
Capraru Alexandru

Duta Adrian
Zorocliu Andra
Agape Mihai
Grosu Vlad
Asproniu Robert
Vasiu Mihnea
Prunescu Flavius
Boescu Anca
Lungu Amelin

16p
16p
15p
13p
10p
10P
10p
9p
9p

CNT
CNT
CNT
CNT
CNT
CNT
CNT
CNT
CNT

prof. Paponiu Dana


prof. Paponiu Dana
prof. Paponiu Dana
prof. Cainiceanu Gheorghe
prof. Cainiceanu Gheorghe
prof. Antonie Rodica
prof. Cainiceanu Gheorghe
prof. Cainiceanu Gheorghe
prof. Cainiceanu Gheorghe

CALIFICATI LA BARAJUL NATIONAL CANGURUL (clasele VII-XII)


Cepesi Cristian
Decebal VII Bobiti Ruxandra
Titeica
XI
Carapencea Constantin
CNT
VIII Rosu Stefan
CNT
XI
Croitoru Razvan
CNT
VIII Bazavan Eduard
Titeica
XII
Sosu Cristian
CNT
VIII Ungureanu Andrei
CNT
XII
Stretcu Otilia
Titeica VIII Enescu Flavius
CNT
XII
Cretu Andrei
CNT
X
Chilom George
CNT
XII
Coanda Oana
CNT
X
Priboi Cristian
CNT
XII
Elevi calificati la Concursul anual al rezolvitorilor Gazetei Matematice,
Pitesti , august 2006
Pit-Rada Andrei
VIII CNT
Raveanu Ioana
X
CNT

Concursuri
Olimpiada de matematic
- etapa judeean 11.03.2006 -

Clasa a V-a
Numele i prenumele
rlui Valeria
Bcanu Alexandru
Pirici Giulia
Pristoleanu Narcis
tefan Andrei
Gimoiu Ruxandra
Sceanu Andrei
Sbrcea Alexandra
Mitran Drago
Nicolae Andrei
Zamfirescu Simona
Rducu Andreea
Anghel Dumitru
Viian Elena Sorina
Florescu Alexandru
Ghiu Alexandru
Marghescu Andreea
Voicu Rzvan
Drig Darie
Bobei Paula
Georgescu Ana
olea Cristian

Scoala
ieica
CNT
ieica
CNT
CNT
CNT
CNT
CNT
CNT
CNT
CNT
CNT
c.Vj.Mare
CNT
ieica
ieica
CNT
CNT
CNT
c.14
CNT
ieica

Pr.
I
II
III
III
M
M
M
M
M
M
M
M
M
M
M
M
M
M
M
M
M
M

Profesor
Ttucu Mariana
Gimoiu Iuliana
Ttucu Mariana
Prajea Manuela

Numele i prenumele
Badea Beatrice
Nistor Oana
Safta Roxana
Tnasie Denisa
Rapcea Rzvan Lucian
Bue tefan
Ctnescu Raluca
Filip Radu
Hinoveanu Elena
Negrea Gabriela
Nu Flavius Andrei
Pcurariu Adrian
Pvlac Bogdan
Arbnai Emil Marius
Axinte Dana
Btmlai Adrian
Calafeteanu Liviu
Doan Cristina
Mohora Eduard
Nicola Silviu
Anghel Cosmin

Scoala
CNT
ieica
ieica
CNT
Lic.Vj.Mare
c.14
c.2
CNT
CNT
ieica
CNT
c.Orova
ieica
ieica
c.2
c.15
c.C.Negreanu
ieica
Lic.Odobleja
CNT
c.Vj.Mare

Pr.
M
M
M
M
M
M
M
M
M
M
M
M
M
M
M
M
M
M
M
M
M

Scoala
c.2
c.14
c.2
c.2
c.2
CNT
CNT
c.2
CNT
c.Strehaia
c.14
c.2
CNT
Lic.Paulian
CNT
CNT
CNT
CNT

Pr.
I
II
III
III
III
III
M
M
M
M
M
M
M
M
M
M
M
M

Profesor
Coada Carmen
Ionica C-tin
Coada Carmen
Coada Carmen
Coada Carmen
Cainiceanu Gh.

Numele i prenumele
Sprlea Octavian
Lungu Amelin
Nichielea Teodor
Diaconeasa Lucian
Pacioag Florentina
Grecu Anda Teodora
Mucic Alexandru
Croitoru Ioana Andra
Mavlea Ionona
Rdulescu Silvia
Andreescu Mdlina
Andrei Lavinia
Belcea Manuela
Clain Andrei
Drguin Costinel
Lupiu Gabriela
Srbu Adrian Mihai
into Mdlina

Scoala
c.14
CNT
c.6
c.2
c.Sergescu
ieica
c.2
c.2
c.Vj.Mare
c.6
CNT
Lic.Odobleja
CNT
CNT
c.5
CNT
ieica
CNT

Pr.
M
M
M
M
M
M
M
M
M
M
M
M
M
M
M
M
M
M

Clasa a VI-a
Numele i prenumele
Fril Remus
Nistor Mihaela
Constantinescu Robert
Guran Maria Mihaela
Toader Simona
Grosu Vlad
Asproniu Robert
Drghia Beatrice
Zugravu Rozalia
Bu Alexandra
Niulescu Maria
Odor Cristina
Roca Ioana
Caragea Flaviu
Drghici Rzvan
Crac Mihai
Mitroi Roxana
Prunescu Flavius

- 62 -

Concursuri

SSM
H

Clasa a VII-a
Numele i prenumele
Teil Bianca
Radu Cornel
Arcu Ana Maria
Mzi Adelina
Gugulici Alexandra
Andreca Mihai
Caplea Luminia
Cepei Cristian
Cheredi Ioan
Doroiman Costinela
uculanu Andreea
Ganolea Adina
Poenaru Alexandra
Popa Bogdan
Booac Mdlina
Budu Anca
Copceanu Adela
Milici Alina Simona

Scoala
ieica
ieica
c.14
ieica
c.2
CNT
CNT
C.Decebal
ieica
Lic.Odobleja
CNT
ieica
c.14
c.14
c.2
c.6
c.2
c.5

Pr.
I
II
III
III
III
M
M
M
M
M
M
M
M
M
M
M
M
M

Profesor
Zaman Irina
Zaman Irina
Palasca V.
Ungureanu O.
Coada Carmen

Numele i prenumele
Papa Florin
Soare Rzvan
Stnioar Sarah
tefan Radu
Tuce Ovidiu
Popescu Edward
Amza Loredana
Andriuc Adina
Boronti Elena
Cilibiu Andrei
Crc Elena
Minescu Mdlina
Pais Amalia
Papa Andreea Mdlina
Pcal Rzvan
Tudosie Claudia
Zorocliu Andra

Scoala
CNT
c.Orova
CNT
ieica
ieica
CNT
ieica
CNT
c.2
c.11
Gr.c.Dl.Tudor
CNT
c.2
CNT
ieica
ieica
CNT

Pr.
M
M
M
M
M
M
M
M
M
M
M
M
M
M
M
M
M

Scoala
ieica
CNT
CNT
CNT
CNT
CNT
CNT
CNT
ieica
CNT
c.2
ieica
c.2

Pr.
I
II
II
III
M
M
M
M
M
M
M
M
M

Profesor
Stretcu Dan
Paponiu Dana
Paponiu Dana
Paponiu Dana

Numele i prenumele
Marin Andreca Cristina
Huza Alexandru
Nef Nicoleta
Rdoi Ana Maria
Rdu Marian
Brgaru Evelina
Ciuciu Lucian Florin
Crng Larisa
Durac Raluca
Miloevici Larisa
Nu Lucian Bogdan
Torsin Ligia

Scoala
ieica
CNT
c.2
ieica
c.2
ieica
ieica
Lic.Odobleja
c.2
Lic.Odobleja
ieica
c.Orova

Pr.
M
M
M
M
M
M
M
M
M
M
M
M

Clasa a VIII-a
Numele i prenumele
Stretcu Otilia
Carapencea Constantin
Croitoru Rzvan
Pi Andrei
Calinovici Paul
Ecobici Gabriela
Nicoar Clin
Du Adrian
Orbu Alexandru
eitan Mihaela
Iorga Cristian
Murean Bogdan
Chelu Adina

Clasa a IX-a
Numele i prenumele
Putan Ctlin
Nicolescu Alexandra
Dunreanu Lidia
Nistor Ovidiu
tefnoiu Anca
Popescu Alina
Mutu Marie-Jeanne
Purcaru Alin
Butaru Nicu
Echim Ioana
Picu Vulpain Bianca

Scoala
ieica
Cnt
ieica
Cnt
Cnt
Cnt
Cnt
ieica
Cnt
Cnt
Cnt

Premiul
I
II
III
III
III
M
M
M
M
M
M

Profesor
Stretcu Daniel
Gimoiu Iuliana
Stretcu Daniel
Cainiceanu Gheorghe
Cainiceanu Gheorghe

Numele i prenumele
Tuce Andreea
Bechir Adriana
Trancot Ioan Andrei
Nica Flavius
Oprea Radu
Rducu Alexandru
Robu Alexandru
Ciouca Eugen
Creu Adina
Lincan Dan

Scoala
ieica
Cnt
Vnju-Mare
Cnt
Cnt
Cnt
Cnt
Cnt
ieica
Cnt

Premiul
M
M
M
M
M
M
M
M
M
M

Scoala
CNT
CNT
CNT
CNT
ieica
CNT
CNT
ieica

Premiul
I
II
II
II
M
M
M
M

Profesor
Cainiceanu Gheorghe
Cainiceanu Gheorghe
Cainiceanu Gheorghe
Giugiuc Leonard

Numele i prenumele
Rveanu Ioana
Suselea Robert
Gal Raluca
Belbu Loredana
Mariescu Radu
Prundeanu Andreea
Sandu Oana
Jiplea Bogdan

Scoala
CNT
CNT
CNT
CNT
CNT
CNT
CNT
CNT

Premiul
M
M
M
M
M
M
M
M

Clasa a X-a
Numele i prenumele
Tigora Andrei
Coand Oana
Tuta Leontin
Ciocea Marina
Gogoloiu Gabriela
Porojean Otilia
Rachieru Adrian
Rdoi Alexandra

- 63 -

Concursuri
Clasa a XI-a
Numele i prenumele
Rapcea Mihai
Rou tefan
Bobii Ruxandra
Dgdi Monica
Mare Alexandra
Gintaru Silviu
Strezariu Iulia
Bzdoac Mihai
Voichia Viorel

Scoala
CNT
CNT
ieica
CNT
CNT
Vnju-Mare
ieica
CNT
Strehaia

Premiul
I
I
II
M
M
M
M
M
M

Profesor
Paponiu Dana
Paponiu Dana
Popescu Rodica

Numele i prenumele
Ivanovici Daniela
Sbrcea Rzvan
Viau Mihai
Pleu Georgiana
Plotogea Ctlin
Ungureanu Ionu
Mnescu Teodora
Popescu Cristina
Turturea Roxana

Scoala
ieica
CNT
Odobleja
Strehaia
CNT
ieica
CNT
CNT
CNT

Premiul
M
M
M
M
M
M
M
M
M

Numele i prenumele
Chilom George
Cretescu Oana
Luca Alexandru
Mare Flavia
Stoica George

Scoala
CNT
CNT
CNT
CNT
CNT

Premiul
M
M
M
M
M

Clasa a XII-a
Numele i prenumele
Ungureanu Andrei
Bzvan Eduard
Paov Iulia
Mirea Theodor
Tirtea Gabriela
Janca Andrei

Scoala
CNT
ieica
CNT
CNT
CNT
ieica

Premiul
I
I
II
M
M
M

Profesor
Prajea Manuela
Stretcu Daniel
Popescu Eleodor

Rezultate obtinute de elevii mehedinteni la concursuri de


matematica in anul 2006
Olimpiada Internaional de Matematic Liubliana Slovenia
Ungureanu Andrei Bogdan
Bazavan Eduard

medalie de aur
medalie de aur

prof. Prajea Manuela


prof. Stretcu Daniel

Olimpiada Balcanic de Matematic - Cipru


Bazavan Eduard

medalie de aur

prof. Stretcu Daniel

Olimpiada Nationala
Ungureanu Andrei Bogdan
Bazavan Eduard
Nicolescu Alexandra
Tesila Bianca
Carapencea Constantin
Tigora Andrei
Pasov Iulia

premiul I
medalie de aur SSM
premiul al III-lea
medalie de aur SSM
mentiune
medalie de argint
medalie de bronz SSM
medalie de bronz SSM
medalie de bronz SSM
medalie de bronz SSM

prof Prajea Manuela


prof. Stretcu Daniel
prof. Gimoiu Iuliana
prof. Zaman Irina
prof. Paponiu Dana
prof. Cainiceanu Gheorghe
prof.Popescu Eleodor

CONCURSUL NATIONAL AL. MYLLER, Iasi


Croitoru Razvan
Carapencea Constantin
Pit-Rada Andrei

premiul al III-lea
mentiune
mentiune

prof. Paponiu Dana


prof. Paponiu Dana
prof. Paponiu Dana

CONCURSUL INTERJUDETEAN GH.TITEICA, Craiova


Pit-Rada Andrei
Tigora Andrei

- 64 -

mentiune
mentiune

prof.Paponiu Dana
prof.Cainiceanu Gheorghe

Concursuri
CONCURSUL INTERJUDETEAN ION CIOLAC , Craiova
Rapcea Mihai

premiul I

prof.Paponiu Dana

SSM
H

CONCURSUL NATIONAL SCOALA CU CEAS, Rm. Vilcea


Proba individuala
Sbarcea Alexandra
Gimoiu Ruxandra
Andreescu Madalina
Mitroi Roxana
Asproniu Robert
Nicoara Calin
Seitan Mihaela
Duta Adrian

premiul al III-lea
premiul al III-lea
premiul al III-lea
mentiune
mentiune
mentiune
mentiune
mentiune

prof. Prajea Manuela


prof. Gimoiu Iuliana
prof. Antonie Rodica
prof. Cainiceanu Gheorghe
prof. Cainiceanu Gheorghe
prof. Paponiu Dana
prof. Paponiu Dana
prof. Paponiu Dana

premiul al III-lea
premiul al II-lea
premiul al III-lea
premiul al II-lea

prof. Prajea Manuela


prof. Cainiceanu Gheorghe
prof. Paponiu Dana
prof. Paponiu Dana

Proba La ceas
Voicu Razvan
Grosu Vlad
Papa Florin
Nicoara Calin

Concursul Kangourou
CALIFICATI LA BARAJUL NATIONAL CANGURUL (clasele VII-XII)
Cepesi Cristian
Carapencea Constantin
Croitoru Razvan
Sosu Cristian
Stretcu Otilia
Cretu Andrei
Coanda Oana

Decebal
CNT
CNT
CNT
Titeica
CNT
CNT

VII
VIII
VIII
VIII
VIII
X
X

Bobiti Ruxandra
Rosu Stefan
Bazavan Eduard
Ungureanu Andrei
Enescu Flavius
Chilom George
Priboi Cristian

Titeica
CNT
Titeica
CNT
CNT
CNT
CNT

XI
XI
XII
XII
XII
XII
XII

BARAJUL NATIONAL CANGURUL (clasele VII-XII), Bucuresti


Numele i prenumele Scoala
Profesor
Cl. punctaj
Premiul
Cepesi Cristian
Sosu Cristian
Carapencea C-tin
Croitoru Razvan
Rosu Stefan Lucian
Bazavan Eduard

CNT
CNT
CNT
CNT
CNT
ieica

PupazaEcaterina
Paponiu Dana
Paponiu Dana
Paponiu Dana
Paponiu Dana
Stretcu Daniel

7
8
8
8
11
12

115.5
118
110
102.75
75
122.5

Premiul II - Tabara Poiana Pinului


Premiul II - Tabara Poiana Pinului
Premiul II - Tabara Poiana Pinului
Premiul III
Premiul III
Premiul I - Excursie Franta

BARAJUL JUDETEAN CANGURUL (clasele V-VI)


Clasa a II-a
Clasa a V-a
Popescu Maria
Popescu Cristiana
Vitus Eduard
Popescu Simona

Sc.3
Lic.Paulian
Sc.13
Sc. 2

Clasa a III-a
Prencea Cassian
Marghescu Gabriel
Dirpes Iulian
Marculescu Cristina

Lic.Paulian
Sc. 2
Sc. 13

Clasa a IV-a
Dragomir Andrei
Mema Serban
Floarea Ionut
Mihutescu Diana
Stoichita Andreea
Ciuta Cora

Lic.Paulian
Lic.Paulian
Strehaia
Strehaia
Strehaia
Lic.Paulian

Doana Cristina
Rapcea Lucian
Baluta Adrian
Stanciu Alexandru
Arbanasi Emil
Nuta Flavius
Nicola Silviu
Borcean Diana
Ciontea Stefan
Stefan Andrei

Vinju Mare
C.N.Traian
Gen.6
Gh.Titeica
C.N.Traian
C.N.Traian
Severinesti
Gh.Titeica
C.N.Traian

Clasa a VI-a
Grosu Vlad
Clain Andrei Catalin
Rosca Ioana
Pirvulescu Eleodor
Ciocoteala Alexandru
Gevelica Simon
Tudosie Dorin

C.N.Traian
C.N.Traian
C.N.Traian
C.N.Traian
C.N.Traian
C.N.Traian

- 65 -

Concursuri
Elevi care au primit diplome din partea
American Mathematics Competitions

AMC 12,AIME
Ungureanu Andrei Bogdan
Carapencea Constantin
Pasov Iulia
Mateisescu Alexandru
Meilescu Simona
Sfetcu Adina
Rapcea Mihai
Chilom George
Turturea Roxana
Rosu Stefan
Croitoru Razvan
Golenteanu Magdalena
Achimescu Andreea
Togoe Bogdan
Bazavan Cristina
Bocse Bogdan
Bejinaru Amalia
Dagadita Monica

239p
168p
159,5p
159p
159p
155,5p
155p
152,5p
150,5p
150p
148p
141p
140p
138p
135,5p
134,5p
122p
122p

CNT
CNT
CNT
CNT
CNT
CNT
CNT
CNT
CNT
CNT
CNT
CNT
CNT
CNT
CNT
CNT
CNT
CNT

prof. Prajea Manuela


prof. Paponiu Dana
prof.Popescu Eleodor
prof. Giugiuc Leonard
prof. Paponiu Dana
prof. Paponiu Dana
prof. Paponiu Dana
prof.Popescu Eleodor
prof. Gimoiu Iuliana
prof. Paponiu Dana
prof. Paponiu Dana
prof. Prajea Manuela
prof. Prajea Manuela
prof. Prajea Manuela
prof. Prajea Manuela
prof. Paponiu Dana
prof. Paponiu Dana
prof. Paponiu Dana

AMC 10, AIME


Tigora Andrei

210p

CNT

prof. Cainiceanu G.

AMC 8
Carapencea Constantin
Andreca Mihai
Pit-Rada Andrei
Capraru Alexandru
Duta Adrian
Zorocliu Andra
Agape Mihai
Grosu Vlad
Asproniu Robert
Vasiu Mihnea
Prunescu Flavius
Boescu Anca
Lungu Amelin

21p
17p
17p
16p
16p
16p
15p
13p
10p
10p
10p
9p
9p

CNT
CNT
CNT
CNT
CNT
CNT
CNT
CNT
CNT
CNT
CNT
CNT
CNT

prof. Paponiu Dana


prof. Paponiu Dana
prof. Paponiu Dana
prof. Paponiu Dana
prof. Paponiu Dana
prof. Paponiu Dana
prof. Paponiu Dana
prof. Cainiceanu G.
prof. Cainiceanu G.
prof. Antonie Rodica
prof. Cainiceanu G.
prof. Cainiceanu G.
prof. Cainiceanu G.

Elevi calificati la Concursul anual al rezolvitorilor Gazetei Matematice,


Pitesti , august 2006
Pit-Rada Andrei
VIII
CNT
Raveanu Ioana
X
CNT

- 66 -

REZOLVITORI

SSM
H

Colegiul National Traian


Prof. Gh.Cainiceanu - clasa a VI-a Pavel Cristian [8], Mituca Anda [8],
Pisleaga Anemona [8], Toma Octavia [8], Oprita Geanina [8],Capitanescu
Madalina [8], Mitroi Roxana [8], Boescu Anca [8] - clasa a IX-a Tanasescu
Vlad [10] - clasa a X-a Susnea George [10], Stavaru Daniel [10], Cretu
Andrei [10], Bojinovici Sergiu [10], Badea Sabin [10], Ghiga Octavian [10],
Prundeanu Andreea [10], Pasat Cristina [10], Vladu Margareta [10], Tuta
Leontin [10], Modalca Denisa [10], Georgescu Alin [10], Ionica Adina [10],
Porojan Otilia [10], Gamala Andreea [10], Mariescu Radu [10], Gociu Andrei
[10], Rachieru Adrian [10], Cotet Alexandra [10], Abagiu Aurel [10], Cucu
Irina [10], Cristea Valentina [10], Lupu Olivia [10], Trasca Mihai [10], Negrea
Nicolae [10], Saftoiu Mihai [10], Buncianu Ilie [10], Marin Daniela [10],
Stoica Raluca [10], Hanes Anca [10], Pitulicu Lorena [10], Dogaru Oana [10],
Ganda Irina [10], Cainiceanu Andrei [10], Urse Ruxandra [10],Hinoveanu
Catalin [10], Belbu Loredana [10], Mituca Atena [10], Tomescu Liana [10],
Nitulescu Maria [10].

Liceul Teoretic Gh. ieica


Prof. Irina Zman clasa a V-a Doan Cristina [17], Fonea Dan [5], Fonea
Alex [5], Florescu Alex [6], Sbiean Roxana [5], Arbnai Emil [7], Ghiu
Alex [11], Mituleu Dana [4], Safta Roxana [5] clasa a VII-a Teil Bianca
[9], Mur Ion [5], Cheredi Ion [6], tefan Radu [6], Botoanu Cornel [6],
Troncu Paul [6].
Scoala general Nr.11
Prof. Victor Saceanu - clasa a-V-a Bica Claudia; Cafadaru Emanuel;
Camenita Daniel; Matinescu Ionut; Scarlat Liliana; Stanciulescu Sidonica;
Ticana Georgiana; Mihaescu Catalin; Matu Diana - clasa a-VI-a Covresvu
Laurentiu; Cilihiu Claudiu; Gavrila Costinel; Iancu Nicusor; Dumitrascu
Ramona; Popescu Laurentiu clasa a-VII-a Buzatu Damian; Carjoi
Valentin; Cucu Bianca; Ghita Gabriela; Mladin Elvira; Munteanu Stefan;
Sarbu Valentina - clasa a-VIII-a Avramescu Alexandra; Bacioiu Loredana;
Cimpoieru Diana; Mosneanu Angela; Roman Georgiana; Zaharescu Victor.

- 67 -

COLABORATORI

prof. Daniel Sitaru


prof. Alexandru Szoros
prof. Angela Nioiu
prof. Antonie Rodica
prof. Bloi Valeria
prof. Calafeteanu Gheorghe
prof. Dan Daniel
prof. Dan Nedeianu
prof. Daniel Sitaru,
prof. Draga Ttucu Mariana
prof. Draga Ttucu Porfirel
prof. Eleodor Popescu
prof. Florentina Roman
prof. Gheorghe Cainiceanu
prof. Giugiuc Constantin
prof. Gorun Sanda
prof. Ion Chilea
prof. Ionel-Vasile Pi-Rada
prof. Iuliana Gimoiu
prof. Leonard Giugiuc
prof. Lucian Dragomir

- 68 -

prof. Manuela Opria


prof. Manuela Prajea
prof. Marica Pi-Rada
prof. Nedeianu Dan
prof. Ovidiu Ticusi
prof. Paponiu Dana
prof. Popescu Octavian Tudor
prof. Stefan Marica
prof. Stoican Victor
prof. Stretcu Daniel
prof. Victor Saceanu
inst. Maria Ungureanu
inst. Mariana Cornea
inst. Niculina Opria
drd. Laviniu Bejenaru
elev Nol Negrea
elev Aida Giurcan
elev Otilia Stretcu
elev Rapcea Mihai
elev Boce Bogdan
drd. Iuliana Radu

REZOLVITORI

SSM
H

Colectivul de redacie:
Gheorghe Ciniceanu
Dan Daniel
Dana Paponiu
Octavian Ungureanu
Popescu Eleodor
Sceanu Victor

Manuela Prajea
Nnui Dan
Dan Nedeianu
Sitaru Dan
Chilea Ion

Decembrie 2006

- 69 -

Das könnte Ihnen auch gefallen